一轮精品 2014高考语文一轮测评手册福建专用 :阶段评估检

阶段评估检测(一)

满分150分,考试时间150分钟。

一、古代诗文阅读(27分)

(一)默写常见的名句名篇(6分)

1.补写出下列名句名篇中的空缺部分。

(1)伤心秦汉经行处,__________________。(张养浩《山坡羊·潼关怀古》)

(2)______________,风流总被雨打风吹去。(辛弃疾《永遇乐·京口北固亭怀古》)

(3)__________________,锦鳞游泳。(范仲淹《岳阳楼记》)

(4)__________________,子无良媒。(《诗经·氓》)

(5)连峰去天不盈尺,__________________。(李白《蜀道难》)

(6)钉头磷磷,__________________。(杜牧《阿房宫赋》)

(二)文言文阅读(15分)

阅读下面的文言文,完成2~5题。

武夷精舍记

[宋]韩元吉

武夷在闽粤直北,其山势雄深磅礴。自汉以来,见于祀事,闽之诸山皆后出也。其峰之最大者,丰上而敛下,巍然若巨人之戴弁。缘隙磴道可登,世传避秦而仙者蜕骨在焉。溪出其下,绝壁高峻,皆数十丈,岸侧巨石林立,磊落奇秀。好事者一日不能尽,则卧小舟抗溪而上,以左右顾视。至其地,或平衍,景物环会,必为之停舟,曳杖徙倚而不忍去。山故多王孙。鸟则白鹇、鹧鸪,闻人声,或磔磔集崖上,散漫飞走,而无惊惧之态。水流有声,草木四时敷华。道士即溪之六曲仅为一庐,以待游者之食息。往往酌酒未半,已迫曛莫而不可留矣。

吾友朱元晦居于五夫里,在武夷山一舍,而近若其后圃,暇则游焉。与其门生弟子挟书而诵,取古诗三百篇及楚人之词,哦而歌之,潇洒啸咏,留必数日。盖山中乐悉为元晦之私也,予每愧焉

淳熙十年,元晦既辞使节于江东,遂赋祠官之禄,则又曰:“吾今营其地,果尽有山中之乐矣。”盖其游益数,而于其溪之五折,负大石屏,规之以为精舍,取道士之庐犹半也。诛锄茅草,仅得数亩。面势幽清,奇石佳木,拱揖映带。使弟子辈具畚锸,集瓦木,相率成之。元晦躬画其处,中以为堂,旁以为斋,高以为亭,密以为室,讲书、肄业、琴歌、酒赋,莫不在是。予闻之,恍然如寐而醒,醒而后,隐隐犹记其地之美也。且曰:“其为我记。”

夫元晦,儒者也。方以学行其乡,善其徒。非若畸人隐士遁藏山谷,服气茹芝,以慕夫道家者流也。然秦汉以来,道之不明久矣。吾夫子所谓志于道,亦何事哉?夫子,圣人也,其步与趋莫不有则。至于登泰山之巅而诵言于舞雩之下,未当不游。胸中盖自有地,而一时弟子鼓瑟铿然,“春服既成”之咏,乃独为圣人所予。古之君子息焉、游焉,岂以是拘拘乎?

元晦既有以识之,试以告夫来者,相与酬酢于精舍之下,俾或自得。其幔亭之风,抑又何如也?

是岁八年,颍川韩元吉记。

(有删改)

〔注〕 ①弁:帽子。②王孙:猴的别称。③华:通“花”。④舍:古代计量单位,一舍为三十里。⑤赋祠官之禄:宋代优待官吏,官吏脱离实际职务,可以挂名掌管某祠,享受俸禄。⑥“春服既成”之咏:语见《子路曾皙冉有公西华侍坐》,孔子与弟子谈志向,曾皙鼓瑟,咏唱“春服既成”,其思想得到了孔子的赞同。⑦幔亭之风:相传秦始皇有仙人武夷君,中秋节在武夷山设幔亭一座,宴请山下的乡人。

2.对下列加点词的解释,不正确的一项是(3分)( )

A.已迫曛莫而不可留矣 莫:通“暮”,天黑 B.规之以为精舍 规:规划

C.元晦躬画其处 躬:亲自 D.善其徒 善:好的

3.下列全都是描写“武夷精舍”环境与布置的一组是(3分)( )

①仅为一庐,以待游者之食息 ②于其溪之五折,负大石屏 ③岸侧巨石林立,磊落奇秀 ④奇石佳林,拱揖映带⑤中以为堂,旁以为斋

A.①③⑤ B.②④⑤ C.③④⑤ D.①②④

4.下列对文章有关内容的概括与分析,不正确的一项是(3分)( )

A.武夷山山势宏伟,山形奇特,历史悠久,充满神话色彩,令人流连忘返,但一日之游时间仓促,游览者往往匆匆而返。

B.因居住地距武夷山较近,朱元晦与门徒常携书前往,颂诗经楚辞,饮酒长啸。辞官后,享受祠官的俸禄,更能常游武夷山水。

C.武夷精舍面溪背石,是朱元晦与门徒亲自建造出来的,其规模比邻近的道庐大了一倍多,而且各功能区域划分明确。

D.朱元晦是一介儒生,心中有“道”,虽居深山,却不行寻仙访道、炼丹吃灵芝之举,而是效仿孔子收徒讲学,逸情于山水。

5.把文中画横线的句子翻译成现代汉语。(6分)

(1)盖山中乐悉为元晦之私也,予每愧焉。(3分)

译文:________________________________________________________________________

(2)元晦既有以识之,试以告夫来者。(3分)

译文:________________________________________________________________________

(三)古代诗歌阅读(6分)

6.阅读下面这首诗歌,回答问题。

野 望

[唐]杜甫

西山白雪三城戍,南浦清江万里桥。海内风尘诸弟隔,天涯涕泪一身遥。

惟将迟暮供多病,未有涓埃答圣朝。跨马出郊时极目,不堪人事日萧条。

〔注〕 ①这首诗作于上元二年(761)成都草堂。②三城戍:当时因受吐蕃侵扰,曾在松、维、保三城设戍。

(1)这首诗以“野望”为题,请具体说明全诗是如何扣住“望”来写的。(3分)

答:________________________________________________________________________

(2)这首诗情感丰富,请作具体分析。(3分)

答:________________________________________________________________________

二、文学名著、文化经典阅读(16分)

(一)文学名著阅读(10分)

7.下列各项中对作品故事情节的表述,不正确的两项是(5分)( )

A.高老太爷病重,生命垂危。在陈姨太的提议下,高家开始为高老太爷捉鬼治病,面对这场闹剧,觉慧无畏地戳穿了它。(《家》)

B.吴老太爷过世,双桥镇农民暴动,吴荪甫为此惊慌失措,打算赶回老家。此时,丝厂总管屠维岳又来向他报告说,丝厂工人罢工了。(《子夜》)

C.爱斯梅拉达被法庭判处死刑。执行死刑的那天,隐修女居第尔思念女儿的痛苦比往常发作得更厉害。当她听说要处死一个埃及姑娘时,她幸灾乐祸地大笑起来。(《巴黎圣母院》)

D.涅赫柳多夫去彼得堡之前,交给马斯洛娃一个信封,里边装着他从姑妈庄园带回来的照片,马斯洛娃见到它很开心。(《复活》)

E.查理发家后把欧也妮忘得一干二净,但为了体面地和美丽的侯爵小姐订婚,查理哄骗善良的欧也妮偿还了自己父亲的债务。(《欧也妮·葛朗台》)

8.简答题。(选做一题)(5分)

(1)阅读下面的《红楼梦》选段,根据原著故事情节,回答问题。

周瑞家的听说,会齐了那几个媳妇,先到迎春房里,回迎春道:“太太们说了,司棋大了,连日他娘求了太太,太太已赏了他配人,今日叫他出去,另挑好的与姑娘使。”说着,便命司棋打点走路。

司棋为何会被赶走?对此,迎春态度如何?司棋回家之后,又发生了什么事?请简述。

(2)阅读下面的《三国演义》选段,根据原著故事情节,回答问题。

统拜别,至江边,正欲下船,忽见岸上一人,道袍竹冠,一把扯住统曰:“你好大胆!黄盖用苦肉计,阚泽下诈降书,你又来献连环计:只恐烧不尽绝!你们把出这等毒手来,只好瞒曹操,也须瞒我不得!”得庞统魂飞魄散。

识破庞统连环计的人是谁?他是如何来到曹营的?后来他是如何避开赤壁大战的?请简述。

答第()题:________________________________________________________________________

(二)文化经典阅读(6分)

9.阅读下面的《论语》和《孟子》选段,回答问题。

有子曰:“信近于义,言可复也。恭近于礼,远耻辱也。”

子曰:“……道之以德,齐之以礼,有耻且格。”

孟子曰:“人不可以无耻,无耻之耻,无耻矣。”

(1)根据上面材料,说说儒家认为怎样的行为是“无耻”的。(3分)

答:________________________________________________________________________

(2)结合上面文段,说说你是如何理解孔子及孟子的耻辱观的。 (3分)

答:________________________________________________________________________

三、现代文阅读(24分)

(一)论述类文本阅读(9分)

阅读下面的文字,完成10~12题。

论幽默

周国平

①幽默是凡人暂时具备了神的眼光,这眼光有解放心灵的作用,使人得以看清世间一切事情的相对性质,从而显示了一切执着态度的可笑。

②有两类幽默最值得一提。一是面对各种偶像尤其是道德偶像的幽默,它使偶像的庄严在哄笑中化作笑料。然而,比它更伟大的是面对命运的幽默,这时人不再是与地上的假神开玩笑,而是直接与天神开玩笑。一个在最悲惨的厄运和苦难中仍不失幽默感的人的确是更有神性的,他藉此而站到了自己的命运之上,并以此方式与命运达成了和解。

③幽默是心灵的微笑。最深刻的幽默是一颗受了致命伤的心灵发出的微笑。受伤后衰竭、麻木、怨恨,这样的心灵与幽默无缘。幽默是受伤的心灵发出的健康、机智、宽容的微笑。

④幽默是对生活的一种哲学式态度,它要求我们与生活保持一定距离,暂时以局外人的眼光来发现和揶揄生活中的缺陷。毋宁说,人这时成了一个神,他通过对人生缺陷的戏侮而暂时摆脱了这缺陷。它教不会、学不了。有一本杂志声称它能教人幽默,从而轻松地生活。我不曾见过比这更缺乏幽默感的事情。也许正由于此,女人不善幽默。

⑤幽默与滑稽是两回事。幽默是智慧的闪光,能博聪明人一笑。滑稽是用愚笨可笑的举止逗庸人哈哈。但舞台上的滑稽与生活中的滑稽又有别:前者是故意的,自知可笑,偏要追求这可笑的效果;后者却是无意的,自以为严肃正经,因而更可笑——然而只有聪明人能察觉这可笑。

⑥幽默和嘲讽都包含某种优越感,但其间有品位高下之分。嘲讽者感到优越,是因为他在别人身上发现了一种他相信自己绝不会有的弱点,于是发出幸灾乐祸的冷笑。幽默者感到优越,则是因为他看出了一种他自己也不能幸免的人性的普遍弱点,于是发出宽容的微笑。幽默的前提是有一种超脱的态度,能够俯视人间的一切是非,包括自己的弱点。嘲讽却是较着劲的,很在乎自己的对和别人的错。

(摘自2011年5月30日《法制资讯》,有删改)

10.下列关于“幽默”的理解,不符合作者原意的一项是(3分)( )

A.面对各种偶像尤其是道德偶像的幽默,是与地上的假神开玩笑。

B.面对命运的幽默比面对各种偶像的幽默更伟大,是因为具备了神的眼光。

C.一个在最悲惨的厄运和苦难中仍不失幽默感的人是直接与天神开玩笑。

D.幽默是心灵的微笑,一个人受伤后如果心灵衰竭、麻木、怨恨,便与幽默无缘。

11.作者认为幽默“教不会、学不了”的理由是什么?(2分)

答:________________________________________________________________________

12.本文论述的是“幽默”,作者为什么要在⑤⑥段谈到“滑稽”与“嘲讽”?(4分)

答:________________________________________________________________________

(二)文学类、实用类文本阅读(选考)(15分)

本题为选做题,请从甲乙两类文本中选定一类作答,不得再做另一类文本的题目。

(甲)文学类文本阅读(15分)

阅读下面的文字,完成13~15题。

贫困却不潦倒的风度

天 歌

张罗同学聚会时,我们这些混得不错的同学很自觉很有风格地把费用划为三等:混得一般的交300;混得不好的,一分钱都不用出;混得好的,聚会缺多少,补多少,兜底。大家图的就是个乐儿,不想让自家兄弟姐妹为几两银子犯难。

尽管这样,班长还是颇为担心地说:九度能来吗?

是啊,九度是那样好面子的一个人,这些年,先是在有着大好前途的机关里受了牵连下岗,后是爱人查出了淋巴癌,再后来跟人合伙开饭店,饭店没开起来,借来的钱却全让人卷走了。九度这些事都是在校友录上,同学们辗转相传的。九度也来校友录,留言却总是说些好事:姑娘考了第一名(尽管那只是小学二年级的期中考试);妻子终于不用再去化疗了。还有,他给报社提供的新闻线索被采纳了,人家给了50块钱。

有同学提议大家凑些钱帮帮九度,好歹阶级兄弟一场。却不想班长乘兴而去,败兴而归,他说:那家伙说他还想捐点钱出来给谁呢!人家的日子过得好着呢!

同学说:你看九度,日子都过成那样了,还是绷着,还装,死要面子活受罪。

是的,九度上学那会儿就是瘦死不倒架的那种人。按照规定,九度是可以申请贫困贷款的,他却硬撑着不肯,同学中一度流传着他的一句名言:把钱留给更困难的同学吧。并把这句话演变成各种版本:把不及格的机会留给渴望人生完美的同学吧!把失恋的痛苦留给没有痛苦过的漂亮女生吧!

那年冬天,东北那叫嘎嘎冷,连件旧棉衣都没有的九度走路全靠跑。他说:本来在零度边缘,这一跑,温度“噌”地蹿了九度。其实,九度说的是39度,不过冻得嘴一哆嗦,“30”就吞了下去。这话风行了起来,九度就叫了九度,并以迅雷不及掩耳之势淹没了他的本名。

聚会那天,九度还真来了。不光来了,还穿得特别利索。一身浅灰色的西装配了一条红格子领带,比好些混得不错的同学收拾得还像成功人士。有同学上去就是一拳,说:你个九度,不装能死啊?咱们哥们儿,谁还笑话你不成?

九度打着哈哈,跑前跑后张罗着。席间,大家喝酒聊天,牢骚怨气差点就把酒店的天花板顶起来:什么股票太没谱了,说套牢就套牢,十几万转眼就打了水漂儿;什么现在的孩子太难管了,钢琴说白买就白买,那玩意儿放家里死占地方,又不能当写字台使;什么现在的官场简直就没法混,大领导之间都别扭着,听谁的话站谁的队,有学问着呢!九度笑着给大家布菜、倒酒,一哥们儿喝得有点大,硬着舌头说:九度,你说你点背的,啥啥没赶上,要不咋也弄个副局干干。

我们都拦着那哥们儿不让他哪壶不开提哪壶,如果同学聚会整成富人对穷人的颐指气使就没劲了。没想到九度却端起酒杯,对大家说:“我九度这些年是遇到些事,但是,还好,都扛过来了。我正有个好消息要告诉大家呢,今年我闺女以全市第一名的成绩考上了省重点,学费全免。”我的天!我暗叫一声,给儿子找了4个补习老师,结果连市重点的门都没摸着。九度说:“我也重新上岗了,电视台聘我当特约记者,咱也混进新闻界了。还有,我爱人从得癌症到今天已经8年了,也就是说,我们每一天都在创造着生命的奇迹……”

九度一口干掉了杯里的酒,不知为什么,刚才那些牢骚满腹或者臭显摆的哥们儿都有点蔫,咋好像自己的日子过得没九度带劲呢?

聚会结束时,班长拿出了账单。账单上,九度的名字下面没有写交钱的金额,而是写着一行字:物品搬放、保管,接送……班长说:九度不肯白白占大家的便宜,他没交钱,一定要以别的方式补偿回来。

还有,他的那身西装是他从前在机关时做的,他的妻子最近很不好……

看着依旧忙忙碌碌送同学的九度,我说:换了我们,谁可以做到像九度那样贫穷却不潦倒,活得生气勃勃的呢?

是的,我们眼里要面子的九度,其实要的是一种有尊严的生活方式。贫穷不可怕,可怕的是看不到生活里的温暖,失去了对生活的热情。在这一点上,九度表现得相当有风度。

(选自《微型小说选刊》)

13. 下列对作品的概括和分析,不正确的两项是(5分)( )

A.小说第二段,班长的担心为后文故事情节的发展设置了悬念,激起了读者的阅读兴趣。

B.毕业后的九度生活境遇不佳,却在校友录上只报喜不报忧,并且拒绝大家的帮助,都是因为他死要面子,这为他后来的转变做了铺垫。

C.一句“把钱留给更困难的同学吧”被同学们演绎出了各种版本,这虽然是在开九度的玩笑,但玩笑中却包含着大家对九度“瘦死不倒架”的敬佩。

D.小说运用倒叙的叙述方式,肖像描写、动作描写、语言描写等描写手法,塑造了主人公九度的形象。

E.小说通过塑造九度这一人物形象,表现了人生中,重要的不是生活境遇,而是面对生活的态度这一主题,引发了我们对人生的思考。

14. 小说的主人公九度是一个什么样的形象?请结合文章内容分条陈述。(4分)

答:________________________________________________________________________

15. 结尾两段,有人说是画蛇添足,有人说必不可少,你的观点是什么?请结合全文进行探究。(6分)

答:________________________________________________________________________

(乙)实用类文本阅读(15分)

阅读下面的文字,完成13~15题。

回望钱学森

卞毓方

一次,是在中科院一位朋友的办公室。我去时,朋友在欣赏一卷《钱学森手稿》。这一套手稿,分两卷,五百多页,是从钱学森早期的手稿中遴选出来的。我拿过来翻了翻,与其说是手稿,不如说是艺术品。无论中文、英文,大字、小字,计算、图表,都工工整整,一丝不苟,连一个小小的等号,也长短有度,中规中矩。钱学森的手稿令我想到王羲之的《兰亭集序》、张择端的《清明上河图》,进而想到他的唯美人格。如是我闻:在美国期间,钱学森仅仅为了解决一道薄壳变形的难题,研究的手稿就累积了厚厚一大摞,在工作进展到五百多页时,他自我感觉是:“不满意!”直到八百多页时,才长舒一口气。他把手稿装进牛皮纸信封,在外面标明“最后定稿”,继而觉得不妥,又在旁边添上一句:“在科学上没有最后!”

对我来说,印象最为深刻的,是他如下的几句老实话。回顾学生时代,钱学森明白无误地告诉人们:“我在北京师大附中读书时算是好学生,但每次考试也就是八十多分;我考去上海交大,并不是第一名,而是第三名;在美国的博士口试成绩也不是第一等,而是第二等。”八十多分,第三名,第二等,这哪里像公众心目中的天才?然而,事实就是事实,钱学森,没有避讳,倒是轮到世人惊讶,因为他们已习惯了把大师的从前和卓越、优异画等号。

钱学森的天才是不容置疑的。麻省理工的学子曾对他佩服不已。有一回,钱学森正在黑板上解一道十分冗长的算式,有个学生问了另一个与此题目无关、但也十分困难的问题,钱学森起初不予理会,继续在四个十英尺长、四英尺宽的黑板上,写满了算式。“光是能在脑袋中装进那么多东西,就已经够惊人了,”一位叫做哈维格的学生回忆,“但是更令我们惊叹的是,他转过身来,把另一个复杂问题的答案同时也解答出来!他怎么能够一边在黑板上计算一个冗长算式,而同时又解决另一个同样繁复的问题,真是令我大惑不解!”

天才绝对来自于勤奋。钱学森在加州理工的一位犹太籍的校友回忆:“一个假日一大早,我在学校赶功课,以为全幢建筑物里只有我一个人,所以把留声机开得特别响。乐曲进入高潮时,有人猛力敲我的墙壁。原来我打扰到钱学森了。后来他送我最新论文,算是对曾经向我大吼大叫聊表歉意。”

关于归国后的钱学森,这里补充一个细节。你注意过钱学森的履历表吗?他先担任国防部五院院长,然后改任副院长。这事不合常规,怎么官越做越小,难道犯了什么错误?不是的。原来,钱学森出任院长时,只有45岁,年富力强,正是干事业的好时光。但是院长这职务是一把手。钱学森不想把精力耗费在琐事上,就主动打报告,辞去院长职务,降为副院长。这种胸怀与情操,很少有人能与之匹敌。

钱学森有着十分粗犷而任性的另一面。他当年的学生们回忆,他上课总要迟到几分钟,正当大家猜测他今天是否会缺席时,他快速冲进教室,二话不说,抓起粉笔就在黑板上写开了,直到用细小而工整的字迹填满所有的黑板为止。又有一次,一个学生举手说:“第二面黑板上的第三个方程式,我看不懂。”钱学森不予理睬。另一个学生忍不住问:“怎么,你不回答他的问题吗?”钱学森硬邦邦地说:“他只是在叙述一个事实,不是提出问题。”又有一次,一个学生问钱学森:“你刚才提供的方法是否万无一失?”钱学森冷冷地瞪了他一眼,说:“只有笨蛋才需要万无一失的方法。”期末考试,钱学森出的题目极难,全班差不多都吃了零蛋。学生有意见,找上级的教授告状。钱学森对此回答:“我又不是教幼儿园!这是研究所!”

还有更加不近人情的描述:钱学森在校园中是个神秘人物。除了上课,教师和学生都只偶尔在古根海姆大楼跟他擦肩而过。他总是把自己关在研究室里,学生跑去请教问题,他随便一句“看来没问题嘛”,就把他们打发走。有时他完全封闭自己,不论谁去敲门,哪怕是事先约好的,他也会大吼一声:“滚开!”

以上细节,恐怕都是真实的。大师就是大师,无一例外充满个性色彩,这样的大师也令人喜爱。

13.下列对文章的概括和分析,最恰当的两项是(5分)( )

A.“我”之所以看到钱学森的手稿就想到王羲之的《兰亭集序》、张择端的《清明上河图》,是因为它们都是非常珍贵的艺术品。

B.钱学森回国后曾担任国防部五院院长,但后来又主动请求辞去院长职务,担任副院长,其目的是有更多的时间从事科研工作。

C.“他快速冲进教室,二话不说,抓起粉笔就在黑板上写开了”是细节描写,体现了钱学森教学时珍惜时间的精神。

D.“大师就是大师,无一例外充满个性色彩,这样的大师也令人喜爱。”这句话表达了作者对一代大师钱学森完美无瑕的个性的赞美。

E.钱学森是我国航天事业的先驱,但本文却没有选取这方面的素材,而是独具匠心,另选素材,塑造了充满个性色彩的钱学森。

14. 这篇传记从哪些方面介绍了钱学森?请分条概述。(4分)

答:________________________________________________________________________

15.文中画线句子“对我来说,印象最为深刻的,是他如下的几句老实话”,语意丰富。请谈谈你的理解,并就一点谈谈给你的启示。(6分)

答:________________________________________________________________________

四、语言文字运用(13分)

16.阅读下面一段文字,按要求回答问题。(3分)

《冷浪漫》精选了“科学松鼠会”多位作者的作品,深入浅出地为读者讲述了与生命、生活密切相关,却□□□□(很少被人知道)的科学知识。它以通俗化的语

言,对那些“冷冰冰”的科学知识进行生动活泼的______② (quán)释,让深奥的理论一下子变得通俗易懂。

这些文章,既给人以科学的教益,也获得美的享受。

(1)请在①处根据“很少被人知道”这一意思,填写一个四字成语。(1分)

答:________________________________________________________________________

(2)请在②处根据拼音填写正确的汉字。(1分)

答:________________________________________________________________________

(3)③处有语病,请改正。(1分)

答:________________________________________________________________________

17. 提取下列材料中的要点,整合成一个单句,为“恶意软件”下定义。(3分)

①恶意软件是一种侵害用户合法权益的软件。

②恶意软件是在未明确提示用户的情况下强行安装的。

③恶意软件的安装未经用户许可,属于强行安装。

④恶意软件是在用户计算机或其他终端上安装运行的。

恶意软件是________________________________________________________________________

18.阅读下面的材料,回答问题。(7分)

近日看到一则消息,标题是“1500硕士竞聘卖猪肉岗位,据称包括海归人士”,言称如今严峻的经济形势下,别说大学生就连硕士博士就业也费劲,就有那么一家天才公司此时出了招聘广告,高薪之下竟引来无数硕士应聘,一下子火了。

对于上述材料,你有什么看法?请简要阐述。(要求:表达简明连贯,言之成理,150字左右)

答:________________________________________________________________________

________________________________________________________________________

________________________________________________________________________

五、写作(70分)

19.阅读下面的材料,根据要求写一篇不少于800字的议论文或记叙文。

一块冰落到了撒哈拉大沙漠上,冰感叹说:“沙漠是冰的地狱,北极才是冰的天堂。”沙对冰块说:“冰在沙漠里才最珍贵,冰在北极里是最不值钱的东西。”

要求:(1)必须符合文体要求;(2)角度自选;(3)立意自定;(4)题目自拟;(5)不得抄袭,不得套作。

全品高考网:www.canpoint.cn阶段评估检测(二) ◄测评手册

阶段评估检测(二)

满分150分,考试时间150分钟。

一、古代诗文阅读(27分)

(一)默写常见的名句名篇(6分)

1.补写出下列名句名篇中的空缺部分。

(1)小楼昨夜又东风,__________________。(李煜《虞美人》)

(2)人生自古谁无死?____________。(文天祥《过零丁洋》)

(3)______________________?只是当时已惘然。(李商隐《锦瑟》)

(4)斯是陋室,________________。(刘禹锡《陋室铭》)

(5)________________,虎啸猿啼。(范仲淹《岳阳楼记》)

(6)落红不是无情物,____________。(龚自珍《己亥杂诗》)

(二)文言文阅读(15分)

阅读下面的文言文,完成2~5题。

钱塘六井记

苏 轼

潮水避钱塘而东击西陵,所从来远矣。沮洳斥卤,化为桑麻之区,而久乃为城邑聚落,凡今州之平陆,皆江之故地。其水苦恶,惟负山凿井,乃得甘泉,而所及不广。唐宰相李公长源始作六井,引西湖水以足民用。其后刺史白公乐天治湖浚井,刻石湖上,至于今赖之。始长源六井,其最大者,在清湖中,为相国井,其西为西井,少西而北为金牛池,又北而西、附城为方井,为白龟池,又北而东至钱塘县治之南为小方井。而金牛之废久矣。嘉祐中,太守沈公文通又于六井之南,绝河而东至美俗坊为南井。出涌金门,并湖而北,有水闸三,注以石沟贯城而东者,南井、相国、方井之所从出也。若西井,则相国之派别者也。而白龟池、小方井,皆为匿沟湖底,无所用闸。此六井之大略也。

熙宁五年秋,太守陈公述古始至,问民之所病。皆曰:“六井不治,民不给于水。南井沟庳而井高,水行地中,率常不应。”公曰:“嘻,甚矣,吾在此,可使民求水而不得乎!”乃命僧仲文、子圭办其事。于是发沟易甃,完缉罅漏,而相国之水大至,坎满溢流,南注于河,千艘更载,瞬息百斛。疏涌金池为上中下,使浣衣浴马不及于上池。而列二闸于门外,其一赴池而决之河,其一纳之石槛,比竹为五管以出之,并河而东,绝三桥以入于石沟,注于南井。

明年春,六井毕修,而岁适大旱,自江淮至浙右井皆竭,民至以罂缶贮水相饷如酒醴。而钱塘之民肩足所任,舟楫所及,南出龙山,北至长河盐官海上,皆以饮牛马,给沐浴。

余以为水者,人之所甚急,而旱至于井竭,非岁之所常有也。以其不常有,而忽其所甚急,此天下之通患也,岂独水哉

(摘编自《苏轼全集》)

〔注〕 ①沮洳斥卤:低洼潮湿的盐碱地。②派别:江河的支流。③庳:低下。④甃:井壁。⑤罅漏:缝隙,漏洞。

2.对下列加点词的解释,不正确的一项是(3分)( )

A.惟负山凿井 负:靠近 B.率常不应 率:大抵

C.并河而东 东:东面 D.六井毕修 毕:全部

3.下列全都属于六井修治措施的一组是(3分)( )

①绝河而东至美俗坊为南井 ②于是发沟易甃,完缉罅漏 ③千艘更载,瞬息百斛 ④而列二闸于门外 ⑤比竹为五管以出之⑥皆以饮牛马,给沐浴

A.①②⑥ B.①③④ C.②④⑤ D.③⑤⑥

4.下列对文章有关内容的概括与分析,不正确的一项是(3分)( )

A.文章开篇介绍钱塘城的形成和地理特征,为下文写钱塘六井的开凿和修治作了必要的铺垫。

B.钱塘六井的开凿始于宰相李长源,终于刺史白乐天,为百姓引来了西湖之水,解决了饮水问题。

C.陈述古到任后,组织人员对钱塘六井进行了修治,使六井在第二年的大旱中发挥了重要作用。

D.本文以记钱塘六井的开凿和修治为主体,但不局限于“记”,而借“记”发“议”,意在“记”外。

5.把文中画横线的句子翻译成现代汉语。(6分)

(1)太守陈公述古始至,问民之所病。(3分)

译文:________________________________________________________________________

(2)此天下之通患也,岂独水哉? (3分)

译文:________________________________________________________________________

(三)古代诗歌阅读(6分)

6.阅读下面这首词,回答问题。

眼儿媚

[宋]朱淑真

迟迟春日弄轻柔,花径暗香流。清明过了,不堪回首,云锁朱楼。 午窗睡起莺声巧,何处唤春愁?绿杨影里,海棠亭畔,红杏梢头。

(1)请简要分析“云锁朱楼”中“锁”字的妙处。(3分)

答:________________________________________________________________________

(2)词的下片主要运用了何种表现手法?表达了词人怎样的思想感情?(3分)

答:________________________________________________________________________

二、文学名著、文化经典阅读(16分)

(一)文学名著阅读(10分)

7.根据原著内容填空。(5分)

(1)元春被册封为妃,皇帝恩准探家。荣国府为了迎接这一大典,修建了极尽奢华的__________。(《红楼梦》)

(3)觉新在花园里与__________再次相遇,觉新向她请求宽恕,并情不自禁地用手帕为她擦泪。(《家》)

(3)丝织厂闹起了工潮,吴荪甫起用年轻而头脑灵活的__________,平息了工潮。

(《子夜》)

(4)卡西莫多因为绑架爱斯梅拉达而被抓,当他在广场示众时,经过此地的指使者__________却头也不回走开了。(《巴黎圣母院》)

(5)涅赫柳多夫把从__________那里继承来的土地租给农民,并把农民交纳的资金当作公益金供给农民自己使用。(《复活》)

8.简答题。(选做一题)(5分)

(1)阅读下面的《三国演义》选段,根据原著故事情节,回答问题。

曹操急令去其伞盖,回顾左右曰:“我向曾闻云长言:翼德于百万军中,取上将之首,如探囊取物。今日相逢,不可轻敌。”

曹操以前是在什么情况下听到关云长这一评价的?张飞与曹军“今日相逢”的结果如何?请简述。

(2)阅读下面的《欧也妮·葛朗台》选段,根据原著故事情节,回答问题。

欧也妮进房把火放在桌上,声音发抖地说:“弟弟,我做了一桩非常对不起你的事;但要是你肯宽恕的话,上帝也会原谅我的罪过。”

“什么事呀?”夏尔擦着眼睛问。

“我把这两封信都念过了。”

夏尔的信是写给谁的?之后,欧也妮是如何对待夏尔的?请简述。

答第()题:________________________________________________________________________

(二)文化经典阅读(6分)

9.阅读下面的《论语》和《孟子》选段,回答问题。

①(子路、曾皙、冉有、公西华侍坐)子路率尔而对曰:“千乘之国,摄乎大国之间,加之以师旅,因之以饥馑;由也为之,比及三年,可使有勇,且知方也。”夫子哂之。(《论语·先进》)

②颜渊、季路侍。子曰:“盍各言尔志?”子路曰:“愿车马衣轻裘与朋友共,敝之而无憾。”颜渊曰:“愿无伐善,无施劳。”子路曰:“愿闻子之志。”子曰:“老者安之,朋友信之,少者怀之。”(《论语·公冶长》)

〔注〕 ①方:义。②伐:夸。③施:夸耀,表白。④怀:关爱。

(1)和弟子相比,孔子之志有什么特点?请简要概括。(2分)

答:________________________________________________________________________

(2)子路之志是什么?请概括并作简要评价。(4分)

答:________________________________________________________________________

三、现代文阅读(24分)

(一)论述类文本阅读(9分)

阅读下面的文字,完成10~12题。

纯文学将会变成一个伪问题

最近几年,“中国作家富豪榜”出炉之后,总会引出一些话题。今年的一个话题,是“纯文学大规模撤退”或“纯文学一年不如一年”。

按常理推断,纯文学作家越来越远离那张榜单,意味着他们的图书销量不高,而图书销量不高又意味着读者不多,于是也就有了媒体的惊呼。但问题是,我们现在还有所谓的纯文学吗?如果有,纯文学读者都到哪里去了?如果纯文学已经出现了问题,纯文学是不是已徒有虚名?

如果说80年代确实存在着一种纯文学观念,90年代以来,这种观念已逐渐被市场经济大潮冲击得七零八落。现如今,虽然还有纯文学这种说法,但或许早已名不符实。也就是说,今天所谓的纯文学早已不再是当年的纯文学了。

再说读者。如果有所谓的纯文学,自然也应该有纯文学读者。而纯文学读者的阅读趣味,往往是在青少年时代开始建立起来的。有人把上个世纪80年代概括为“精神阅读”时期,以此和90年代的“物质阅读”、新世纪以来的“功利阅读”相区别。从文学接受的角度看,这意味着80年代存在着数量不少的纯文学读者。当然,那个时代的儿童文学数量不多,青春文学还没面世,青少年读者自然就只能多去读一些名家名著。在文学名著的世界中待久了,便会形成一种纯正的阅读趣味。

然而,随着“物质阅读”时代的来临,读者的阅读趣味逐渐为另一种文学样式所建构。比如,以前是通俗文学,如今又有了青春文学。青春文学究竟能让青少年读者形成怎样的阅读趣味,如今还缺乏足够的研究,但无论怎样,恐怕都不大可能与纯文学有关。而一旦青春文学的阅读趣味形成之后,读者既读不懂纯文学,也很可能会排斥纯文学。于是,当今的纯文学,首先失去了大量的青少年读者。

青少年读者缺席,不是还有中老年读者吗?当然没错。但问题是时过境迁,这一类读者很可能已失去了当年的文学阅读热情,于是,崔健所唱的“看看电视听听广播念念报纸吧”,就成了他们的生活常态。

另一方面,真正的文学阅读还需要有点闲心,那样才能让阅读落到实处。作家马原有言:“看小说一定要沏一杯茶,安安静静的,没有人打扰,心里很闲。不会想:今天股票涨了还是跌了?”说的就是这种情况。但中年读者有闲吗?估计够呛。老年读者倒是闲下来了,但他们已人老眼花,精力不济。这就是所谓的贼心贼胆都有了,贼没了。

所以,今天纯文学读者很可能已经少之又少。而失去了大量读者的阅读与呵护,纯文学的生产可能就会出现问题。于是我们看到,与影视联姻的纯文学作家越来越多了,这种做法自然也能让一些观众再度变成读者,但问题是,一旦某部小说很适合影视改编,文学的纯粹性便打了折扣。结果,纯文学又开始变得不纯了。

小品中有纯爷们一说,实际生活中却是这种爷们越来越少,文学似乎也是如此。

(选自2010年12月13日《新京报》,作者赵勇,有删改)

10.下列有关“纯文学”的理解,不符合原文意思的一项是(3分)( )

A.最近几年,纯文学的发展一年不如一年,而证据之一便是纯文学作家越来越远离“中国作家富豪榜”。

B.如今,虽然还有纯文学这种说法,但随着市场经济大潮的冲击,纯文学早已不再是当年的纯文学了。

C.纯文学在上世纪80年代有较广阔的存在空间,这主要是因为当时经济不发达,人们可读的书不多,没有选择的余地。

D.纯文学的发展需要一定数量的读者,而如今,纯文学正面临着失去大量青少年读者的危机。

【一轮精品】2014高考语文一轮测评手册(福建专用):阶段评估检

11.下列的理解和分析,不符合原文意思的一项是(3分)( )

A.媒体的惊呼“纯文学一年不如一年”,主要的依据是“中国作家富豪榜”中纯文学作家的位次。

B.通俗文学、青春文学等流行文学使青少年读者形成了低俗的阅读趣味,从而影响了纯正阅读趣味的培养。

C.真正的文学阅读还需要有点闲心,那样才能让阅读落到实处,而中年读者缺少的恰恰是这样的“闲心”。

D.纯文学作家与影视联姻的做法虽然能让一些观众再度变成纯文学的读者,但也会使纯文学的纯粹性打了折扣。

12.请结合文本简要概括如今“纯文学读者”“少之又少”的主要原因。(3分)

答:________________________________________________________________________

(二)文学类、实用类文本阅读(选考)(15分)

本题为选做题,请从甲乙两类文本中选定一类作答,不得再做另一类文本的题目。

(甲) 文学类文本阅读(15分)

阅读下面的文字,完成13~15题。

补鞋匠苏小青

高 军

苏小青的补鞋摊上有一个微型录音机,她放的音量不大不小,来找她补鞋的人坐在摊子前的小马扎上正好能听得清楚:“日落黄昏风凄凄,我肚内无食身上少衣……”

听着王汉喜这凄楚的唱腔,来补鞋的人会疑惑地抬起头来看看她,只见苏小青脸色平静,一副心满意足的神态,并没有丝毫潦倒相。她头上黑色中掺杂着少许变白的发丝,脸上干干净净,穿着也干干净净,绝对没有常见的补鞋摊主那副尘灰傍土样。只有垫在腿上放鞋的那块帆布上,时常落上粘在鞋底的土屑,但每修好一只鞋,她会及时地轻轻抖落掉,然后才拿起另一只鞋子来补。

她以前是县吕剧团的台柱子,后来因不景气剧团解散了,经过一段时间的沉默后,她就购置了一些简易设备,到街头补鞋了。二十年过去,她成了县城里手艺过硬的补鞋人。现在已很少有人知道她的身世了,只是时常响起的吕剧声透出些微信息。

“……到后来才知你挪到外村住,赌志气搬得远远的。”张爱姐唱到这里,苏小青一双鞋子也修好了,苏小青拿起一块抹布仔细地擦拭,鞋面上一点尘土也没有时,才用手捏着递过来:“穿穿试试。”

鞋主赶紧接过来,程式化地一看,就蹬到脚上去了,然后从兜里掏出钱来递过去。

这时,苏小青并不抬手去接,而是从摊子上的一个提包侧面的兜里拿出一把银亮的镊子来,向前伸过来,用镊子口夹住钱币,放入盛钱的提包口里,再用镊子拨拉一下,把应回找的钱从包里用镊子取出来,夹到客人的手中。

有的顾客觉得她不必多此一举,就劝她,“俺的手不是也拿鞋了嘛。”

苏小青浅浅一笑,并不接这个话茬,但对下一位来补鞋的人她照样如此重复收钱找钱程序。

《王汉喜借年》放完,苏小青起身,“咔吧”一声又换上另一盒磁带,录音机“哧哧啦啦”一阵响。

“马大宝喝醉了酒忙把家还……”《借亲》正唱着,果真来了醉酒者。

“补——鞋。”一个骑摩托车的小伙子歪歪斜斜地走过来,在摊前的小马扎上一坐,满嘴酒气喷过来。苏小青一看,崭新的鞋子上裂开了一道大口子。小伙子坐在那里,把脚直直地伸着,并没有脱鞋。

苏小青拿起腿上铺的那块帆布轻轻抖动一下,仔细地在腿上铺展好,做好了为小伙子补鞋的准备。看小伙子的腿仍直直地伸在那里,苏小青的右手就随着录音机里的唱腔在膝盖上一下一下拍起来。

“脱鞋。”小伙子又往前伸伸脚,见没动静,慢慢抬起头来,看苏小青。苏小青神情淡定,笑中带威,威中含笑,慢慢伸出手来,等着小伙子把鞋子递过来。小伙子翻翻眼皮,瞪了一阵子,眼光暗淡下去,低下头,慢慢把自己的鞋子脱下来,递到苏小青的手上。

苏小青先把鞋子认真擦拭一遍,然后用双手将撕裂的茬口仔细地对接着,看看对准了,拿出一盒胶水来,挤出一些,在茬口的两面抹均匀,放下晾着。然后把缝纫机整理一下,先顺溜一下上面,纫入缝纫机针头丝线,再看看底线,等到鞋子接口处的胶水已经干了,就赶紧对接起来,压紧,然后缝出一行细密匀称的针脚,等用剪子清理好多余的线头线尾后,苏小青露出了满意的神情,鞋面上除了多出一行针脚外,并没有丝毫接茬的痕迹。

“好了。”她又拿起抹布,把鞋子细心擦了一遍,给小伙子递过来。

“哪用费这么多的事儿啊,回去我就扔了。”小伙子看后说。

苏小青淡淡一笑:“补好每一双鞋是我的事儿,至于补后怎么处理,那是你的事儿,请便。”尽管表面上这么说,苏小青心里还是一颤,她知道小伙子说的不假,回到家,小伙子就不会再穿这双鞋了。

小伙子可能还为刚开始的不礼貌有些后悔,在苏小青用镊子找他钱的时候,连连摆手,真诚地说:“不用找了,不用找了。”

苏小青好似没听到一样,镊子直直地举在他的眼前,小伙子抬起眼皮与苏小青对视了半天,眼光慢慢弯下去,伸出手接了过去。

苏小青笑笑:“骑车小心点,注意别磨了鞋子。”

小伙子郑重地点点头,起身离去了。

极富苍凉之感的《小姑贤》又在鞋摊上响起来:“千年的大道走成河……”

(选自《小小说选刊》,有删改)

13.下列对作品的概括和分析,不正确的两项是(5分)( )

A.文章一开头描写苏小青作为一个补鞋匠,却没有常见的补鞋摊主那种潦倒样,不仅脸上干干净净,穿着也干干净净,给读者留下了悬念。

B.苏小青收钱和找钱时都是用镊子夹住钱币,这个细节表明,在内心深处苏小青仍旧把自己看作一名吕剧演员,她依然保留着作为演员的清高。

C.听到小伙子的话后,“苏小青心里还是一颤”,这个细节说明她也意识到自己的补鞋方式有时并不值得,对不尊重自己劳动成果的人表示了反感。

D.醉酒青年从态度傲慢到自己脱鞋递给苏小青,再到对方找钱时因对刚才行为后悔而表露出真诚,这些态度的变化,反映了苏小青人格力量对他的感染。

E.本文描写了一个生活在底层的补鞋匠的故事,主人公在平凡的工作中努力地寻求自身生存的价值,表现了一个人的尊严和气节。

14.在小说中,录音机里不断放出的唱词起到了什么作用? (4分)

答:________________________________________________________________________

15.有人认为苏小青一生都沉迷在演员的角色中,还有人认为苏小青能心安理得地做一个补鞋匠,是一个超脱的人。你是如何看待这个人物的?请结合全文,谈谈你的观点和理由。(6分)

答:________________________________________________________________________

(乙)实用类文本阅读(15分)

阅读下面的文字,完成13~15题。

一个人的突围

张謇是中国传统文人中的异类,是思想和行动的“先知”。“先知”往往意味着孤独,意味着不被理解,意味着遭遇世俗的非难和抵制。无论是形而上的思想,还是形而下的实践,张謇一生都在进行一场艰苦卓绝、惨烈无比的突围。张謇,一介书生,常常独自以横刀立马的姿态出现在人们的视野中。

科举考场是他人生突围的第一个战场。初次参加州试,张謇名列百名之外。先生宋蓬山训斥道:“就算一千个人去考,如果录取九百九十九人,只有一个人不取,那就是你!”张謇羞愤交加,回头就在屋子里都贴上写有“九百九十九”的纸条。睡觉时,他用青竹板将辫子夹住,身体一翻转,辫子牵动头皮就疼醒了,看到满眼都是“九百九十九”,立刻爬起来点起油灯读书,每夜必“尽油二盏”。一次次“卧薪尝胆”,张謇不仅中了举人,而且获得了“江南才子”的美誉。但后面的科举之路却走得十分艰难。从16岁到42岁,张謇的科举苦旅历经26年,直至42岁第五次进京应试,才大魁于天下。

如果张謇止步于科举上的成功,满足于高官厚禄的前程,中国不过多了一个幸运的读书人,多了一个光宗耀祖的官员;但他没有,他在酝酿着一场更为精彩、更加宏伟的突围。

张謇高中状元之时,正值列强环伺、民族危亡的时代,他审时度势,毅然辞官回乡,“遁居江海,自营其事”。张謇远离官场并非出于文人的清高或英雄迟暮的消极,而是以强国拯民为己任,将一腔“救亡图存、振兴民族”的爱国情怀,书写在江海大地。

清末是中西方文化冲突最为激烈的时期,很多醉心于传统文化的人抱有历史偏见和民族情绪,抱残守缺,张謇却表现出难得的气度和远见,将“国学”和“西学”融会贯通。为探究“明治维新”后日本一跃而成东亚强国的原因,1903年4月,50岁的张謇第一次踏上了日本的土地。大阪博览会展示了日本工业革命取得的成果,当时电灯等电器在大清国尚属罕见,而日本已经极其普遍了。这令张謇大开眼界。在日本考察的两个多月中,张謇先后8次前往博览会参观。他认为,中国要在世界贸易中占有一席之地,必须大力发展工商业。回国以后,一场轰轰烈烈的经济突围开始了。

在张謇的倡议和推动下,我国第一个全国博览会——南洋劝业会成功举办,吸引了30多万人的目光,展览会起到了“开一时之风气,策异日之富强”的作用。与此同时,张謇“实业救国”的宏大计划也以人们难以想象的速度有序推进。

张謇说:“一个人办一县事,要有一省之眼光,办一省事,要有一国之眼光,办一国事,要有世界之眼光。”到20世纪20年代初,大生集团旗下超过70家企业,成为当时中国最大的民营企业集团。难能可贵甚至令人叹为观止、不可思议的还在于,这些企业组成了产业链上下游纵向联系和相关产业横向联系的经济生态网络。1920年张謇筹建了南通绣织局,并在美国、法国、瑞士、意大利等国设立分局和办事处,成为中国民族资本走向世界的一个重要里程碑。

在筹建张謇纪念馆时,人们赫然发现,因张謇创办实业的门类太多、成果太丰,竟无法找到一个合适的体系加以概括和划分。一个人的业绩让后人连概括、分类都颇有难度,当年创办之时的艰辛可想而知。

张謇依然抱持着“兼济天下”的士人梦想。他没有将目光局限于大生集团,他追求的是整个南通地区的均衡发展。在张謇的努力下,南通成为万人瞩目的“全国模范县”,给了很多人一个梦想的现实参照和慰藉。有人看到过当年国外发行的世界地图,中国许多大城市都没有标注,却标有“南通”这个小城。

1926年8月1日中午,张謇已感不适,但为了察看长江保坍工程进展情况,他还是冒着酷暑和工程师一起来到施工现场。察看工地后,张謇因病情加剧卧床不起,8月24日与世长辞,应验了他曾说过的“予为事业生,当为事业死”的豪言。

“文革”期间,造反派以“破四旧”为名掘开张謇墓穴,开棺后,发现内中仅有礼帽一顶,眼镜一副,折扇一柄。这一刻,张謇完成了他一生中最后一次人格突围,这次突围深深感动了后人。

张謇常说自己一生办事做人,只有“独来独往、直起直落”八个字。“我要去做东家,难有伙计;要做伙计,难有东家。”他一生孤独,精神支撑是内心崇高的社会理想,是念念不忘的兴国之梦。为了这个梦想,他奋斗一生,不断突围。

13.下列对文章的概括和分析,不正确的两项是(5分)( )

A.从张謇读书时“卧薪尝胆”的行为来看,他是一个不服输、肯吃苦的人。这也为他以后的成功奠定了基础。

B.科举考场是张謇人生突围的第一个战场,他从16岁到42岁参加了五次科举考试,最终才“大魁于天下”。

C.张謇屡经磨难高中状元,但他审时度势,认为应该独善其身,便毅然辞官回乡,“遁居江海,自营其事”。

D.张謇认为干事业应该放开眼界,正是在这种思想的指引之下,他筹建的南通绣织局成为中国民族资本走向世界的一个重要里程碑。

E.张謇说自己一生办事做人,只有“独来独往、直起直落”八个字,从中可知人无完人,张謇在性格上也是非常孤僻的。

14.请结合文章内容简要概括张謇是个什么样的人。(4分)

答:________________________________________________________________________

15.文章结尾说张謇“独来独往”“一生孤独”“我要去做东家,难有伙计;要做伙计,难有东家”,可见他的一生确实是孤独的。孤独与成功有没有关系?请对此进行探究。(6分)

答:________________________________________________________________________

四、语言文字运用(13分)

16.阅读下面一段文字,按要求回答问题。(3分)

①调研表明,职业状况是制约新生代农民工精神文化生活的首要因素。②微薄的工资收入、不明朗的发展预期、体力劳动的强度高,抑制了他们对精神文化生活的追求。③尽管有些企业拥有比如________、篮球场等文体设施,④但是由于农民工休息时间不足、精力不济等原因并不经常使用。

(1)第②句存在语病,请修改。(2分)

答:________________________________________________________________________

(2)为了不出现语病,第③句横线上应填写的恰当名词是:__________。(1分)

17.下面是根据王羲之的《兰亭集序》相关内容拟出的上联,请对出下联。(3分)

上联:少长咸集,观自然万物

下联:________________________________________________________________________

18.阅读下面两则材料,回答问题。(7分)

材料一:“我国有2亿多的中小学生,按照现行的评价标准,其中5000万属于差生,差生中80%是男生。而且男生越来越缺乏阳刚之气。”近日,全国人大代表、上海市教育发展基金会董事长王荣华在2012年两会上呼吁关注教育中的“男孩危机”。

材料二:著名青少年问题专家孙云晓曾出版了《拯救男孩》一书,他在书中列举了一系列触目惊心的数字,印证着男孩的“节节败退”:2006年~2007年、2007年~2008年连续两个年度全国5万名国家奖学金获得者中,大学女生人数均为男生的两倍左右;与此相对应,男性青少年网民上网成瘾比例为13.29%,女性为6.11%;男孩患多动症、自闭症、学习障碍、智力障碍等方面的比率也比女孩高。

对于上述现象,你有什么看法?请简要阐述。(要求:表达简明连贯,言之成理,150字左右)

答:________________________________________________________________________

________________________________________________________________________

________________________________________________________________________

五、写作(70分)

19.阅读下面的材料,根据要求写一篇不少于800字的议论文或记叙文。

周立波微博上一段话挺火:“电话,打一次没有接,就不要再打第二次;短信,发两次没有回,就不要再发第三次。没有这么卑微的等待,如果你重要,对方迟早会回过来的。没有必要为不懂得珍惜你的人犯贱,如果一个人开始怠慢你,请选择离开。保住一份尊严,宁可高傲到发霉,也不要死缠到发疯。”

这话有点道理,特别适用于年轻人之间的交往。不过,央视记者柴静却有不同的体验。柴静最近去英国布里斯托市,没想到在那儿碰上了《冰冻星球》摄制组的制片人凯瑟琳,柴静很喜欢她,想上去采访。但是,上头没给这个任务,拍了可能没地方播;贸然采访,也可能被拒绝,使自己没面子。但是柴静还是主动上去采访,结果相谈甚欢。柴静深有感触,事后在博客里写了一句话:“看见喜欢的,就得主动犯个贱。”

要求:(1)必须符合文体要求;(2)角度自选;(3)立意自定;(4)题目自拟;(5)不得抄袭,不得套作。 全品高考网:www.canpoint.cn阶段评估检测(三) ◄测评手册

阶段评估检测(三)

满分150分,考试时间150分钟。

一、古代诗文阅读(27分)

(一)默写常见的名句名篇(6分)

1.补写出下列名句名篇中的空缺部分。

(1)钉头磷磷,________________。(杜牧《阿房宫赋》)

(2)________________,抱明月而长终。(苏轼《赤壁赋》)

(3)________________,哀民生之多艰。(屈原《离骚》)

(4)西当太白有鸟道,________________。(李白《蜀道难》)

(5)________________;处江湖之远则忧其君。(范仲淹《岳阳楼记》)

(6)________________,此时无声胜有声。(白居易《琵琶行》)

(二)文言文阅读(15分)

阅读下面的文言文,完成2~5题。

方 干

[元]辛文房

干,字雄飞,桐庐人。幼有清才,散拙无营务。大中中,举进士不第,隐居镜湖中。湖北有茅斋,湖西有松岛,每风清月明,携稚子邻叟,轻棹往返,甚惬素心。所住水木幽闭,一草一花,俱能留客。家贫,蓄古琴,行吟醉卧以自娱。

徐凝初有诗名,一见干器之,遂相师友,因授格律。干有赠凝诗云:“把得新诗草里论。”时谓反语为村里老,疑干讥诮,非也。干貌陋兔缺,性喜凌侮。王大夫廉问浙东,礼邀干至,误三拜,人号为“方三拜”。王公嘉其操,将荐于朝,托吴融草表,行有日,王公以疾逝去,事不果成。干早岁偕计,往来两京,公卿好事者争延纳,名竟不入手,遂归,无复荣辱之念。浙中凡有园林名胜,辄造主人,留题几遍。

初李频学干为诗,频及第,诗僧清越贺云:“弟子已折桂,先生犹灌园。”咸通末卒。门人相与论德谋迹。谥曰玄英先生。乐安孙郃等缀其遗诗三百七十余篇,为十卷,王赞论之曰:“镘肌涤骨,冰莹霞绚。嘉肴自将,不吮余隽。丽不葩芬,苦不癯棘。当其得志,倏与神会。词若未至,意已独往。”郃亦论曰:“其秀也,仙蕊于常花;其鸣也,灵鼍于众响。”观其所述论,不过矣。

古黔娄先生死,曾参与门人来吊,问曰:“先生终,何以谥?”妻曰:“以‘康’。”参曰:“先生存时,食不充鬳,衣不盖形,死则手足不敛,傍无酒肉。生不美,死不荣,何乐而谥为康哉?”妻曰:“昔先生国君用为相,辞不受,是有余贵也。君馈粟三十钟,辞不纳,是有余富也。先生甘天下之淡味,安天下之卑位,不戚戚于贫贱,不遑遑于富贵,求仁得仁,求义得义,谥之以康,不亦宜乎?”方干,韦布之士,生称高尚,死谥玄英,其梗概大节,庶几乎黔娄者耶!

(选自《唐才子传》卷七)

〔注〕 ①兔缺:嘴唇豁缺。②廉问:察访查问。③偕计:又称“计偕”,指举人赴会试。④黔娄:战国时齐国贤士。⑤鬳:一种炊具。

2.对下列加点词的解释,不正确的一项是(3分)( )

A.举进士不第 第:及第 B.一见干器之 器:器重

C.名竟不入手 竟:竟然 D.咸通末卒 卒:去世

3.下列全都可以说明方干“庶几乎黔娄者”的一组是(3分)( )

①行吟醉卧以自娱 ②干早岁偕计,往来两京 ③遂归,无复荣辱之念 ④其秀也,仙蕊于常花 ⑤求仁得仁,求义得义⑥韦布之士,生称高尚,死谥玄英

A.①③④ B.①③⑥ C.②⑤⑥ D.②④⑤

4.下列对文章有关内容的概括与分析,不正确的一项是(3分)( )

A.方干从小才华出众,家境贫寒,但他却懒于谋划生计,常常吟诗饮酒,自娱自乐。

B.徐凝虽然比方干更早有名气,但并不以前辈自居,而是与方干相处甚欢,亦师亦友。

C.方干科举失意之后,醉心山水,浙中有些园林,他甚至几次前往,几次题字留念。

D.文章补叙古隐士故事,既说明方干当得起“玄英”之谥,也表明作者的敬仰之情。

5.把文中画横线的句子翻译成现代汉语。(6分)

(1)行有日,王公以疾逝去,事不果成。(3分)

译文:________________________________________________________________________

(2)先生甘天下之淡味,安天下之卑位。(3分)

译文:________________________________________________________________________

________________________________________________________________________

(三)古代诗歌阅读(6分)

6.阅读下面这首诗歌,回答问题。

平 凉

[明]李攀龙

春色萧条白日斜,平凉西北见天涯。唯余青草王孙路,不属朱门帝子家。

宛马如云开汉苑,秦兵二月走胡沙。欲投万里封侯笔,愧我谈经鬓有华。

〔注〕①平凉:明代边塞,在今甘肃省平凉县。

(1)请简要概括“白日斜”在诗中的作用。(2分)

答:________________________________________________________________________

________________________________________________________________________

(2)尾联抒发了诗人什么样的思想感情?请简要分析。(4分)

答:________________________________________________________________________

________________________________________________________________________

二、文学名著、文化经典阅读(16分)

(一)文学名著阅读(10分)

7.下列各项中对作品故事情节的表述,不正确的两项是(5分)( )

A.探春理家时,发现了贾府管理的弊端。于是,她和宝钗等商量决定先免掉头油脂粉钱,然后把园子中的花草、果木、稻田等交给专人管理。(《红楼梦》)

B.高老太爷要把鸣凤嫁给冯乐山。听到这个消息,鸣凤马上去找觉慧,可觉慧并不爱鸣凤,因此没听完她的哀诉就把她遣走了。鸣凤含泪离开投进了晶莹的湖水中。(《家》)

C.端午节过去了,上海的公债市场波动不定,谣言四起。吴荪甫和赵伯韬的纠纷依然没有解决。李玉亭打算做和事佬,他认为吴荪甫的刚愎自用是祸根。(《子夜》)

D.涅赫柳多夫要经过庄园附近的车站,姑妈派马斯洛娃去见他。她来到车站,由于时间紧急,只和涅赫柳多夫说了几句话,火车就开动了。(《复活》)

E.葛朗台临死前,神甫来作临终法事,把一个镀金的十字架送到他唇边亲吻。葛朗台见了金子就拼命想抓住。这一下的努力最终使他送了命。(《欧也妮·葛朗台》)

8.简答题。(选做一题)(5分)

(1)阅读下面的《三国演义》选段,根据原著故事情节,回答问题。

却说曹仁见关公落马,即引兵冲出城来;被关平一阵杀回,救关公归寨,拔出臂箭。原来箭头有药,毒已入骨,右臂青肿,不能运动。

关羽中了毒箭,谁主动上门医治?后来,此人提出用什么方法医治曹操的头痛病?曹操如何对待此人?

(2)《巴黎圣母院》一书写道:伽西莫多在乞丐进攻被镇压后发现爱斯梅拉达不见了。爱斯梅拉达究竟发生了什么事?最后,弗罗洛怎样处置她?请简述。

答第()题:________________________________________________________________________

(二)文化经典阅读(6分)

9.阅读下面的《孟子》选段,回答问题。

白圭曰:“丹之治水也愈于禹。”孟子曰:“子过矣。禹之治水,水之道也,是故禹以四海为壑。今吾子以邻国为壑。水逆行谓之洚水,洚水者,洪水也,仁人之所恶也。吾子过矣。”(《孟子·告子下》)

〔注〕 ①白圭:名丹,战国时期著名商人。②壑:指泄洪的大水坑。

(1)根据原文,请分别概括白圭和禹的治水方法。(4分)

答:________________________________________________________________________

________________________________________________________________________

(2)从道德层面上看,孟子认为白圭的治水方法错在哪里?(2分)

答:________________________________________________________________________

________________________________________________________________________

三、现代文阅读(24分)

(一)论述类文本阅读(9分)

阅读下面的文字,完成10~12题。

①斑驳的墙壁上,苔藓丛生,悠闲的常青藤爬满回忆,地上的方形石块已经磨平;街角的一家老店,让你不经意间与历史遭遇……

②冯骥才逛完巴黎发出赞叹:“巴黎的历史感,并不仅仅来自于埃菲尔铁塔、凯旋门、卢浮宫和巴黎圣母院。那是旅游者眼里的历史,或只是历史的几个耀眼的顶级的象征。巴黎真正的历史感是在城中随处可见的那一片片风光依旧的老街。”

③而此时,在以历史悠久著称的中国,正在进行大规模的城市改造,“旧的不去,新的不来”“破旧立新”“不破不立”得到广泛呼应与奉行。

④“城市记忆是在历史长河中一点一滴地积累起来的,从文化景观到历史街区,从文物古迹到地方民居,从传统技能到社会习俗等,众多物质的与非物质的文化遗产,都是形成一座城市记忆的有力物证,也是一座城市文化价值的重要体现。”单霁翔说。

⑤身为文物保护专家,单霁翔对一些城市在“旧城改造”“危旧房改造”中的作为痛心疾首。“由于急功近利作祟、经济利益驱使等人为因素,实施过度的商业化运作,采取大拆大建的开发方式,致使一片片积淀丰富人文信息的历史街区被夷为平地、一座座具有地域文化特色的传统民居被无情摧毁,一处处文物保护单位被拆迁和破坏的事件屡见不鲜。由于忽视对文化遗产的保护,造成这些历史性城市文化空间的破坏、历史文脉的割裂、社区邻里的解体,最终将导致城市记忆的消失。”

⑥一位德国历史学家针对北京的旧城改造曾说:“我们现在有的,你们将来都会有;而你们现在有的,我们永远不会有。”如今再回头看这句意味深长的话,实际上提醒了一个全新的角度,来审视对待旧城的观念。

⑦随着意识的增强和法律法规的健全,有些名气的遗址和遗迹已经没有人敢强拆,然而如果没有整体风格的保持,最多只是多了一些城市孤儿,高楼大厦包围下的古建筑,已经没有生命。

⑧单霁翔指出,对于历史文化城市的保护,现在国际上通行的做法是:对整个城市加以保护,像埃及的开罗、意大利的罗马、德国的吕贝克等。如果做不到这一点,也应成片成地段地保护,而不是只保护有文物价值的单个建筑。因为城市的一切,包括人们的生活方式,都是城市文化的一部分。

⑨我国城市化仍在高速运转,一座座新城拔地而起,大城市也普遍面临扩容压力。在这个过程中,如何正确对待历史,单霁翔的提醒显得尤为必要。

(节选自2010年5月19日《人民日报》)

10.下列关于“旧城改造”的理解,有误的一项是(3分)( )

A.“旧城改造”往往打着“旧的不去,新的不来”“破旧立新”等旗号,以此博得人们的广泛呼应。

B.“旧城改造”往往过多考虑经济收益,实施过度的商业化运作,采取了大拆大建的开发方式。

C.“旧城改造”往往忽视对文化遗产的保护,破坏历史性的城市文化,进而导致城市记忆的消失。

D.“旧城改造”没有强拆有名的遗址遗迹,但周边的高楼却阻塞了它们的交通,使它们失去了生命力。

11.下列说法不符合原文意思的一项是(3分)( )

A.冯骥才认为,巴黎的历史感不在于埃菲尔铁塔、凯旋门、卢浮宫和巴黎圣母院,而在于随处可见的老街。

B.城市记忆是一座城市文化价值的重要体现,包括文化景观、历史街区、文物古迹、地方民居、社会习俗等。

C.单霁翔认为,对于历史文化城市的保护,最好是对整个城市加以保护,而不是只保护有文物价值的单个建筑。

D.目前,我国正处于城市化高速运转时期,无论是新城建设,还是大城市扩容,都不能忽视对城市历史文化的保护。

12.第⑥段画横线的句子引用一位德国历史学家的话,表达了作者对“旧城改造”的什么观点?请联系全文简要概括。(3分)

答:________________________________________________________________________

________________________________________________________________________

(二)文学类、实用类文本阅读(选考)(15分)

本题为选做题,请从甲乙两类文本中选定一类作答,不得再做另一类文本的题目。

(甲)文学类文本阅读(15分)

阅读下面的文字,完成13~15题。

白玉盅

林清玄

①在所有的蔬菜里,苦瓜是最美的。

②苦瓜外表的美是难以形容的,它晶润透明,在阳光中,仿佛是白玉一般,连它长卵形的疣状突起部分也长得那么细致,触摸起来清凉滑润,也是玉的感觉。所以我觉得最能代表苦瓜之美的,是清朝的玉器“白玉苦瓜”。白玉苦瓜是清朝写实性玉雕的代表之作,历来只看到它的雕工之细、写实之美,我觉得最动人的是雕这件作品的无名艺匠,他把“白玉”和“苦瓜”做一结合,确实是一个惊人的灵感。比较起来,虽然“翠玉白菜”的声名远在“白玉苦瓜”之上,但是我认为苦瓜是比白菜更近于玉的质地。

③苦瓜俗称“锦荔枝”“癞葡萄”,白玉苦瓜表现了形象的美,但是我觉得它还不能完全表现苦瓜的内容,以及苦瓜的味觉。苦瓜切开也是美的,它的内部和种子是鲜红色,像是有生命流动的鲜血。有一次我把切开的苦瓜摆在白瓷的盘子里,红白相映,几乎是画笔所无法表达的。人站在苦瓜面前,尤其是夏天,心中就漫上一股凉意,那也只是一种感觉而已。

④不管苦瓜有多么美丽,它还是用来吃的。我年幼的时候最怕吃苦瓜,因为老使我想起在灶角熬着的中药,总觉得好好的鲜美蔬菜不吃,为何一定要吃那么苦的瓜。偏偏家里就种着几株苦瓜,有时抗议无效,常被妈妈通告苦着脸吃苦瓜,说是苦瓜可以退火,其实是因为家中的苦瓜生产过剩。

⑤嗜吃苦瓜还是这几年的事,也许是年纪大,经历的苦事一多,苦瓜也不以为苦了;也许是苦瓜的美,让我在吃的时候忘却了它的苦;我想最主要的原因,应该是我发现苦瓜的苦不是涩苦,不是俗苦,而是在苦中自有一种甘味,好像人到中年怀想起少年时代惆怅的往事,苦乐相杂,难以析辨。

⑥苦瓜有很多种吃法,我最喜欢的一种是江浙馆子里的“苦瓜生吃”,把苦瓜切成透明的薄片,蘸着酱油、醋和蒜末调成的酱,很奇怪,苦瓜生吃起来是不苦的,而是又香又脆,在满桌的油腻中,它独树一帜,没有一道菜比得上。有一回和画家王蓝一起进餐,他也最嗜苦瓜,一个人可以吃下一大盘,看他吃苦瓜,就像吃糖,一点也不苦。有一家江浙馆里别出心裁,把这道菜叫做“白玉生吃”,让人想起白玉含在口中的滋味,吃在口里自然想起故宫的白玉苦瓜,里面充满了美丽的联想。

⑦画家席德进生前也爱吃苦瓜,不但懂吃,自己还能下厨;他最拿手的一道菜是苦瓜灌肉,每次请客都亲自做这道菜,上市场挑选最好的苦瓜,还有上好的腱子肉,把肉细心地捣碎以后,塞在挖空的苦瓜里,要塞到饱满结实,或蒸或煮,别有风味。一次,画家请客,我看到他在厨房里剁肉,小心翼翼塞到苦瓜中去,到吃苦瓜灌肉时,真觉得人生的享受无过于此。我们开玩笑地把画家的拿手菜取名为“白玉盅”,如今画家去了,他拿手的白玉盅也随他去了,我好几次吃这道菜,总品不出过去的那种滋味。

⑧苦瓜真是一种奇异的蔬菜,它是最美的和最苦的结合,这种结合恐怕是造物者美丽的错误。以前有一种酸酸甜甜的饮料,广告词是“初恋的滋味”,我觉得苦瓜可以说是“失恋的滋味”,恋是美的,失是苦的,可是有恋就有失,有美就有苦,如果一个人不能尝苦,那么也就不能体会到那苦中的美。

⑨我们都是吃过苦瓜的,却少有人看过苦瓜树。去年我在南部,看到一大片苦瓜田里长出累累的苦瓜,农民正在收采,他们把包着苦瓜的纸解开,采摘下来,就像在树上取下一颗颗的白玉。我站在田边,看着篮中满满的苦瓜,心中突然感动不已,我想,真正苦瓜生命里的美,是远远比故宫橱窗里的苦瓜还令人感动的。

⑩我买了一个刚从田里采下的苦瓜,摆在家里,舍不得吃;放置几天以后,苦瓜枯萎了,失去了它白玉般的晶亮与透明,吃起来也丝毫不苦,风味尽失。这使我想起了人世间的许多事,美与苦是并生的,人不能只要美而不要苦,那么苦瓜的创作不能说是美丽的错误,它是人生真实的一个小影。

13.下列对作品的概括和分析,不正确的两项是(5分)( )

A.文章以“白玉盅”为题,不仅点出了写作的对象,而且突出了苦瓜的外表美,流露出作者对苦瓜深深的喜爱之情。

B.苦瓜不但外表美,而且切开也是美的,像是有生命流动的鲜血。人站在苦瓜面前,特别是在夏天,心中就会有凉意漫上。

C.第④段写作者年幼的时候最怕吃苦瓜,一方面写出了当年家庭的艰难,另一方面表达了由怕吃苦瓜到嗜吃苦瓜的情感变化。

D.作者在第⑧段说苦瓜是造物者“美丽的错误”,在第⑩段又说“苦瓜的创作不能说是美丽的错误”,这貌似矛盾,其实并不矛盾。

E.文章运用托物言志的手法,借苦瓜来寄托人生的感悟,告诫世人要像苦瓜一样,不但要有外表美,而且要拥有心灵美。

14.作者在第②段采用哪些手法来表现苦瓜之美?请结合文本简析。(4分)

答:________________________________________________________________________

15.作者为什么在文章的最后要写看苦瓜树和买苦瓜的经历?请结合全文简要分析。(6分)

答:________________________________________________________________________

________________________________________________________________________

(乙)实用类文本阅读(15分)

阅读下面的文字,完成13~15题。

新文化运动的先驱,语文改革的猛将

钱玄同(1887~1939),原名夏。语言文字学家、国学大师,“五四”新文化运动的先驱。1917年,他向陈独秀主办的《新青年》杂志投稿,倡导文学革命,成为“五四”新文化运动的揭幕人之一,成为鼓吹新文化,攻击封建主义,提倡民主、科学的勇士。他在《论应用之文亟宜改良》的《写作大纲》中提出改革大纲十三事,如用“国语”作文;规定语法之词序;小学课本、新闻纸旁注注音字母;文章加标点符号;用阿拉伯号码书写数目字;用公元纪年;书写方式改左行直下为右行横迤等,都是有关文化教育方面重大改革的首倡。

在任《新青年》杂志编辑期间,他曾动员鲁迅给《新青年》写文章。鲁迅的小说《狂人日记》就是钱玄同催促他写出的头一篇作品,并且头一次用鲁迅作笔名。众所周知,《狂人日记》不但是篇白话文,而且是攻击吃人的封建礼教的第一炮。

钱玄同在教学和学术研究方面的贡献也是很显著的。他所著的《文字学音篇》是我国高等学校最早的音韵学教科书,数十年来,影响颇大,迄今仍为音韵学家所称引。当代许多音韵学家如罗常培、魏建功、丁声树等或是他的学生,或受过他的教益。钱玄同对于“经学”创见甚多。他有两句名言:“考古务求其真,致用务求其适。”他发表在《古史辨》上讨论上古历史和儒家经书的文章,独见很多,影响很大。郭沫若对钱玄同在古史研究方面的一些观点非常赞赏,说:“这些见解与鄙见不期而同,但都是先我而发的。”

钱玄同在国语运动、文字改革方面有突出的贡献。他把文字音韵学精深的学术研究应用到国语统一、文字改革的实际工作中来。他既是国语运动的理论家,又是语言文字工作的实干者。钱玄同早年曾与鲁迅、黄侃等人师从章太炎学国学,研究音韵、训诂及《说文解字》,古文造诣很深,但是他却认为汉字难认、难记、难写,不利于普及教育、发展国语文学和传播科学技术知识,主张废除方块汉字,积极宣传汉语改用拼音文字,曾采用国际音标制定汉语拼音字母。后来他和赵元任、黎锦熙等数人共同制定“国语罗马字拼音法式”。1935年他抱病坚持起草了《第一批简字表》。可以说,建国以后文字改革的三大任务(简化汉字,推广普通话,制定和推行《汉语拼音方案》),钱玄同早在半个世纪以前就做过很多坚实的奠基工作,他称得起是文字改革工作的前驱。

钱玄同反对封建主义文化制度,也反对帝国主义的侵略。1925年“五卅惨案”发生后,他写过一篇《关于反抗帝国主义》的文章,他把反帝和反封建两项战斗任务结合起来考虑,主张一面积极反抗帝国主义的政治经济侵略,一面用民主、科学思想和现代的文化知识“唤醒国人”,使国人爱护自己的国家。他也说过“欧化”,说中国根本改革之路在“欧化”,他对“欧化”的解释是“全世界之现代文化,非欧洲所私有,不过欧洲人闻道较早,比我们先走了几步”。抗日战争开始,北平沦陷,北平师范大学西迁陕西。钱玄同因病未能随校赴陕,只得留在北平。1938年春,他恢复了旧名“钱夏”,表示“夏”而非“夷”,不做敌伪的顺民。他常间接寄语随北平师大迁至城固的好友黎锦熙等,说“玄同绝不污伪命”。1939年1月14日,钱玄同还到孔德学校处理李大钊的遗留图书《九通》,把它卖给当时的北京女师大,帮助解决李大钊的子女生活窘迫问题。1月17日,钱玄同忽然右脑部溢血,在德国医院逝世。

钱玄同逝世后,当时的进步刊物《文献》上发表署名乐颜的《悼钱玄同先生》中说:“平津沦陷以后,北方文化界处于暴日的铁蹄之下,居境非常悲惨;但钱先生保持着高洁的节操,不受包围,不被污辱,这种难能可贵的民族精神的表现,也是使得我们感动兴奋的。壮年以斗士领导青年,中年以学者努力学术,晚年以义士保持名节,钱先生总算是对得起自己、对得起国家民族的一位完人了。”

(摘编自《钱玄同:新文化运动的先驱,语文改革的猛将》)

13.下列对文章的概括和分析,最恰当的两项是(5分)( )

A.钱玄同在他的《写作大纲》中提出改革大纲十三事,都是有关文化教育方面重大改革的首倡,这使他成为“五四”新文化运动的揭幕人之一。

B.钱玄同动员鲁迅给《新青年》写文章,于是鲁迅写了《狂人日记》,向吃人的封建礼教开了第一炮,这是钱玄同作为“五四”文化先驱的贡献之一。

C.钱玄同倡导的新文化改革对后世影响很大,比如文章加标点符号、用公元纪年、使用阿拉伯数字、改为右行横迤书写等都成为今天通行的规则。

D.钱玄同到孔德学校处理李大钊的遗留图书,帮助解决李大钊的子女生活窘迫问题,几天之后就因右脑部溢血而逝世,体现出了他与李大钊的感情之深。

E.钱玄同的学术研究涉及诸多方面,这些研究往往彼此关联有时甚至是相互矛盾的,比如他古文造诣很深,但又认为汉字难认而主张废除方块汉字。

14.钱玄同在学术研究方面有哪些贡献?请简要概括。(4分)

答:________________________________________________________________________

15.乐颜在《悼钱玄同先生》一文中认为钱玄同是“对得起自己、对得起国家民族的一位完人”,你赞同这一评价吗?请结合文本谈谈你的理解。(6分)

答:________________________________________________________________________

四、语言文字运用(13分)

16.阅读下面材料,按要求回答问题。(3分)

长期以来我们在吃四大发明的老本,在一种虚______(wàng)的怀旧安慰中构筑自己的民族自尊心。这种自尊心是必要的,但这种思想方法却制造着一种后顾的文化模式。因为后顾,我们对世界的许多新变化有一种阿Q式的“我祖先比你阔多了”的情绪。这正造就了我们的长期封闭,亦造成了我们裹足不前。

(1)根据文中的汉语拼音写出汉字。(1分)

汉字:__________

(2)请写出一个与“裹足不前”词义相近的成语。(2分)

成语:__________

17.“神舟七号”飞船返回过程可分为四个阶段,请根据下面一段话,概括出其他三个阶段。(3分)

飞船调整飞行姿态,点燃发动机制动,完成离轨操作任务,进入返回轨道,保持无动力的飞行状态,返回舱和推进舱成功分离,第二次进入大气层,当返回舱降至离地面约10公里时,回收着陆系统开始工作,实现软着陆。

①制动飞行阶段;②________阶段;③________阶段;④________阶段。

18.阅读下面的材料,回答问题。(7分)

2011年7月中南大学大三学生刘路破解了“西塔潘猜想”,震惊国际数学逻辑界。随后,中南大学特批他硕博连读,并为其“量身打造”培养方案。同时,还将他作为青年教师后备人才,进入数学家侯振挺教授研究所从事研究工作。2012年3月20日,中南大学召开新闻发布会,破格聘请刘路为中南大学正教授级研究员(教授级研究员和教授在待遇上有区别,后者负责给学生讲课,前者主要做科学研究),并由学校推荐参加国家“青年千人计划”的评选。与此同时,刘路还获得100万元奖励,分别用于改善科研和生活条件。

对于上述事实,你有什么看法?请简要阐述。(要求:表达简明连贯,言之成理,150字左右)

答:________________________________________________________________________

________________________________________________________________________

________________________________________________________________________

五、写作(70分)

19.请以“墙”为题,自行立意,写一篇不少于800字的议论文或记叙文。 全品高考网:www.canpoint.cn阶段评估检测(四) ◄测评手册

阶段评估检测(四)

满分150分,考试时间150分钟。

一、古代诗文阅读(27分)

(一)默写常见的名句名篇(6分)

1.补写出下列名句名篇中的空缺部分。

(1)________________,以手抚膺坐长叹。(李白《蜀道难》)

(2)女之耽兮,____________!(《诗经·氓》)

(3)纵一苇之所如,______________。(苏轼《赤壁赋》)

(4)母、孙二人,____________,是以区区不能废远。(李密《陈情表》)

(5)山原旷其盈视,______________。(王勃《滕王阁序》)

(6)凭谁问:廉颇老矣,____________?(辛弃疾《永遇乐·京口北固亭怀古》)

(二)文言文阅读(15分)

阅读下面的文言文,完成2~5题。

原山狼

[清]赵执信

原山有穴焉,处非极颠,与村墟密迩。而经路险僻,旧盖人所居也。溪谷阴黝,草树蒙密。顷岁,一狼据之。狼状丑且秽,其声甚怪,出入百兽,且能为人。每日暮,则负穴而嗥,为儿啼,为嫠妇泣,为冤鬼哭,为市井喧哗,为军阵鼓吹,林壑响应,村人震悚,久而憎恶之,然莫敢犯。山中鸟兽闻之,以为雄杰也,群往附焉。有黠狐者,善媚,请奉为山之王,群小咸和。狼大喜,声益凶厉,间出行山下,窃鸡豚食之。与人值,亦未敢搏噬。黠狐曰:“此去城市才数里,盍往游乎?”狼意蹙缩,内自揣:吾以夜往,脱有不虞,返吾穴而据焉,若我何?至,则阑入吏舍。吏者海壖人也,所习惟鱼鳖,见狼则大惊,为具酒脯,致礼敬。狼意得甚,恣饮啖。由是数过吏,浸不避昼日。市人或遇之,骇而走,相恐以妖,竞闭其户。士人者从墙上视之曰:“狼也!”语吏不可近。吏既不能绝狼,因不听,狼益无所惮,泽其毛衣,侈然往来。自村及城中,皆相戒,谨避之。

有道士,抱奇术,隐深山。闻而造士人曰:“何子之柔也?是将食人。今不除,子与众且给其齿牙矣!”曰:“固知之,众不一,无有弓矢网罟,且吏昵之。”道士曰:“吏亦何利?语有之:‘野兽入室,主人将去。’既入其室矣,无已,吾助子。”乃为檄文,告诸神。其夕,大雷电夷其穴。狼适未归,归则无所据矣。狐兽尽散,狼窜于野,厌然不复能自奋。暮声凄断,但如鬼而已。村人相与谋,彼已无能为,操梃棓,敛瓦石,将毙之。城市知而争出,道士笑之曰:“属者神殛之也,幸其未食人,贷其命。汝曹顾欲死之,昔者奈何相戒而莫敢犯乎?汝曹之甚也。”众乃止,狼亦遁去。

评曰:狼无能为也,恐动群兽,弄吏以为威,由有所据以发其恶声也。丧其穴,败矣。地固不可以假狼哉!

(选自《饴山文集》卷十二)

〔注〕①值:相遇。②阑入:擅自进入。③海壖(ruán):海边。④浸:渐渐地。⑤侈然:放纵的样子。⑥厌然:畏缩的样子。⑦属者:近时。殛(jí):诛杀。

2.对下列加点词的解释,不正确的一项是(3分)( )

A. 则负穴而嗥 负:背靠 B. 间出行山下 间:有时

C. 盍往游乎 盍:通“何” D. 子与众且给其齿牙矣 且:将

3.下列直接表现狼“外强中干”的一组是(3分)( )

①狼状丑且秽,其声甚怪 ②狼意蹙缩,内自揣 ③间出行山下,窃鸡豚食之

④骇而走,相恐以妖 ⑤泽其毛衣,侈然往来 ⑥厌然不复能自奋

A.①②⑥ B.②③④ C.①④⑥ D.②③⑥

4.下列对文章有关内容的概括与分析,不正确的一项是(3分)( )

A. 原山狼所占据的洞穴,本是人所居住的地方,后来因为位置偏僻而放空;狼占据之后,洞穴成为狼逞凶的一大凭借。

B.狼一开始对人也心怀畏惧,只是偶尔偷些东西,至于入城,更是做好了随时逃跑的准备。后来,在人们的纵容之下,才变得肆无忌惮。

C.道士建议众人除狼,官吏因与狼关系亲昵而不同意,道士以“野兽入室,主人将去”之语劝导,官吏才同意。

D.作者认为,狼并不可怕,它之所以能作恶,与其有所凭借不无关系,因此,杜绝“狼”的出现,关键在于不给它生存的空间。

5.把文中画横线的句子翻译成现代汉语。(6分)

(1)吾以夜往,脱有不虞,返吾穴而据焉,若我何?(4分)

译文:________________________________________________________________________

(2)幸其未食人,贷其命。(2分)

译文:________________________________________________________________________

(三)古代诗歌阅读(6分)

6.阅读下面这首词,回答问题。

如梦令

[宋]苏轼

为向东坡传语, 人在玉堂深处。 别后有谁来?雪压小桥无路。 归去,归去, 江上一犁春雨。

〔注〕 ①诗人被贬黄州时曾开田数十亩,并命名为“东坡”。

(1)这首词可分为几个层次?它们分别写了什么内容 ?(3分)

答:________________________________________________________________________

(2)清人周济说:“人赏东坡粗豪,吾赏东坡韶秀。”你同意本词是苏轼的韶秀之作吗?请具体分析。(3分)

答:________________________________________________________________________

二、文学名著、文化经典阅读(16分)

(一)文学名著阅读(10分)

7.下列各项中对作品故事情节的表述,不正确的两项是(5分)( )

A.宝钗到潇湘馆找黛玉,恰逢宝玉来找黛玉,为避嫌疑,便抽身去找别的姊妹。宝钗见一双玉色蝴蝶,遂取出扇子扑蝶,来到滴翠亭上,却无意中听到小红和坠儿在谈闺房私话,为避免尴尬,宝钗便装出找黛玉的样子,大叫道:“颦儿,我看你往哪里藏!”(《红楼梦》)

B.吴老太爷死后,吴府大办丧事。各界客人纷至沓来,名为吊丧实则各怀鬼胎,或是打听战况,或是谈生意搞社交。这时,吴荪甫也与赵伯韬商谈筹办“益中信托公司”。他拿着“益中公司”的草案,踌躇满志,仿佛看到了美好的未来,不禁微笑了。(《子夜》)

C.觉民对琴低声说:“琴妹,我们学堂明年暑假要招收女生。”听到这消息,琴十分高兴,说到时候要第一个去报名。回家后,她找母亲商量,母亲的态度虽然让她很失望。但当她读了易卜生的《娜拉》后,又充满了希望,并写信给许倩如,表明自己的决心。(《家》)

D.一天,弗罗洛发现爱斯梅拉达身边有一个穿红黄两色外衣的男人抱着加里,十分紧张,立即从密室中赶往巴尔维广场,经过钟楼时,他发现伽西莫多专心致志地望着广场,他妒火中烧,逼问伽西莫多和爱斯梅拉达的关系。(《巴黎圣母院》)

E.葛朗台到了弥留的时候,看见来做临终法事的神甫把镀金的十字架送到他唇边,让他亲吻基督的圣像,他却作了一个骇人的姿势想把十字架抓在手里,这一最后的努力送了他的命。(《欧也妮·葛朗台》)

8.简答题。(选做一题)(5分)

(1)阅读下面的《三国演义》选段,根据原著故事情节,回答问题。

周瑜沉吟未决。正寻思没个机会,忽报蒋干又来。瑜大喜,一面分付庞统用计;一面坐于帐上,使人请干。

“蒋干又来”江东的目的是什么?“庞统用计”的“计”是什么?庞统是如何利用蒋干完成此计的?请简述。

(2)阅读下面的《复活》选段,根据原著故事情节,回答问题。

“我要生活,我要家庭和孩子,我想过人的生活。”就在他迈着很快的步子,连眼皮也不抬,走进办公室的时候,他脑海里掠过这样一些想法。

“他”是谁?“他”为什么会产生这些想法?请简述。

答第()题:________________________________________________________________________

(二)文化经典阅读(6分)

9.阅读下面的《论语》和《孟子》选段,回答问题。

①子曰:“君子哉蘧伯玉!邦有道,则仕;邦无道,则可卷而怀之。”(《论语·卫灵公》)

②孟子曰:“天下有道,以道殉身;天下无道,以身殉道。未闻以道殉乎人者也。”(《孟子·尽心上》)

〔注〕 ①殉:同“徇”,顺从。

(1)请简要概括孔子与孟子对于社会“无道”时的处世态度。(3分)

答:________________________________________________________________________

(2)“未闻以道殉乎人者也”表达了孟子什么样的观点?请结合文段②简要分析。(3分)

答:________________________________________________________________________

三、现代文阅读(24分)

(一)论述类文本阅读(9分)

阅读下面的文字,完成10~12题。

中国古典山水诗究竟源于何时?近人范文澜说:“写作山水诗起自东晋初庾阐诸人。”这话不无根据,写作山水诗形成一种潮流,一种时尚,的确是在东晋时期,《兰亭集》就是一个突出的例证。

东晋出现大量的山水诗,主要是纷乱的国情使然。东迁的文士几乎都有“风景不殊,正自有山河之异”的慨叹,加上受政治暴力和军事暴力的迫害,失落感愈来愈沉重。从清丽无比的江南山水风物中寻求抚慰和解脱,是行之有效的办法,于是流连山水,写作山水诗便相因成习,以致蔚然成风。另外,在新的哲学思潮如玄学的冲击下,汉朝以来“罢黜百家,独尊孔子”的思想控制日趋软弱松弛,于是出现“越名教而任自然”“法自然而为化”之类的主张。“自然”指宇宙自然规律,岿然不动的山和变动不居的水,则最充分、最完美地体现了这种规律,也就成了师法的对象,成了精神力量不竭的泉源。

孔子曰:“知者乐水,仁者乐山。”所谓“知者”,就是智慧之人;“仁者”则是仁义之人。为什么说“知者乐水,仁者乐山”?朱熹的解释是:“知者达于事理,而周流无滞,有似于水,故乐水。仁者安于义理,而厚重不迁,有似于山,故乐山。”原来,知者、仁者的品德情操与山水的自然特征和规律性具有某种类似性,因而产生乐水乐山之情。

另外,与山水亲近还可丰富知识,培养和提高审美情趣以及模山范水的能力。古今诗文大家、艺术巨匠大抵都有“读万卷书,行万里路”的经历。“读万卷书”正好可以弥补“行万里路”的不足。由于受种种条件的限制,人们无法遍览全国各地的山水胜迹,便可持山水诗集为“卧游”之具,作纸上的观瞻。这间接得到的知识和印象,与亲身所历、亲目所见自然隔了一层,但收获往往更快捷,也更精粹。

好的山水诗总是包含着作者深刻的人生体验,不单是模山范水而已。如“欲穷千里目,更上一层楼”以理势入诗,兼有教化和审美的双重功能,它表现出的求实态度和奋进精神,对读者无疑是有力的鞭策和激励。又如“蝉噪林逾静,鸟鸣山更幽”,除生动再现山林特有的幽静氛围,还揭示了矛盾的对立统一关系,显然有启迪智慧、拓展襟怀的作用。

优秀的山水诗大都具有“诗中有画,画中有诗”的特征。所谓“诗中有画”,就是用画笔把山水风物中精深微妙的蕴意点染出来,使读者获得直接的审美感受。如孟浩然的诗句:“相望始登高,心随雁飞灭。……天边树若荠,江畔洲如月。”用远景烘托远意。“远意”没有明说,只是影影绰绰地表现在“心随雁飞灭”的描摹上。勾画远景的笔墨也不多,却很有层次,显示出和谐的韵律与虚静阔远的美,像“天边树若荠,江畔洲如月”,清新淡远,与隐者(包括诗人自身)恬淡高远的情趣相表里,几乎达到天然淡泊的完美境界。

(选自袁行霈《盛唐的山水诗》,有删节)

10.下列对于山水诗形成原因的表述,不正确的一项是(3分)( )

A.“山河之异”的国情变化对文人的影响,受政治暴力和军事暴力的迫害使文人的失落感愈来愈沉重。

B.清丽无比的江南山水风物可以使文人们寻求抚慰和解脱,写作山水诗便相因成习,以致蔚然成风。

C.汉朝以来儒家思想控制日趋软弱松弛,“越名教而任自然”“法自然而为化”之类的主张出现。

D.人们可以从山水诗中看到自然风景,虽然与亲身所历、亲目所见自然隔了一层,但收获往往更快捷。

11.下列理解和分析,不符合原文意思的一项是(3分)( )

A.关于中国古典山水诗起源问题,作者认为范文澜“写作山水诗起自东晋初庾阐诸人”的说法是有一定依据的,《兰亭集》就是一个突出的例证。

B.孔子说的“知者乐水,仁者乐山”,可以从知者、仁者的品德情操与山水的自然特征和规律性具有某种类似性的角度来解释诗人喜爱山水的原因。

C.由于受种种条件的限制,人们无法遍览全国各地的山水胜迹,所以古今诗文大家、艺术巨匠大抵都用“读万卷书”来弥补“行万里路”的不足。

D.“欲穷千里目,更上一层楼”以理势入诗,兼有教化和审美的双重功能,它表现出的求实态度和奋进精神,是作者深刻的人生体验在诗句中的凝聚。

12.文章最后两段运用了许多古人诗句,有何作用,请简要概括。(3分)

答:________________________________________________________________________

(二)文学类、实用类文本阅读(选考)(15分)

本题为选做题,请从甲乙两类文本中选定一类作答,不得再做另一类文本的题目。

(甲)文学类文本阅读(15分)

阅读下面的文字,完成13~15题。

一只套鞋

[前苏联]左琴科

电车实在太拥挤了,而且你不能乱动,如果你不听劝告,非要在那狭小的空间里展示你的活泼,那你一定保不住你的套鞋。

当然,只是一只套鞋,很多人根本不会放在心上。

但如果你的套鞋在两分钟内就没了,你一定不会装作若无其事的。

我再清楚不过了,上电车的时候两只套鞋都在脚上,但等到下车的时候,结果却是:两只套鞋已经分居了。所有的衣物都老老实实地待在它应在的地方,唯有我右脚上的那只套鞋不见了。

车已经载着那只套鞋飞驰而去了……

我脱了剩下的那只套鞋,用报纸包上,就这么上班去了。等着吧:下班后我一定把它找回来。

下班了,这成了我的头号大事。我先找了一个认识的电车司机,希望从他那里得到些有用的信息。

他的话让我心里踏实多了。他说:

“嗯!是在电车上啊!应该没有什么问题。要是丢在别的公共场所,那就不保险啦。丢在电车上,找到的希望有百分之九十以上。我们局里有个失物招领处,到那儿就能领回失物,他们专负责这种事。”

“噢,谢天谢地,”我说,“现在我心就定啦。唉,我的套鞋是全新的。”

很快,我就找到了失物招领处。

“朋友,我的一只套鞋在电车上弄丢了,我希望能在这里找回来。”

“可以,”招领处的人回答说,“请描述一下您的套鞋吧。”

“套鞋嘛,好像没有什么特别之处,”我说,“鞋号是十二号。”

“十二号的鞋,我们这里可能有一万二千多只,你再细细地说一下吧。”

“特点嘛,也很普通,那是绿颜色的,鞋的两旁有白色条纹。”

“这样的鞋我们这儿也有上千只,说得再详细点好吗?”

“那是一只全新的套鞋,连鞋油都没来得及上。”

“请您稍等。”

瞧,她手里的确拿着我的套鞋。

我当时真是激动极了。我想,这里的工作真出色,工作人员竟在一只套鞋上花这么大的工夫。

“谢谢,”我说,“朋友,真不知如何感谢你的帮助,这对我来说太重要了。快给我吧,我好穿上。谢谢你啦!”

“不行,尊敬的同志,我仍不能确定这套鞋的真正主人。”

“我何必去骗一只套鞋呢?”

“我们丝毫不怀疑这一点。很可能这就是您丢的那只套鞋,但现在不能给您。请您开个证明来,证明您确实是丢了套鞋。让居委会再开个证明确保一下吧!这样才符合我们的工作程序。”

“朋友,”我说,“好同志,可是我的街坊并不知道我出了这档子事,他们可能不给开这样的证明。”

“他们一定会帮你的,而且……”

他坚持原则,我只好无奈地离开了。

第二天,我找到了居委会主任,对他说:

“请给我开个证明,我丢了一只套鞋。”

“这是事实吗?我可是上过不少次当了!是不是想捞个非分之财?”居委会主任说。

“真的,”我说,“我是丢了套鞋。”

他说:“那就拿一张电车公司的证明,单凭你一句话,我可不敢胡乱开证明,我必须为居委会的声誉负责。”

我说:“就是他们让我来这儿开证明的。”

他说:“那你打个报告吧。”

我说:“怎么写呢?”

他说:“你就写:某年某月某日丢失套鞋一只……,再加上点保证,就说你以什么样的名义起誓……”

我写了报告,随后便拿到了居委会的证明。

我拿着证明又到了失物招领处。好在一切都很顺利,套鞋被我拿了回来。

现在我终于拿回了我的那只套鞋,并把它重新穿到我的脚上。“瞧,他们的服务态度多好!要是别的单位,为一只套鞋肯定不会花那么多时间!从车上扔出去完事了。虽然花了一个星期的时间,但毕竟不是一无所获。”

但事情总不是那么尽如人意,又有一回,我丢了另一只套鞋——一星期以来,我把它包在报纸里一直随身夹带着。这次可记不得丢在哪里了。但我有一点可以确定,那就是一定不是在电车上。

虽然有所损失,但总算没全白忙活,现在我把它放在五斗柜上。每当心里烦闷时,只要朝这只套鞋看上一眼,我就心平气和了。那时我心里总会想:总会有像这样优秀的机构给我帮助的。

这只套鞋对我来说已远远地超出了它应有的价值,我一定会永远地保留着。

13.下列对作品的概括和分析,不正确的两项是(5分)( )

A.“我”在电车上丢失了一只套鞋,本不应该把这件事放在心上。但因为套鞋在很短的时间内丢失,才使得“我”心里很难受。

B.下班后,“我”在电车公司找了一个认识的人,希望通过他找到丢失的套鞋。事实上,我也由此顺利地找到了丢失的套鞋。

C.在失物招领处,工作人员一再要求“我”提供详细的信息。此时“我”也表现得很有耐心,对找回套鞋抱有希望。

D.“我”到了居委会开证明,结果受到了刁难。向电车公司打了报告后,才拿到居委会的证明,最终拿到了丢失的套鞋。

E.另一只套鞋丢失后,“我”没有上次那么幸运能找到套鞋。前后对比,更加突出了之前的工作人员认真负责的工作态度。

14.小说中的“我”为了找到丢失的一只套鞋,都做了哪些事?请简要概括。(4分)

答:________________________________________________________________________

15.小说结尾提到“我”对于丢失的另一只套鞋不抱找到的希望。这样写有什么用意?请简要分析。(6分)

答:________________________________________________________________________

(乙)实用类文本阅读(15分)

阅读下面的文字,完成13~15题。

不朽的贝多芬

[法]罗曼·罗兰

在大风雨中,大风雪中,一声响雷中,1827326日,贝多芬咽了最后一口气,一只陌生的手替他阖上了眼睛。

贝多芬!多少人颂赞他艺术上的伟大。但,贝多芬远不止是音乐家中的第一人。他是近代艺术的最英勇的力。对于一般受苦而奋斗的人,他是最大、最好的朋友。当我们对世界的劫难感到忧伤时,他会到我们身旁来,好似坐在一个穿着丧服的母亲旁边,一言不发,在琴上唱着他隐忍的悲歌,安慰那哭泣的人。当我们斗争到疲惫的辰光,到这意志与信仰的海洋中浸润一下,将获得无可言喻的裨益。

贝多芬分赠我们的是一股勇气,一种奋斗的欢乐。他在《致不朽的爱》一信中有言:“当我有所克服的时候,我总是快乐的。”《致韦格勒》一信中又说:“我愿把生命活上千百次……我非生来过恬静的日子的。”他还分赠我们一种感到与神同在的醉意。他仿佛在和大自然不息的沟通之下,竟感染了自然的深邃的力。申德勒说:“贝多芬教了我大自然的学问,在这方面的研究,他给我的指导和在音乐方面没有分别。使他陶醉的并非自然的律令,而是自然的基本威力。”格里尔巴策对贝多芬钦佩之中含有惧意,在提及他时说:“他所到达的那种境界,艺术竟和狂野与古怪的元素混合为一。”舒曼提到《第五交响曲》时也说:“尽管你时常听到它,它对你始终有一股不变的威力,有如自然界的现象,虽然时时发生,总教人充满着恐惧与惊异。”贝多芬的密友申德勒赞叹:“他抓住了大自然的精神。”——这是不错的:贝多芬是自然界的一股力;一种原始的力和大自然其余的部分接战之下,便产生了荷马史诗般的壮观。

贝多芬的一生宛如一天雷雨的日子。——先是一个明净如水的早晨,仅仅有几阵懒懒的微风,但在静止的空气中,已经有隐隐的威胁,沉重的预感。然后,突然之间巨大的阴影卷过,悲壮的雷吼,充满着声响的、可怖的静默,一阵复一阵的狂风,有如《英雄交响曲》与《第五交响曲》。然而白日的清纯之气尚未受到损害。欢乐依然是欢乐,悲哀永远保存着一缕希望。但自1810年后,心灵的均衡丧失了,目光变得异样。最清楚的思想,也看来似水汽一般在升华:忽而四散,忽而凝聚,它们的又凄凉又古怪的骚动,罩住了心;往往乐思在薄雾之中浮沉了一二次以后,完全消失了,淹没了,直到曲终才在一阵狂飙中重新出现,即是快乐本身也蒙上苦涩与狂野的性质。所有的情操里都混合着一种热病,一种毒素。贝多芬1810年5月2日致韦格勒书中有言:“噢,人生多美,但我是永远受着毒害……”黄昏来临,雷雨也随着酝酿。尔后是沉重的云,饱蓄着闪电,把黑夜染成乌黑,夹带着大风雨,那是《第九交响乐》的开始。——突然,当风狂雨骤之际,黑暗裂了缝,夜在天空被赶走,由于意志之力,白日的清明重又还给了我们。

什么胜利可和这场胜利相比?波拿巴的那一场战争?奥斯特利茨—拿破仑1805年12月大获胜利之地那一天的阳光?曾经达到这种超人的努力的光荣?曾经获得这种心灵从未获得的凯旋?一个不幸的人,贫穷,残废,孤独,由痛苦造成的人,世界不给他欢乐,他用他的苦难来铸成欢乐,好似他的那句豪语所说的——那是可以总结他一生,可以成为一切英勇心灵的箴言——“用痛苦换来欢乐。”

(选自《贝多芬传》)

13.下列对文章的概括和分析,不正确的两项是(5分)( )

A.文章第三段多次引用贝多芬的原话,直接展示了贝多芬的精神,增强了文章的感染力,也为作者的评述提供了依据。

B. 文章引用申德勒和格里尔巴策等人的话,使文章对人物的评述更加全面客观、真实可信,也能从侧面塑造贝多芬的性格。

C. 文章第二段画线的句子运用了比喻的手法,生动形象地写出当我们遭受失败和挫折时,可以从贝多芬的身上获得信心和勇气。

D.文章第四段写道,“贝多芬的一生宛如一天雷雨的日子”,这句话说明了贝多芬的一生就像雷雨一样令人可怖,永远充满着悲哀。

E.《第五交响曲》令人感到恐惧与惊异,因为在乐曲中表现了人与命运的悲壮搏斗,以及不屈的抗争精神和生命的欢乐。

14. 文章开头写了贝多芬去世时天气的恶劣和“陌生的手替他阖上了眼睛”这一细节,有何作用?请分别概括说明。(4分)

答:________________________________________________________________________

15.贝多芬的墓碑上铭刻着奥地利诗人格里尔巴策的题词:“当你站在他的灵柩跟前的时候,笼罩着你的并不是志颓气丧,而是一种崇高的感情;我们只有对他这样一个人才可以说:他完成了伟大的事业……”请结合文本,谈谈你对这句话的理解。(6分)

答:________________________________________________________________________

四、语言文字运用(13分)

16.依次填入下面一段文字横线处的语句,衔接最恰当的一组是(3分)( )

一转眼,不知怎样一来,整个燕园竟成了二月兰的天下。_______。_______。_______;_______。_______。_______。大有凌驾百花之上的势头了。

①最初只有一朵,两朵,几朵

②但是它却以多取胜,每到春天,和风一吹拂,便绽开了小花

③二月兰是一种常见的野花,花朵不大,紫白相间

④如果只有一两棵,在百花丛中,决不会引起任何人的注意

⑤花形和颜色都没有什么特异之处

⑥但是一转眼,在一夜间,就能变成百朵,千朵,万朵

A.①③②⑥⑤④ B.②④③⑥①⑤ C.③⑤④②①⑥ D.④①⑥②⑤③

17.阅读下面的材料,根据语境在横线上补写恰当的语句。要求:语意连贯,表达得体,不超过30字。(3分)

开 始

狂风过后,一株老树倒下了。我叹息道:“老树的生命结束了。”但我听见造物者说:“看看它身边的幼苗吧,一切才刚开始!”冬天近了,寒风中,我沉吟道:“一年了,一年又快结束了。”但我听见造物者说:“________________________________________________________________________。”

18.阅读下面的材料,回答问题。(7 分)

一本名为《浅薄》的书认为:“网络是一个叫人草草阅读、浅浅思量、快快走神、泛泛学习的环境,即使因特网能给我们提供便捷通道,使我们轻而易举地得到大量信息,但它毕竟在改变我们大脑的结构,分散我们的注意力,降低我们的理解力,从而把我们变成肤浅的思想者。”

《布鲁姆伯格商业周刊》的书评表示不同意《浅薄》关于使用因特网的人不能深思的说法,文中写道:“感谢因特网,现在许多人能对更多的事情进行更深入的思考”“我们现在有比过去多得多的人在想问题、写东西,仅在中国就有两亿博客。”

对于上述材料,你有什么看法?请简要阐述。(要求:表达简明连贯,言之成理,150 字左右)

答:________________________________________________________________________

________________________________________________________________________

________________________________________________________________________

五、写作(70分)

19.阅读下面的材料,根据要求写一篇不少于800字的议论文或记叙文。

明朝都察院长王廷相对新御史们讲了这样一件事:昨天雨后乘轿进城,轿夫穿的是双新鞋。开始,他小心地挑着干净地面落脚,后来一不小心踩进泥水里,于是便不再顾及新鞋。一御史听后说道:“终身不敢忘。”

要求:(1)必须符合文体要求;(2)角度自选;(3)立意自定;(4)题目自拟;(5)不得抄袭,不得套作。 全品高考网:www.canpoint.cn阶段评估检测(五) ◄测评手册

阶段评估检测(五)

满分150分,考试时间150分钟。

一、古代诗文阅读(27分)

(一)默写常见的名句名篇(6分)

1.补写出下列名句名篇中的空缺部分。

(1)故木受绳则直,______________。(《荀子·劝学》)

(2)月出于东山之上,________________。(苏轼《赤壁赋》)

(3)雕栏玉砌应犹在,_________________。(李煜《虞美人》)

(4)____________________,到黄昏、点点滴滴。(李清照《声声慢》)

(5)莫笑农家腊酒浑,__________________。(陆游《游山西村》)

(6)__________________,形影相吊。(李密《陈情表》)

(二)文言文阅读(15分)

阅读下面的文言文,完成2~5题。

周梅圃君家传

[清]姚鼐

梅圃君,长沙人,周氏,讳克开,字乾三,梅圃其自号也。以举人发甘肃,授陇西知县,调宁朔。其为人明晓事理,敢任烦剧,耐勤苦。宁朔属宁夏府,并河有三渠:曰汉来、唐延、大清,皆引河水入渠,以灌民田。唐延渠行地,多沙易漫,君治渠使狭而深,又颇改其水道,渠行得安,而渠有暗洞,以泄水于河,故旱涝皆赖焉。唐延渠暗洞坏,宁夏县吏欲填暗洞,而引唐渠水尽入汉渠,以利宁夏民,而宁朔病矣。君力督工修复旧制,两县皆利。大清渠者,康熙年始设,长三十余里,久而首尾石门皆坏,民失其利,君修复之,皆用日少而成功远。君在宁夏多善政,而治水绩最巨,民以所建曰周公闸、周公桥云。

寻擢贵州都匀知府。从总督吴达善、侍郎钱维城治贵州逆苗狱,用法有失当者,力争无少逊。调贵阳,亦以强直忤巡抚宫兆麟,因公累解职。引见,复授山西蒲州知府,调太原。清厘积狱,修复风峪山堤堰,障山潦,导之入汾,民德之。

累擢至江西吉南道,以过降官,复再擢为浙江粮储道。当是时,王亶望为浙江巡抚。吏以收粮毒民以媚上官者,习为恒矣,君素闻,疾之。至浙,身自誓不取纤毫润,请于巡抚,约与之同心。抚臣姑应曰善,而厌君甚,无术以去之也。反奏誉君才优,粮储常事易治,而其时海塘方急,请移使治海塘。于是调杭嘉湖海防道。君改建海岸石塘,塘大治,被劳疾,卒于任,而王亶望在官卒以贪败。世言苟受君言,岂徒国利,亦其家之安也。君卒后,家贫甚,天下称清吏者,曰周梅圃云。

姚鼐云:梅圃,乾隆间循吏也。夫为循吏传,史臣之职,其法甚严。不居史职,为相知之家作家传,容有泛滥辞焉。余嘉梅圃之治,为之传,取事简,以为后有良史,取吾文以登之列传,当无愧云。

(选自《惜抱轩全集》,有改动)

2.对下列加点词的解释,不正确的一项是(3分)( )

A.敢任烦剧 剧:繁重 B.君力督工修复旧制 制:式样

C.因公累解职 累:积累 D.为相知之家作家传 知:交好

3.下列全都表现周梅圃悉心为民的一组是(3分)( )

①民失其利,君修复之 ②以强直忤巡抚宫兆麟 ③君素闻,疾之 ④而其时海塘方急,请移使治海塘 ⑤君改建海岸石塘,塘大治⑥君卒后,家贫甚

A.①④⑥ B.②③⑥ C.①③⑤ D.②④⑤

4.下列对文章有关内容的概括与分析,不正确的一项是(3分)( )

A.针对唐延渠“多沙易漫”的特点,深谙治水之道的周梅圃采用深挖水道、改变水道走向、修复暗洞泄洪等方法,有效整治了唐延渠。

B.周梅圃廉洁正直,作者虽未明说百姓因此如何感激、爱戴他,但从文中百姓把他主持的治水工程称为周公闸、周公桥就可见一斑。

C.周梅圃就任浙江粮储道后,因为极力反对“毒民”恶政,成为贪官王亶望的眼中钉;调任杭嘉湖海防道后,终因积劳成疾死在任上。

D.作者写作这篇文章,意在以家传的形式彰显传主周梅圃的政绩、人品,同时,期待以后有优秀的史官能将本文收录到正史列传中。

5.把文中画横线的句子翻译成现代汉语。(6分)

(1)修复风峪山堤堰,障山潦,导之入汾,民德之。(3分)

译文:________________________________________________________________________

(2)吏以收粮毒民以媚上官者,习为恒矣。(3分)

译文:________________________________________________________________________

(二)古代诗歌阅读(6分)

6.阅读下面这首元曲,回答问题。

〔中吕〕 朝天曲

张养浩

柳堤,竹溪,日影筛金翠。杖藜徐步近钓矶。看鸥鹭闲游戏。农父渔翁,贪营活计,不知他在图画里。对这般景致,坐的,便无酒也令人醉。

(1)简析“日影筛金翠”一句中“筛”字的妙处。(3分)

答:________________________________________________________________________

(2)结合全曲,简要分析曲中表达的情感。(3分)

答:________________________________________________________________________

二、文学名著、文化经典阅读(16分)

(一)文学名著阅读(10分)

7.下列各项中对作品故事情节的表述,不正确的两项是(5 分)( )

A.觉民期待着朦胧的爱情,他父亲却说:“你爷爷希望有个重孙。”于是,为他定了一门亲事,觉民匆匆完婚。觉民也从此断绝了与心上人的爱情之路。(《家》)

B.少女时代的林佩瑶与雷鸣相恋,但雷鸣随即南下,开始军旅生涯。林佩瑶在苦等之下没有音讯,嫁给了自己并不喜欢的吴荪甫,婚后的她一直空虚寂寞。(《子夜》)

C.弗罗洛与甘果瓦商量怎样使爱斯梅拉达逃过被绞死的命运。甘果瓦主动要求跟爱斯梅拉达交换身份代替她绞死,是为了报答之前爱斯梅拉达的救命之恩。(《巴黎圣母院》)

D.葛朗台虽已积累了巨大的财富,但过日子却一直和庄稼人差不多,喝的老是坏酒,吃的老是烂果子,面包是自己家烤的,肉食蔬菜靠佃户供应。(《欧也妮·葛朗台》)

E.收到玛斯洛娃减刑的信,涅赫柳多夫高兴地告诉玛斯洛娃这个消息,虽然感激涅赫柳多夫,但是玛斯洛娃坚持要和西蒙松一起走。(《复活》)

8.简答题。(选做一题)(5 分)

(1)歇后语“周瑜打黄盖——一个愿打,一个愿挨”,说的是什么事?请简述这个故事。

(2)阅读下面的《红楼梦》选段,根据原著故事情节,回答问题。

“你是素日知道我的,从来不信什么是阴司地狱报应的。凭是什么事,我说要行就行。”“你瞧瞧我忙的,那一处少了我。既应了你,自然快快的了结。”

这是王熙凤答应净虚老尼的话,请简述她所答应之事的来龙去脉。

答第()题:________________________________________________________________________

(二)文化经典阅读(6分)

9.阅读下面的《论语》选段,回答问题。

①子曰:“回也,其心三月不违仁;其余,则日月至焉而已矣。”(《论语·雍也》)

②子曰:“仁远乎哉?我欲仁,斯仁至矣。”(《论语·述而》)

③子曰:“君子无终食之间违仁,造次必于是,颠沛必于是。”(《论语·里仁》)

〔注〕 ①三月:指较长时间。下文的“日月”指较短的时间。②造次:最紧迫的时刻。

(1)在行仁上,孔子对弟子有着什么样的评价?请简要概括。(2分)

答:________________________________________________________________________

________________________________________________________________________

(2)孔子为什么说培养仁德很容易又很困难?请结合上面选段简要分析。(4分)

答:________________________________________________________________________

________________________________________________________________________

三、现代文阅读(24分)

(一)论述类文本阅读(9分)

阅读下面的文字,完成10~12题。

扁平时代的写作

韩少功

美国学者托马斯·弗里德曼几年前推出《世界是平的》一书,认为因特网的廉价推广,促成了技术、资本、信息三个“民主化”同时到来,深刻改变着世界经济发展的方式和格局。他是一个敏锐的观察家,但谈得不够多和不够深的是文化,其中包括文学。“民主化”的文学是否可能?如果说“民主化”意味着一个有核心、有级差、有组织的塔状结构,让位于一个无核心、无级差、无组织的面状结构,那么这一前景是否值得万众欢呼?或者是否仅仅值得欢呼?

这当然是更为复杂的一个问题。

一个“扁平”的世界里众声喧沸。从原则上说,由编辑、审查、批准一类关卡所组成的文化权力体系几近瓦解,每一个IP地址自由发声,都可能成为强大的文化媒体。不过问题的另一面是,胡说比深思容易,粗品比精品多产,优秀者至少没有数量上的优势。文化产量中庸质与恶质的占比大大攀升,低端文化产业不仅无法淘汰,还可能日益滚大和做大。一些优秀作品即使生产出来,也可能在过量的文化淹没中,在受众们暴饮暴食式的阅读之后,在食欲不振的这些快餐

者们那里,出现影响力的严重折扣。一旦肠胃已经吃坏了,再多的良药也都无济于事

一个“扁平”的世界里多数为王。在一般的情况下,有些潮流可以修复民众良知,是真理的脱颖而出;有些潮流泯灭民众良知,是泡沫和垃圾的霸道横行。但不管是哪种情况,多数人的理解力构成潮流的边界,那么大众型和通俗化的真理尚有机会,而冷门的、偏僻的、艰险的、高难的——又常常是重要的文化探索,则可能缺氧。一种品质趋下的文化诱导机制,在这种情况下几乎难以避免。

一个“扁平”的世界还有易破难立的特点。特别是自18世纪启蒙运动以来,敬畏感随着上帝一同消失。叛逆比服从更流行,权利比责任更动心,无论左右翼都造反成癖,在获得解构主义一番学术装备后更是见立必破,打倒一切。越来越多的迹象表明,一旦失去文化的约束和引导机制,一个扁平的世界就是没有方向的世界,是无深度和无高度的世界。即使有成打的托翁和莎翁再世,他们通常也形同刺猬而不是狮子,是暗燃而不是火炬。

时值二十一世纪,人类有了前所未有的文化自由选择权,但为什么从这时起人类倒变得如此犹疑不定、六神无主、手足无措、茫然无计,竟找不到自己真正信赖和需要的东西?如果人类长期处于这样一种文化消费中的自我分裂和自我对抗,那么这种所好即所疑、所乐即所耻、所爱即所憎的左右两难,是不是一种文化狂欢之下的精神死机状态?也许需要重新启动,重新确定一个方向,一个重建精神价值的方向。

(摘自《新华文摘》2010年第6期,有删节)

10.下列对“扁平时代的写作”的理解,不符合文意的一项是(3分)( )

A.粗品比精品多产,优秀作品在数量上没有优势,影响力下降。

B.文学“民主化”可能意味着面状结构为塔状结构所取代。

C.容易失去文化的约束和引导机制,缺乏方向、深度和高度。

D.拥有文化自由选择权,但缺乏文明教化的正常体系。

11.下列说法中,不符合文意的一项是(3分)( )

A.弗里德曼认为,世界经济发展的方式和格局随着因特网的廉价推广以及技术、资本、信息三个“民主化”的同时到来而发生着深刻改变。

B.如果多数人的理解力构成潮流的边界,大众型和通俗化的真理可能被接受,冷门偏僻、艰险高难但又十分重要的文化探索则容易被冷落。

C.每一个IP地址自由发声,都可能成为强大的文化媒体,因此,由编辑、审查、批准所组成的文化权力体系几近瓦解,优秀作品被低端文化淹没。

D.自18世纪启蒙运动以来,人们少了几分敬畏,多了几分叛逆,无论左右翼都造反成癖,以至发展到见立必破,打倒一切的地步。

12.根据文意,理解“一旦肠胃已经吃坏了,再多的良药也都无济于事”的含意。(3分)

答:________________________________________________________________________

(二)文学类、实用类文本阅读(选考)(15分)

本题为选做题,请从甲乙两类文本中选定一类作答,不得再做另一类文本的题目。

(甲)文学类文本阅读(15分)

阅读下面的文字,完成13~15题。

武夷山:我的读后感

梁 衡

①名山也已登过不少,但当我有缘作武夷之游时,却惊奇地发现这次却不劳攀援之苦,只要躺在竹筏上默读两岸的群山就行。只这一点就足够迷人了。

②我半躺在筏上的竹椅里,微醉似的看两边的景色,听筏下汩汩的水声。耳边是船工喃喃的解说,这石、那峰、天王、玉女,还有河边的“神龟出水”,山坡上的“童子观音”。山水毕竟是无言之物,一般人耐不得这种寂寞,总要附会出一些故事来说。我却静静地读着这幅大水墨。

③这条曲曲弯弯的溪水美得纯真,是上游五十平方公里的群山中,滴滴雨露轻落在叶上草上,渗入根下土中,然后,沙滤石挤,再溢出涓涓细流,又由无数细流汇成这能漂筏行船的大河。所以这水就轻软得可爱。没有凶险的水涡,没有震山的吼声,只是悄悄地流,静静地淌,逢山转身回秋眸,遇滩蹑足曳翠裙。每当筏子转过一个急弯时,迎面就会扑来一股爽人的绿风,这时我就将身子压得更低些,顺着河谷看出去,追视这幅无尽的流锦,一时如离尘出世,不知何往。在这种人仙参半的境界中,我细品着溪水的清、凉、静、柔,几时享受过这样的温存与妩媚呢?回想与水的相交相识,那南海的狂涛,那天池的冰冷,黄河壶口的“虎啸”,长江三峡的“龙吟”,今天我才找到水之初的原质原貌,原来她“最是那一低头的温柔,不胜凉风的娇羞”。在世间一切自然美的形式中,怕只有山才这样的磅礴逶迤,怕只有水才这样的尽情尽性,怕也只有武夷山水才会这样的相间相错,相环相绕,相厮相守地美在一起,美得难解难分,教你难以名状,难以着墨。我才信山水也是如情人,如名曲,可以让人销魂铄骨的。一处美的山水就是一个暂栖身心的港湾,王维有他的辋川山庄,苏东坡有他的大江赤壁,朱自清有他的月下荷塘,夏丏尊有他的白马湖,今天我也找到了自己的武夷九溪。

④筏过五曲溪时,崖上有“五曲幼溪津”几个大字,那幼字的“力”故意写得不出头。原来这幼溪是一个明代人,名陈省,字幼溪,在朝里做官出不了头,便归隐此地来研究《易经》。石上还刻有他发牢骚的诗。细看两岸石壁,又有许许多多的古人题刻,我也渐渐在这幅山水画中读出了许多人物。那个曾带义兵归南宋,“而今识尽愁滋味,欲说还休”的词人辛弃疾,那个“但悲不见九州同”的诗人陆游,那个理学大师朱熹,都曾长期赋闲于此,并留下笔墨。还有那个一代名将戚继光,石壁上也留着他的铮铮诗句:“一剑横空星斗寒,甫随平北复征蛮。他年觅得封侯印,愿学幽人住此山。”这是些什么样的人啊,他们是从刀光剑影中杀出来的英雄,是从书山墨海中走过来的哲人,他们每个人的胸中都有一座起伏的山,都有一片激荡的海。可是当他们带着人世的激动,风尘仆仆地走来时,面对这高邈恬静的武夷,便立即神宁气平,束手恭立了。

⑤人在世上待久了,难免有这样那样的烦恼和这样那样的重负。为解脱这一切,历来的办法有二:一是皈依宗教,向内心去求平衡;二是到自然中去寻找回归。我登泰山时,曾感到山水对人的激励,登峨眉时,曾感到山水给人的欢娱,而今我在武夷的怀抱里,立即感到一种伟大的安详,朴素的平静,如桑拿浴后的轻松,如静坐功后的空灵。这种感觉怕只有印度教徒在恒河里洗澡,佛教徒在五台山朝拜时才会有的。我没有宗教的体验,却真正接受了一次自然对人的洗礼。武夷一小游,退却十年愁。对青山明镜,你会由衷地默念:什么都抛掉,重新生活一回吧。难怪这山上专有一处名“换骨岩”呢。

⑥我正庆幸自己在默读中悟出了一点道理,突然眼前一亮,竹筏已漂出九曲溪,水面顿宽,一汪碧绿。回头一望,亭亭玉女峰正在晚照中梳妆,船工还在继续着他那说不完的故事。

(选自梁衡散文集《觅渡》,有删节)

13.下列对作品的概括和分析,不正确的两项是(5分)( )

A.作者游览武夷山时,用的是乘坐竹筏观赏两岸风光的形式,这样既可省却攀援之苦,又另有一番情趣。

B. 游览时,船工用低柔的声调讲述了“神龟出水”“童子观音”等传说,并详细介绍了“五曲幼溪津”的来历。

C. 武夷山的溪水具有“清、凉、静、柔”的特点,对比南海、天池、黄河、长江的水,它更能体现水的原质原貌。

D.武夷山有许多古人题刻,这些古人都是些在人世中不得志的名人,他们后来都在高邈恬静的武夷山中隐居下来。

E. 本文虽是一篇游记,但作者并不仅限于记游,而是在风景的描写中穿插了许多联想和感悟,从而增加了文章的内涵。

14.文中两次提到“船工”的解说,有何作用?请简要概括。(4分)

答:________________________________________________________________________

15.本文是一篇游记,作者却以“读后感”为标题,这是为什么?请结合文章简要分析。(6分)

答:________________________________________________________________________

(乙)实用类文本阅读(15分)

阅读下面的文字,完成13~15题。

重返北京

陈孝全

白云悠悠,人世悠悠。

朱自清离开北京整整五年,想不到如今又回来了。举目无亲,只好先住在朝阳门边一位朋友的家里。他在北大读了四年书,虽也玩过几回西山,但多在城圈子里待着,始终没到过清华,对它很是陌生。

清华设在北京西北部的清华园,环境幽静,风景优美,原是端王载漪的王府。清华大学前身为“清华留美预备学校”,于1911年正式开办,它的任务就是培养留美学生。1925年清华进行改革,增设大学部,朱自清就是因此而被聘的。

那时清华大学的教务长是张仲述,朱自清不认识他,于是和那位朋友商量写一封信去,约定第三天上午前往拜访。那天几经周折,看表已经12点了,才到达清华大学。坐在客厅等一会儿,出来一个高个子长脸的,样子很能干的人,这就是他所要会见的教务长张仲述,谈到12点过,宾主才客气地分手了。

过了两天,朱自清带着简便的行李,从朝阳门朋友家搬出,住进了清华园古月堂。清华园很美,绵密的绿树丛中,蜿蜒着清清的溪流,郁葱的伞松,青青的草地,宽敞的教室,巍峨的礼堂,小小的荷池晃荡着岸边小树的倒影,池莲迎风起舞,散发出阵阵幽香。这样的风味和南方自不相同,别有一番气韵。但朱自清孤身一人,刚来乍到,没有什么朋友,心里十分寂寞。在江南时,他晚上睡眠极好,照例是一觉到天明,北来之后,却睡不安稳,夜夜有梦,而且从来没有一个是清清楚楚的,醒来不知所云,恍然若失。

最难堪的是每早将醒未醒之际,残梦依人,腻腻的不去;忽然双眼一睁,如坠深谷,万象寂然——只有一角日光在墙上痴痴地等着!此时决不起来,必凝神细想,欲追回梦中滋味之万一;但照例是想不出,只惘惘然茫茫然似乎怀念着些什么而已。纷乱的梦境反映的是不宁的心绪。其实,朱自清到北京之后,一直强烈地怀念着南方那段生活。

一天,他实在闷得慌,乃决意进城去,在海淀下了汽车,找了一个小饭馆,拣了临街的一张小桌子,坐在长凳上,要了一碟苜蓿肉,两张家常饼,二两白玫瑰,自斟自酌,不由又想起在江南的生活,情动于衷,从袋里摸出纸笔,在桌上写了一首《我的南方》:

我的南方,

那儿是山乡水乡!

那儿是醉乡梦乡!

五年来的彷徨,

羽毛般的飞扬!

呵!他怎能忘了南方的山山水水,乡土人情?那里有他的亲朋故友,有他年老的父母和弱妻稚子。在那里,他有过快乐,也有过痛苦,南方毕竟是他耕耘过的土地,汗水洒过的地方啊!

10月的一天,他接到南方来的一封信,是父亲寄的,其中写道:我身体平安,唯膀子疼痛利害,举箸提笔,诸多不便,大约大去之期不远矣。看到这里,朱自清不禁悲从中来,泪如泉涌,想到父亲待自己的种种好处,特别是八年前料理祖母丧事完毕,父子同车北上,在浦口车站分别的情景,犹如电影镜头一样历历在目。又想到,父亲少年出外谋生,独力支持,东奔西走,可家中光景竟一日不如一日,以致老境如此颓唐。又想到,他近来情郁于衷,常常动怒,但始终惦念着自己和自己的儿子。哀伤和想念之情如滔滔潮水,铺天盖地而来,在晶莹的泪光中,他仿佛又看见父亲肥胖的、穿着青布棉袍黑布马褂的背影!“我与父亲不相见已是二年余了,我最不能忘记的是他的背影。”他含着泪水,伏案疾书,以朴实的笔调细致地叙写那次和父亲别离的情景,透过父亲的一言一动,揭示了他对儿子的无限怜惜、体贴、依依难舍的深情。心灵在纸上疾走,他对父亲的刻骨思念之情,如涓涓流水,倾泻于字里行间,溶注于父亲的背影之中。写到最后,他深情地呼告道:“唉!我不知何时再能与他相见!”平淡一语,蕴蓄着他对年迈父亲的刻骨相思。

22年后,当《文艺知识》编者问他写作这篇《背影》的情况时,他答道:“我写这篇文章只写实,似乎说不到意境上去。”这种从表面上看起来简单朴素,而实际上却能发出极大的感动力的文章,最可以作为朱先生的代表作品,因为这样的作品,正好代表了作者之为人。由于这篇短文被选为中学国文教材,在中学生心中“朱自清”这三个字已经和《背影》成为不可分的一体。

这是由文品论及人品了。

13.下列对文章的概括和分析,不正确的两项是(5分)( )

A.清华大学增设大学部,聘请朱自清北上任教,在与教务长张仲述相见的两天后,朱自清即入住清华园。

B.朱自清重返北京后,孤独寂寞,工作艰辛,加上身体虚弱,常常夜不能寐,他强烈地怀念着南方的那段美好生活。

C.朱自清不能忘记南方的生活,南方的山山水水和乡土人情,故而情动于衷,写下了《我的南方》一诗。

D.朱自清父亲少年时出外谋生,东奔西走,老境辛苦颓唐,他深感身体有诸多不便,怕是要不久于人世。

E.本文叙写朱自清重返北京后的一段经历,同时也分析了他的作品,指出当时动荡时局对其作品的影响。

14.画线部分有何特点?请加以赏析。(4分)

答:________________________________________________________________________

15.李广田说:“《背影》一篇,寥寥数十行,不过千五百言,它之所以能历久传诵而有感人至深的力量,只是凭了其中所表达的真情。”请结合文本分析朱自清的“真情”所在。(6分)

答:________________________________________________________________________

四、语言文字运用(13分)

16.在横线处填入恰当的关联词,使语意连贯。(2分)

考试与教学内容涉及的范围必须一致,____,考试依据《教学大纲》无疑是正确的。但考试与教学又有所不同。教学是使学生从不知或知之较少到知或知之较多,从能力较低到能力较高的过程,而考试则是检验学生的学习结果。《教学大纲》规定如何教学,《考试说明》____规定考试的性质、内容、形式等。

答:①________________ ②________________

17.我国第十二届全运会将于2013年9月在辽宁省举行。下图是该运动会的志愿者标志,请从构形角度说明该标志的创意。要求语言鲜明生动,语意简明连贯,句子通顺完整。(100字左右)(4分)

答:________________________________________________________________________

________________________________________________________________________

________________________________________________________________________

________________________________________________________________________

________________________________________________________________________

________________________________________________________________________

18.阅读下面两则材料,回答问题。(7分)

材料一:近日从全国老龄工作委员会上传来消息,《老年人权益保障法》修订工作已基本完成,其中最让人关注的是,子女“常回家看看”将入法。

材料二:最新的统计数据显示,目前,我国1.67亿老人中,有一半过着“空巢”生活。由于子女工作、求学、婚姻等各种原因长期离家在外,独守“空巢”的老人在生活中容易发生困难。

对于上述现象,你有什么看法?请简要阐述。(要求:表达简明连贯,言之成理,150字左右)

答:________________________________________________________________________

________________________________________________________________________

________________________________________________________________________

五、写作(70分)

19.阅读下面的材料,根据要求写一篇不少于800字的议论文或记叙文。

著名导演张艺谋有一句名言:“人生在世,都有一份作业。”其实,人生的过程就是不断书写作业、创造精彩的过程……

请以“人生的作业”为题作文。

要求:(1)必须符合文体要求;(2)角度自选;(3)立意自定;(4)不得抄袭,不得套作。 全品高考网:www.canpoint.cn阶段评估检测(六) ◄测评手册

阶段评估检测(六)

满分150分,考试时间150分钟。

一、古代诗文阅读(27分)

(一)默写常见的名句名篇(6分)

1.补写出下列名句名篇中的空缺部分。

(1)吾师道也,_______________________?(韩愈《师说》)

(2)无丝竹之乱耳,___________________。(刘禹锡《陋室铭》)

(3)扪参历井仰胁息,_______________________。(李白《蜀道难》)

(4)山河破碎风飘絮,_______________________。(文天祥《过零丁洋》)

(5)___________________,一弦一柱思华年。(李商隐《锦瑟》)

(6)___________________,谁家新燕啄春泥。(白居易《钱塘湖春行》)

(二)文言文阅读(15分)

阅读下面的文言文,完成2~5题。

复吴南屏

[清]曾国藩

三月初旬,奉覆一函,想已达览。旋接上年腊月惠书,并大著诗文全集各五十部,就审履祺康胜,无任企仰

大集古文敬读一过,现昔年仅见零篇断幅者,尤为卓绝。大抵节节顿挫,不矜奇辞奥句,而字字若履危石而下,落纸乃迟重绝伦。其中闲适之文,清旷自怡,萧然物外,如《说钓》《杂说》《程日新传》《屠禹甸序》之类,若翱翔于云表,俯视而有至乐。国藩尝好读陶公及韦、白、苏、陆闲适之诗,观其博揽物态,逸趣横生,栩栩焉神愉而体轻,令人欲弃百事而从之游。而惜古文家少此恬适之一种,独柳子厚山水记,破空而游,并物我而纳诸大适之域,非他家所可及,今乃于尊集数数遘之。故编中虽兼众长,而仆视此等尤高也。

与欧阳筱岑书中,论及桐城文派,不右刘、姚,至比姚氏于吕居仁,讥评得无少过?刘氏诚非有过绝辈流之诣,姚氏则深造自得,词旨渊雅,其文为世所称诵者,如《庄子章文库》《礼笺序》《朱竹君传》《仪郑堂记》《〈南园诗存〉序》等篇,皆义精而词俊,夐绝尘表。其不厌人意者,惜少雄直之气、驱迈之势。姚氏固有偏于阴柔之说,又尝自谢为才弱矣。其论文亦多诣极之语,国史称其有古人所未尝言,鼐独抉其微发其蕴。亟顿称海峰,不免阿于私好。要之方氏以后,惜抱固当为百年正宗,未可与海峰同类而并薄之也。浅谬之见,惟希裁正。

国藩回任江表,眴逾半年,辖境敉平,雨泽沾足,岁事可望丰稔。惟是精力日衰,前发疝气,虽已痊愈,目光蒙雾,无术挽回。吏治兵事,均未能悉心料理,深为愧悚。吾乡会匪窃发,益阳、龙阳等城,相继被扰。此辈游荡无业,常见逐风尘而得逞。湘省年年发难,剿之而不畏,抚之而无术,纵使十次速灭,而设有一次迁延,则桑梓之患,不堪设想,殊以为虑。

(有删改)

〔注〕①吴南屏:清代散文家吴敏树,号南屏。②履祺康胜:书信敬辞,意即祝吉祥安好。③无任企仰:敬辞,“不胜仰慕”之意。④刘、姚:指桐城三祖中的刘大櫆(号海峰)、姚鼐(世称惜抱先生)。⑤夐(xiōng)绝:超绝。⑥敉(mǐ):平定,安定。

2.对下列加点词的解释,不正确的一项是(3分)( )

A.不矜奇辞奥句 矜:自夸

B.论及桐城文派,不右刘、姚 右:尊崇

C.亟顿称海峰,不免阿于私好 阿:偏袒

D.未可与海峰同类而并薄之也 薄:接近

3.下列全都属于作者推崇姚鼐的一组是(3分)( )

①清旷自怡,萧然物外 ②令人欲弃百事而从之游 ③并物我而纳诸大适之域 ④深造自得,词旨渊雅 ⑤义精而词俊,夐绝尘表⑥论文亦多诣极之语

A.①②④ B.④⑤⑥ C.①③⑤ D.②③⑥

4.下列对文章有关内容的概括与分析,不正确的一项是(3分)( )

A.曾国藩在给吴敏树的回信中论文讲学,对吴敏树闲适类散文高度称许。

B.曾国藩和吴敏树虽有论文讲道之谊,但两人的文学观点不尽一致。

C.曾国藩把姚鼐和吕居仁相提并论,推崇姚鼐文章为百年正宗。

D.吴敏树对刘大櫆和姚鼐有微词,曾国藩在信中坦言吴敏树“讥评少过”。

5.把文中画横线的句子翻译成现代汉语。(6分)

(1)其不厌人意者,惜少雄直之气、驱迈之势。(3分)

译文:________________________________________________________________________

(2)吏治兵事,均未能悉心料理,深为愧悚。(3分)

译文:________________________________________________________________________

(三)古代诗歌阅读(6分)

6.阅读下面这首词,问答问题。

浣溪沙

宋秦观

漠漠轻寒上小楼,晓阴无赖似穷秋。淡烟流水画屏幽。 自在飞花轻似梦,无边丝雨细如愁。宝帘闲挂小银钩。

(1)请从情与景关系的角度赏析这首词的情感。(3分)

答:________________________________________________________________________

(2)“自在飞花轻似梦,无边丝雨细如愁”中“轻”与“细”两字新奇、精妙,请简要分析。(3 分)

答:________________________________________________________________________

二、文学名著、文化经典阅读(16分)

(一)文学名著阅读(10分)

7.下列各项中对作品故事情节的表述,不正确的两项是(5分)( )

A.因为主政的张军长表示要施行新政,并支持学生们的进步活动,所以觉慧积极参加《黎明周报》的工作。后因周报言论过激而被查封,于是觉民和琴便决定筹办《利群周报》。(《家》)

B.吴老太爷到了上海,吴家儿女接他的汽车开动了。忽然吴老太爷锐声叫了起来:《道德经》!面对这一声叫喊,大家都怔住了,只有吴荪甫明白老太爷要的是什么。(《子夜》)

C.爱斯梅拉达和小山羊“佳丽”来到圣母院前的广场上表演,小山羊模仿了手铳队队长和宗教法庭检察官的可笑模样,观众对此给予了热烈的掌声。(《巴黎圣母院》)

D.法院开庭审判马斯洛娃的案件,涅赫柳多夫作为贵族代表参加陪审。他在法庭上认出了马斯洛娃,马斯洛娃的现状使他十分震惊,他的灵魂进行了强烈的斗争。(《复活》)

E.仆人拿侬将家里杂务全包办了,一年的工薪也只有60法郎,她在葛朗台家辛勤劳作的第20年上,葛朗台终于“大方”地赏了她一只旧表,那是她到手的唯一的礼物。(《欧也妮·葛朗台》)

8.简答题。(选做一题)(5分)

(1)阅读下面的《红楼梦》选段,根据原著故事情节,回答问题。

凤姐笑道:“这个孩子扮上,活像一个人,你们再看不出来。”宝钗心里也知道,便只一笑,不肯说。宝玉也猜着了,亦不敢说。史湘云接着笑道:“倒像林妹妹的模样儿。”

史湘云说出大家心中的想法时,宝玉有何举动?对于宝玉的这一举动,史湘云和黛玉各自有何态度?请简述。

(2)阅读下面的《三国演义》选段,根据原著故事情节,回答问题。

张飞曰:“哥哥差矣。量此村夫,何足为大贤!今番不须哥哥去;他如不来,我只用一条麻绳缚将来!”玄德叱曰:“汝岂不闻周文王谒姜子牙之事乎?文王且如此敬贤,汝何太无礼!今番汝休去,我自与云长去。”

张飞口中的“村夫”指谁?此次前往,“村夫”正“昼寝未醒”,对此,张飞和刘备各有什么样的举动?请简述。

答第()题:________________________________________________________________________

(二)文化经典阅读 (6分)

9.阅读下面的《论语》选段,回答问题。

①子路问政。子曰:“先之劳之。”请益。曰:“无倦。”(《论语·子路》)

②子曰:“其身正,不令而行;其身不正,虽令不从。”(《论语·子路》)

③子曰:“苟正其身矣,于从政乎何有?不能正其身,如正人何?”(《论语·子路》)

〔注〕①先之:先,引导,先导,即教化。之,指老百姓。②劳之:指为政者亲身去干,以自身的“先劳”,带动老百姓都勤劳地干。③益:增加。

(1)请简要概括孔子对为政者的要求。(2分)

答:________________________________________________________________________

(2)孔子把“正身”看作是从政为官的重要方面,这有什么道理?请简要分析。(4分)

答:________________________________________________________________________

三、现代文阅读(24分)

(一)论述类文本阅读(9分)

阅读下面的文字,完成10~12题。

美和实际人生的距离

朱光潜

①北方人初看到西湖,平原人初看到峨嵋,虽然审美力薄弱的村夫,也惊讶它们的奇景;但生长在西湖或峨嵋的人除了以居近名胜自豪以外,心里往往觉得西湖和峨嵋实在也不过如此。新奇的地方都比熟悉的地方美,东方人初到西方,或是西方人初到东方,都往往觉得面前景物件件值得玩味。本地人自以为不合时尚的服装和举动,在外方人看,却往往有一种美的意味。

②这些经验你一定也注意到过。它们是什么缘故呢!

③这全是观点和态度的差别。看倒影,看过去,看旁人的境遇,看稀奇的景物,都好比站在陆地上远看海雾,不受实际的切身的利害牵绊,能安闲自在地玩味目前美妙的景致。看正身,看现在,看自己的境遇,看习见的景物,都好比乘海船遇着海雾,只知它妨碍呼吸,只嫌它耽误程期,预兆危险,没有心思去玩味它的美妙。持实用的态度看事物,它们都只是实际生活的工具或障碍物,都只能引起欲念或嫌恶。要见出事物本身的美,我们一定要从实用世界跳开,以“无所为而为”的精神欣赏它们本身的形象。总而言之,美和实际人生有一个距离,要见出事物本身的美,须把它摆在适当的距离之外去看。

④艺术须与实际人生有距离,所以艺术与极端的写实主义不兼容。写实主义的理想在妙肖人生和自然,但是艺术如果真正做到妙肖人生和自然的境界,总不免把观者引回到实际人生,使他的注意力旁迁于种种无关美感的问题,不能专心致志地欣赏形象本身的美。

⑤艺术上有许多地方,乍看起来,似乎不近情理。古希腊和中国旧戏的角色往往戴面具,穿高底鞋,表演时用歌唱的声调,不像平常说话。埃及雕刻对于人体加以抽象化,往往千篇一律。波斯图案画把人物的肢体加以不自然地扭曲,中世纪“哥特式”诸大教寺的雕像把人物的肢体加以不自然的延长。中国和西方古代的昼都不用远近阴影。这种艺术上的形式化往往遭到人们唾骂,它固然时有流弊,其实也含有至理。这些风格的创始者都未尝不知道它不自然,但是他们的目的正在使艺术和自然之中有一种距离。说话不押韵,不论平仄,做诗却要押韵,要论平仄,道理也是如此。艺术本来是弥补人生和自然缺陷的。如果艺术的最高目的仅在妙肖人生和自然,我们既已有人生和自然了,又何取乎艺术呢?

⑥艺术都是主观的,都是作者情感的流露,但是它一定要经过几分客观化。艺术都要有情感,但是只有情感不一定就是艺术,许多人本来是笨伯,而自信是可能的诗人或艺术家。他们常埋怨道:“可惜我不是一个文学家,否则我的生平可以写成一部很好的小说。”富于艺术材料的生活何以不能产生艺术呢?艺术所用的情感并不是生糙的而是经过反省。蔡琰在丢开亲生子回国时绝写不出《悲愤诗》,杜甫在“入门闻号啕,幼子饥已卒”时绝写不出《自京赴奉先县咏怀五百字》。这两首诗都是“痛定思痛”的结果。艺术家在写切身的情感时,都不能同时在这种情感中过活,必定把它加以客观化,必定由站在主位的尝受者退为站在客位的观赏者。一般人不能把切身的经验放在一种距离以外去看,所以情感尽管深刻,经验尽管丰富,终不能创造艺术。

(选自朱光潜《谈美书简》)

10.下列表述,不符合原文意思的一项是(3分)( )

A.若以实用的态度看事物,就会把它们看做实际生活的工具或障碍物,只能引起欲念或嫌恶,难以见出事物本身的美。

B.艺术本来是弥补人生和自然缺陷的。因此,如果人生和自然有了缺陷,那么,唯一的办法就只能是靠艺术来弥补。

C.许多人有富于艺术材料的生活和深刻的情感,却没有创作出艺术作品,在于他们没有在一定的距离以外将其客观化。

D.艺术家要在一定的距离上将切身的情感客观化,但同时也承认丰富的经验和深刻的情感对于艺术的重要作用。

11.第①段举的一些例子,主要用来说明什么观点?请加以概括。(3分)

答:________________________________________________________________________

12.阅读第④段至第⑥段,概述作者认为的艺术要与实际人生有距离的理由。(3分)

答:________________________________________________________________________

(二)文学类、实用类文本阅读(选考)(15分)

本题为选做题,请从甲乙两类文本中选定一类作答,不得再做另一类文本的题目。

(甲)文学类文本阅读(15分)

阅读下面的文字,完成13~15题。

故乡的沉沦

耿 立

曾看到过一幅照片:一个农民在故乡新建成的楼房前呆坐,他的肤色是久在风雨暴晒下才有的酱色。我心中涌动起莫名的风雨飘絮的黍离之情,只觉得无边的乡村在沉沦,或者说在一点点坍塌。

乡土的中国,故乡的中国,真的转换这么快?一夜之间,土地里不再种出庄稼,而是种出了成片的高楼。“农民上楼”,就如镰刀割下了谷子,这不是一次收割的事件,而是一个精神的事件。有人说这世界消失的方式不是一声巨响,而是一声呜咽。谷穗碰到镰刀是呜咽,大树遭遇斧头是呜咽,而故乡田园风光的消逝更是呜咽。

有一个成语叫背井离乡,“背”是背离,这是孩子都能理解的。但我宁愿理解“背”为背负,一个背负着故乡井水的人是有底气的,无论走到哪里都有故乡井水的滋润,有故乡做依靠。记得,在一次文人雅集的酒桌上,友人问我,你的眼睛为何这样亮?我说那是故乡的水井!又问,你的头上隐隐像有什么东西,那是什么呢?也许,是我醉酒的缘故,我回答:那是故乡的屋檐。友人愣住了,不知如何应答,他有点黯然,然后醉了。他说,我没有故乡的屋檐。然后就伏在桌子上呜呜大哭起来。

故乡是一个人的血地。台湾把故乡叫做原乡,作家钟离和说:“原乡人的血,只有回到原乡,他的血才能停止沸腾。”真是彻肤彻骨,透到了骨髓。

故乡是一种容器,故乡是收藏我们童年哭声的地方,一石一础,一草一叶,井栏树冠,那都是我们的见证,那里勾留了我们的年轮,涂抹了黄昏时我们读书的影子,还有那塞满草的窗子。当我们夜晚背诵课文的时候,常仰着脖颈望着星空,像是背诵着夜。现在那里的夜还是那样纯净么?没有一丝阴翳,没有污染,没有毁容?

一个人不能没有灵魂。曾记得一个台湾老兵的故事。他把装着故乡泥土的玻璃瓶子弄丢了,他的魂魄也随之丢失了。老兵住院,什么样的医术也疗救不了他这种思乡之痛。他的事传播开来,人们同情他,一个研究生翻找资料,在实验室里为老兵配制了他家乡的土。土里特别多放了一点盐分,用以配出老兵家人在这土地上流过的汗水。但细心的老兵呢?看出了黄土是用色素染成的!他说,故乡的土,是不可以配制的,那些童年的声音留在土里的,那些炊烟留在土里的,那些牛羊的哞叫留在土里的,这些怎能够配制出来?老兵最后说,这一瓶配出的黄土里面缺一样最要紧的东西:当初,母亲把土摊在白纸上,戴好老花镜看过、拣过,弄得干干净净,两滴眼泪落在土里,这一大瓶配置的土里却没有!

老兵带着那瓶故乡的泥土走过七个省,最后越过台湾海峡。

我不知道这个老兵最后的归宿,但我揪心的是即便他的灵魂得以还乡,在那被毁容整容后的故乡,他能找得到魂牵梦绕的归路么?他能找得到在路口大树下苦苦遥望的母亲么?

我的老家的村口也曾有几株明代的柿子树,经历了400年的沧桑啊!几年前被连根移走了。我回老家为母亲上坟,看到移走古树后留下的大大的树坑,如枯干的泪眼,无助无望。我童年留恋的柿子树,那曾荫庇过多少代家乡子孙的老树啊!

没有故乡的人,没有根基,没有身世。

“归去来兮,田园将芜胡不归?”是千年前的陶潜在时空外呼唤如今疲惫的心灵么?

其实对沉沦的故乡来讲,连荒芜也不配,只是一片钢筋水泥的狰狞。

我看不见灵魂的归路,我只隐约听见灵魂的呜咽!

(有删改)

13.下列对作品的概括和分析,正确的两项是(5分)( )

A.中国许多乡村建起了成片的高楼,表达了作者对故乡中国转换如此之快的感叹。

B.作者回忆童年在故乡背诵课文时常常仰望星空,旨在说明他从小就有高远的志向。

C.作者有意曲解成语“背井离乡”的含义,是强调故乡在游子心中无可替代的作用。

D.研究生通过造假的方式欺骗台湾老兵,虽然是好意,但极大地伤害了老兵的感情。

E.本文的语言饱含深情,深切的回忆交织着对农村现实的深刻思考,富有批判精神。

14.本文以“故乡的沉沦”为题,有什么作用?请联系全文简要回答。(4分)

答:________________________________________________________________________

15.文中说:“一个人不能没有灵魂。”结合本文,联系实际,谈谈你对这句话的认识和感悟。(6分)

答:________________________________________________________________________

(乙)实用类文本阅读(15分)

阅读下面的文字,完成13~15题。

中国文化大规模走向世界

陈 原

从今年10月开始,比利时各地几乎成了中国文化的舞台。大红灯笼高高悬挂,典雅的牌楼竖立在市中心广场,茶室茶亭弥漫着浓郁的茶香,各大小展览馆不断展示中国从古至今的艺术精品,各剧院不时演出中国的话剧、戏曲、音乐、舞蹈、歌曲、木偶、皮影、武术,中国电影在这里上演,莫言、余华、叶延滨等中国著名作家也来到这里访问,一系列有关中国的文化讲座在这里持续亮相。所有这一切,都因为欧罗巴利亚中国艺术节的举行!

比利时皇家大法院距今已有120多年的历史,雄踞布鲁塞尔的最高处,除教堂之外,129米的总高度使它成为欧洲现存最高大的古典建筑。在这样一个巨大的欧洲古典建筑上空悬着巨幅中国山水画,不能不令人震惊。巨作的作者就是徐龙森,他近年来创作的10幅山水巨作这次都展现在欧洲观众眼前,尺度高达十米以上,让人感觉恍若置身群山之间。比利时视觉艺术家科恩感慨道:“这些画给我留下了深刻的印象。置身这里的群山之中,我觉得自己似乎无法呼吸。它们让我产生了一种时空的错位感,不知自己身在何处。这确实是一种全新的艺术创作形式。”比利时社会科学院院士、汉学家魏查理说:“看到这些画,我的第一感觉是自己仿佛到了四川或者福建。这些画既传统又现代,把它们与这座古老的建筑放在一起显得很和谐。”

徐龙森的画作显示出艺术在中国正在成为一种生活的体验。展示中国人的生活状态,就是欧罗巴利亚中国艺术节的主旨。在这里,已经将中国文化走出去的概念,从过去文学艺术的小范畴,扩展为文化生活的方方面面,文学、舞蹈、戏曲、话剧、木偶、杂技、皮影、交响乐、民乐、电影、美术、书法、摄影之外,还增添了园林、中医、建筑、文物、茶艺、日常生活等,色彩缤纷、无所不包,让欧洲观众全方位地了解中国人。

欧洲观众的文化素养很高,对文化展览和演出都讲究来龙去脉,追究文化背后的历史和生活。因此,讲座和研讨会也是欧罗巴利亚中国艺术节的重要内容,通过中国文化专家的大量讲解,让欧洲观众深入了解相关知识,并与中国文学家、艺术家互动,沟通双方的感情,弥补文化方面的差异。已经举办过的讲座和研讨会,总是座无虚席,发言、提问踊跃,问题之细,表现出听众对中国社会生活的热情。

在欧罗巴利亚中国艺术节中,中国与比利时的文化对话特别令人关注,对话的表现方式是将中国艺术与比利时艺术同台展出,或者联合演出。从这些对话中,我们能深切感受到文化背后的生命状态。“事物状态——中比当代艺术交流展”选择了中国艺术家26位,比利时艺术家24位。欧盟委员会主席巴罗佐看后认为:中国的当代艺术实在令人赞叹。

文化部外联局局长董俊新说,从中法文化年、中俄文化年到这次欧罗巴利亚中国艺术节,都是双方共同举办的,这样的形式比中国主办更具影响力,可以深入所在国的社会,在主流剧院和展览馆进行。这些文化活动,不但将中国的传统文化、民间文化、少数民族文化全面地推向世界,而且也注重介绍中国的当代文化创造,展现改革开放以来中国在文化建设方面的巨大成就。除了重大的艺术节外,目前中国在世界各地已经建立的7个文化中心也发挥着重要作用。在文化中心,展示中国文化是一种常态。今后,还将有更多的文化中心相继建立,中国文化大规模走出去正在成为现实!

今天的比利时,在机场,你会听见一声轻轻的中国话问候;在咖啡馆,或许服务员会向你要一元中国“小费”留作纪念;在巧克力店,可能会询问你来自何方,当听见北京时,你会听见一句惊呼:啊,奥林匹克!这就是今天的比利时,中国文化在这里落地开花。

(选自《人民日报》,有删改)

13.下列对文章的概括和分析,最恰当的两项是(5分)( )

A.欧罗巴利亚中国艺术节的举行,使一系列有关中国的文化讲座在比利时陆续亮相。

B.雄踞布鲁塞尔最高处的比利时皇家大剧院距今已有120多年的历史,在目前欧洲现存的古典建筑中它是最高的。

C.在欧罗巴利亚中国艺术节中,中国与比利时的文化对话特别引人关注,从这些对话中我们能深切感受到文化背后的生命状态。

D.徐龙森的画作显示出艺术在中国成为了一种生活的体验。展示中国人的状态,就是欧罗巴利亚中国艺术节的主旨。

E.欧罗巴利亚中国艺术节,是由中国和比利时双方共同举办的,这样的形式比中国主办更具影响力。这些文化活动只注重将中国的传统文化、民间文化、少数民族文化全面地推向世界。

14.新闻介绍了欧罗巴利亚中国艺术节的哪些主要活动?试分点归纳。(4分)

答:________________________________________________________________________

15.欧罗巴利亚中国艺术节的这些活动各有什么意义?请结合文本作简要评价。(6分)

答:________________________________________________________________________

四、语言文字运用(13分)

16. 阅读下面的文字,按要求回答问题。(3分)

与阮籍一样,陶潜采取的是一种政治性的退避。__只有他,才真正做到了这种退避,宁愿归耕田园,蔑视功名利__(lù),“宁固穷以济意,不委屈而累己。既轩冕之非荣,岂缊袍之为耻。诚谬会以取拙,且欣然而归止”。不是外在的轩冕荣华、功名学问,而是内在的人格和不委屈而累己的生活,才是正确的人生道路。所以只有他,算是找到了生活快乐和心灵安慰的较为现实的途径。无论人生感叹或政治忧伤,都在自然和农居生活的爱恋中得到了安息。

(1)在①处填上一个恰当的关联词。(1分)

答:________________________________________________________________________

(2)在②处根据拼音填写正确的汉字。(1分)

答:________________________________________________________________________

(3)画线的句子在语言表达方面存在错误,请找出并加以改正。(1分)

答:__________________改为____________________________________________

17.将下面的四个短句改写成一个长单句,要求语言通顺,语意不变,可适当增删词语。(3分)

①近日,中国科学家打造出了一顶新颖的“纳米皇冠”。

②皇冠制作的原材料是铂金。

③皇冠的制作运用了分子纳米技术。

④皇冠制作过程中得到了中国自然科学基金委员会“纳米科技重大研究计划”小组以及中国科学院“百人计划”小组的支持。

答:________________________________________________________________________

18.阅读下面的材料,回答问题。(7分)

最近,在一堂大学语文应用写作的公共课上,彭书雄教授随机让5名学生口述请假条,竟没有一人能准确回答。《百家讲坛》特约主讲、武汉大学教授李敬一也表示,即便是博士生,有时候也会在论文中出现标点错误和错字。

对于上述现象,你有什么看法?请简要阐述。(要求:表达简明连贯,言之成理,150字左右)

答:________________________________________________________________________

________________________________________________________________________

五、写作(70分)

19.阅读下面的材料,根据要求写一篇不少于800字的议论文或记叙文。

两只蚂蚁想翻越一段墙,寻找墙那边的食物。这段墙长有20米,高有10米。其中一只蚂蚁来到墙脚就毫不犹豫地向上爬去,可每爬到大半时,就会因劳累跌落下来。可是它不气馁,它相信只要付出就会有回报。一次次跌下来,它都迅速地调整一下自己,重新开始向上爬。

而另一只蚂蚁观察一下,决定绕过这段墙。很快地,这只蚂蚁绕过这段墙来到食物面前,开始享用起来;而那只“勇敢”“坚定”的蚂蚁还在不停地跌落下去,又重新开始。

请结合上面的材料,以“坚持与选择”为题目作文。

要求:(1)必须符合文体要求;(2)角度自选;(3)立意自定;(4)不得抄袭,不得套作。

全品高考网:www.canpoint.cn阶段评估检测(七) ◄测评手册

阶段评估检测(七)

满分150分,考试时间150分钟。

一、古代诗文阅读(27分)

(一)默写常见的名句名篇(6分)

1.补写出下列名句名篇中的空缺部分。

(1)总角之宴,____________________。(《诗经·氓》)

(2)箫鼓追随春社近,__________________。(陆游《游山西村》)

(3)是日也,天朗气清,________________。(王羲之《兰亭集序》)

(4)____________________,郁郁青青。(范仲淹《岳阳楼记》)

(5)浩荡离愁白日斜,____________________。(龚自珍《己亥杂诗》)

(6)斯是陋室,____________________。(刘禹锡《陋室铭》)

(二)文言文阅读(15分)

阅读下面的文言文,完成2~5题。

朱治传

[晋]陈寿

朱治字君理,丹杨故鄣人也。初为县吏,后察孝廉,州辟从事,随孙坚征伐。中平五年,拜司马,从讨长沙、零、桂等三郡贼周朝、苏马等,有功,坚表治行都尉。从破董卓于阳人,入洛阳。表治行督军校尉,特将步骑,东助徐州牧陶谦讨黄巾。

会坚薨,治扶翼策,依就袁术。后知术政德不立,乃劝策还平江东。时太傅马日在寿春,辟治为掾,迁吴郡都尉。是时吴景已在丹杨,而策为术攻庐江,于是刘繇恐为袁、孙所并,遂嫌隙。而策家门尽在州下,治乃使人于曲阿迎太妃及权兄弟,所以供奉辅护,甚有恩纪。治从钱唐欲进到吴,吴郡太守许贡拒之于由拳,治与战,大破之。贡南就山贼严白虎,治遂入郡,领太守事。策既走刘繇,东定会稽。

权年十五,治举为孝廉。后策薨,治与张昭等共尊奉权。建安七年,权表治为(九真)[吴郡]太守,行扶义将军,割娄、由拳、无锡、毗陵为奉邑,置长吏。征讨夷越,佐定东南,禽截黄巾余类陈败、万秉等。黄武元年,封毗陵侯,领郡如故。二年,拜安国将军,金印紫绶,徙封故鄣。

权历位上将,及为吴王,治每进见,权常亲迎,执版交拜,飨宴赠赐,恩敬特隆,至从行吏,皆得奉贽私觌,其见异如此。

初,权弟翊,性峭急,喜怒快意,治数责数,谕以道义。权从兄豫章太守贲,女为曹公子妇,及曹公破荆州,威震南土,贲畏惧,欲遣子入质。治闻之,求往见贲,为陈安危,贲由此遂止。

权常叹治忧勤王事。性俭约,虽在富贵,车服惟供事。权优异之,自令督军御史典属城文书,治领四县租税而已。然公族子弟及吴四姓多出仕郡,郡吏常以千数,治率数年一遣诣王府,所遣数百人,每岁时献御,权答报过厚。是时丹杨深地,颇有奸叛,亦以年向老,思恋土风,自表屯故鄣,镇抚山越。诸父老故人,莫不诣门,治皆引进,与共饮宴,乡党以为荣。在故鄣岁余,还吴。黄武三年卒,在郡三十一年,年六十九。

(选自《三国志·吴书十一》)

〔注〕 ①扶翼:辅佐。②贽:礼物。③觌:见。

2.对下列加点词的解释,不正确的一项是(3分)( )

A.会坚薨 会:适逢 B.贡南就山贼严白虎 就:投靠

C.治数责数 数:屡次 D.莫不诣门 诣:造访

3.下列全都表现朱治忧勤王事的一组是(3分)( )

①后知术政德不立,乃劝策还平江东 ②后策薨,治与张昭等共尊奉权 ③治每进见,权常亲迎④征讨夷越,佐定东南,禽截黄巾余类陈败、万秉等 ⑤治闻之,求往见贲,为陈安危 ⑥以年向老,思恋土风,自表屯故鄣

A.①②⑥ B.②③⑤ C.②④⑥ D.①④⑤

4.下列对文章有关内容的概括与分析,不正确的一项是(3分)( )

A.朱治一开始做了县吏,又跟随孙坚征讨逆贼周朝、苏马等人,立下战功,而后率领步兵骑兵帮助陶谦讨伐黄巾军。

B.孙坚去世后,朱治又辅佐孙策,劝孙策脱离袁术而还军平定江东;孙策死后,朱治又与张昭等人共同拥立孙权。

C.曹操攻破荆州,孙贲害怕,想送儿子到曹营做人质,朱治前往劝孙贲,陈述利害,让他不要依附曹操,孙贲没有听从。

D.朱治一生效力于孙家,为东吴立下战功,然而本性节约,只领四县租税;他热情对待乡亲父老,也深受乡亲们的爱戴。

5.把文中画横线的句子翻译成现代汉语。(6分)

(1)于是刘繇恐为袁、孙所并,遂搆嫌隙。(3分)

译文:________________________________________________________________________

(2)策既走刘繇,东定会稽。(3分)

译文:________________________________________________________________________

(三)古代诗歌阅读(6分)

6.阅读下面这首诗歌,回答问题。

金陵怀古

[宋]王珪

怀乡访古事悠悠,独上江城满目秋。一鸟带烟来别渚,数帆和雨下归舟。

萧萧暮吹惊红叶,惨惨寒云压旧楼。故国凄凉谁与问,人心无复更风流。

〔注〕 王珪时任北宋左相,写作此诗时,北宋在与西夏的两次战争中均遭失败。

(1)简析首联在全诗中的作用。(3分)

答:________________________________________________________________________

(2)第六句有一个字用得好,请找出来,并简要分析好在哪里。 (3分)

答:________________________________________________________________________

________________________________________________________________________

二、文学名著、文化经典阅读(16分)

(一)文学名著阅读(10分)

7.下列各项中对作品故事情节的表述,不正确的两项是(5分)( )

A.吴荪甫异样地狂笑着,站起身来就走出了那书房,一直跑上楼去。现在知道什么都完了,他倒又镇静起来了。(《子夜》)

吴荪甫彻底破产了,绝望得甚至想自杀。但他告诉丁医生自己没事,随后就叫妻子赶快收拾,晚上就上轮船,他将携着全家人到牯岭避暑。

B.他们开始捶门,又叫“三少爷”,也没有用。觉慧在里面大声说:“我不开。我屋里没有鬼!”他索性走到床前,躺下去,用手蒙住耳朵,不去听外面的叫声。(《家》)

高老太爷病重,在四太太的提议下,开始为高老太爷捉鬼治病。当整个高公馆都笼罩在浓重的祈神活动之中时,“只有这两扇门是紧紧关住的”。

C.如果将来并无子女,则夫妇双方的财产,包括动产不动产,一律全部互相遗赠。任何一方身故后,不得举办遗产登记,因为免除该手续才不致损害继承人或权益持有者。(《欧也妮·葛朗台》)

欧也妮答应嫁给公证人的儿子、初级裁判所所长特·蓬风。这是特·蓬风在签订婚书的时候写的条款。

D.对涅赫柳多夫来说,决定命运的时刻到了。他不断责备自己,上次见面没有说出主要的话,就是他打算跟她结婚。现在他下定决心要把这话说出来。(《复活》)

文中的“上次”指的是第一次探监,而“现在”指的是第二次探监,涅赫柳多夫决心向马斯洛娃求婚。

E.看了他那凶狠的眼光,她打算逃开去,他又抓住她,摇晃她,把她推倒在地上,然后拽着她漂亮的胳膊,拖着她迈开大步向罗兰塔拐角上走去。到了那里,他转身问她道:“最后一次回答我,你愿不愿意属于我?”她使劲地回答:“不!”(《巴黎圣母院》)

弗罗洛威胁爱斯梅拉达,要她顺从自己,不然就把她杀了。爱斯梅拉达宁死不屈。愤怒的弗罗洛把她拖到罗兰塔,交给军警。

8.简答题。(选做一题)(5分)

(1)《三国演义》中,曹操在赤壁战败后,过华容道时曾“三笑一哭”。请问,他为谁而哭?最后一笑引来了谁?接下来发生了什么事?

(2)阅读下面的《红楼梦》选段,根据原著故事情节,回答问题。

“嗳,我也是知道艰难的!但俗语说的,‘瘦死的骆驼比马大’,凭他怎么,你老拔根寒毛,比我们的腰还粗呢。”

这段话是谁说的?是对谁说的?为何说这段话?

答第()题:________________________________________________________________________

(二)文化经典阅读(6分)

9.阅读下面的《论语》和《孟子》选段,回答问题。

①孟子曰:“人之所不学而能者,其良能也;所不虑而知者,其良知也。孩提之童无不知爱其亲者,及其长也,无不知敬其兄也。亲亲,仁也;敬长,义也。无他,达之天下也。”(《孟子·尽心上》)

②其为人也孝弟,而好犯上者,鲜矣;不好犯上,而好作乱者,未之有也。君子务本,本立而道生。(《论语·学而》)

(1)孟子认为,孩提之童都知道孝敬父母,等长大后都知道尊敬兄长,说明爱亲敬长是人的天性,有了这些就能够___________,这个选段体现了孟子的______________的观点。(2分)

(2)孔子所谓的“本”指什么?为什么说“本立而道生”?请结合上面的选段简要分析。(4分)

答:________________________________________________________________________

三、现代文阅读(24分)

(一)论述类文本阅读(9分)

阅读下面的文字,完成10~12题。

从孩子的照相说起

鲁 迅

①因为长久没有小孩子,曾有人说,这是我做人不好的报应,要绝种的。房东太太讨厌我的时候,就不准她的孩子们到我这里玩,叫作“给他冷清冷清,冷清得他要死!”但是,现在却有了一个孩子,虽然能不能养大也很难说,然而目下总算已经颇能说些话,发表他自己的意见了。不过不会说还好,一会说,就使我觉得他仿佛也是我的敌人。

②他有时对于我很不满,有一回,当面对我说:“我做起爸爸来,还要好……”甚而至于颇近于“反动”,曾经给我一个严厉的批评道:“这种爸爸,什么爸爸!?”

③我不相信他的话。做儿子时,以将来的好父亲自命,待到自己有了儿子的时候,先前的宣言早已忘得一干二净了。况且我自以为也不算怎么坏的父亲,虽然有时也要骂,甚至于打,其实是爱他的。所以他健康,活泼,顽皮,毫没有被压迫得瘟头瘟脑。如果真的是一个“什么爸爸”,他还敢当面发这样反动的宣言么?

④但那健康和活泼,有时却也使他吃亏,九一八事件后,就被同胞误认为日本孩子,骂了好几回,还挨过一次打——自然是并不重的。这里还要加一句说的听的,都不十分舒服的话:近一年多以来,这样的事情可是一次也没有了。

⑤中国和日本的小孩子,穿的如果都是洋服,普通实在是很难分辨的。但我们这里的有些人,都有一种错误的速断法:温文尔雅,不大言笑,不大动弹的,是中国孩子;健壮活泼,不怕生人,大叫大跳的,是日本孩子。

⑥然而奇怪,我曾在日本的照相馆里给他照过一张相,满脸顽皮,也真像日本孩子;后来又在中国的照相馆里照了一张相,相类的衣服,然而面貌很拘谨,驯良,是一个地道的中国孩子了。

⑦为了这事,我曾经想了一想。

⑧这不同的大原因,是在照相师的。他所指示的站或坐的姿势,两国的照相师先就不相同,站定之后,他就瞪了眼睛伺机摄取他以为最好的一刹那的相貌。孩子被摆在照相机的镜头之下,表情是总在变化的,时而活泼,时而顽皮,时而驯良,时而拘谨,时而烦厌,时而疑惧,时而无畏,时而疲劳……照住了驯良和拘谨的一刹那的,是中国孩子相;照住了活泼或顽皮的一刹那的,就好像日本孩子相。

⑨驯良之类并不是恶德。但发展开去,对一切事无不驯良,却绝不是美德,也许简直倒是没出息。“爸爸”和前辈的话,固然也要听的,但也须说得有道理。假使有一个孩子,自以为事事都不如人,鞠躬倒退;或者满脸笑容,实际上却总是阴谋暗箭,我实在宁可听到当面骂我“什么东西”的爽快,而且希望他自己是一个东西。

⑩但中国一般的趋势,却只在向驯良之类——“静”的一方面发展,低眉顺眼,唯唯诺诺,才算一个好孩子,名之曰“有趣”。活泼,健康,顽强,挺胸仰面……凡是属于“动”的,那就未免有人摇头了,甚至于称之为“洋气”。又因为多年受着侵略,就和这“洋气”为仇;更进一步,则故意和这“洋气”反一调:他们活动,我偏静坐;他们讲科学,我偏扶乩;他们穿短衣,我偏着长衫;他们重卫生,我偏吃苍蝇;他们壮健,我偏生病……这才是保存中国固有文化,这才是爱国,这才不是奴隶性。

⑪其实,由我看来,所谓“洋气”之中,有不少是优点,也是中国人性质中所本有的,但因了历朝的压抑,已经萎缩了下去,现在就连自己也莫名其妙,统统送给洋人了。这是必须拿它回来——恢复过来的——自然还得加一番慎重的选择。

⑫即使并非中国所固有的罢,只要是优点,我们也应该学习。即使那老师是我们的仇敌罢,我们也应该向他学习。我在这里要提出现在大家所不高兴说的日本来,他的会摹仿,少创造,是为中国的许多论者所鄙薄的,但是,只要看看他们的出版物和工业品,早非中国所及,就知道“会摹仿”绝不是劣点,我们正应该学习这“会摹仿”的。“会摹仿”又加以有创造,不是更好么?否则,只不过是一个“恨恨而死”而已。

⑬我在这里还要附加一句像是多余的声明:我相信自己的主张,绝不是“受了帝国主义者的指使”,要诱中国人做奴才;而满口爱国,满身国粹,也于实际上的做奴才并无妨碍。

八月七日

(选自鲁迅《且介亭文集》,略有删改)

〔注〕 ①扶乩:亦称扶箕、扶鸾,一种请神的迷信活动。由两人扶一丁字形木架,下垂木杆在沙盘上画字,作为指示。

10.下列对原文的概括和分析,不正确的一项是(3分)( )

A.儿子对鲁迅“这种爸爸,什么爸爸!?”的批评,虽颇近于“反动”,却并未受到鲁迅的压制,因而其子健康、活泼、顽皮,而非瘟头瘟脑。

B.中日的小孩子,若皆着洋服,则温文尔雅,不大言笑动弹的,是中国孩子;健壮活泼,不怕生人,大叫大跳的,是日本孩子。

C.驯良之类并非恶德,但发展成自以为是,事事都不如人,鞠躬倒退,或者满脸笑容,实际上却总是阴谋暗箭,就绝非美德了。

D.作者认为,所谓“洋气”之中,有不少优点,也是中国人性质中本有的,就必须有选择地拿它回来,恢复过来。

11.请简要分析第段到第⑬段的论述思路。(3分)

答:________________________________________________________________________

12.联系全文,谈谈你对第⑬段画线句子的理解。(3分)

答:________________________________________________________________________

(二)文学类、实用类文本阅读(选考)(15分)

本题为选做题,请从甲乙两类文本中选定一类作答,不得再做另一类文本的题目。

(甲)文学类文本阅读(15分)

阅读下面的文字,完成13~15题。

晶莹的泪滴

陈忠实

①我手里捏着一张休学申请书朝教务处走去。

②我要求休学一年。

③我敲了敲教务处的门板,获准以后我便推开了门,一位年轻的女先生正伏在米黄色的办公桌上,手里拿着长杆蘸水笔在一厚本表册上填写着什么。“老师,给我开一张休学证书。”

④她抬起头来,诧异地瞅了我一眼,拎起我的申请很快看完了,又专注地把目光停滞在纸页下端班主任签写的一行意见和校长更为简洁的意见上面,似乎两个人连姓名在内的十来个字的意见批示,看上去比我大半页的申请书还要费时更多。她终于抬起头来问:

“就是你写的这些理由吗?”

“是的。”

“不休学不行吗?”

“不行。”

“亲戚全都帮不上忙吗?”

“亲戚……也都穷。”

“可是……你休学一年,家里的经济状况也不见得能改变,一年后你怎么能保证复学呢?”

⑤于是我就信心十足地告诉她我父亲的精确计划:待到明年我哥哥初中毕业,父亲让他报考师范学校,师范生的学杂费和伙食费全由国家供给,据说还发三块零花钱。那时候我就可以复学接着念初中了。我没有做更多的解释。我的爱面子的弱点早在此前已经形成。我不想再向任何人重复叙述我们家庭的困窘。

⑥她轻轻舒了口气,拉开抽屉取出一个公文本在桌子上翻开,又停下手,问:“你家里就再想不出办法了?”我看着那双浮着忧郁气色的眼睛,忽然联想到姐姐的眼神。突然意识到因为我的休学致使她心情不好。她只是教务处的一位抄抄写写的年轻职员,我和她几乎没有说过话,甚至至今也不知道她的姓名。我便说:“老师,没关系。休学一年没啥关系,我年龄小。”她说:“白白耽搁一年多可惜!”随之又换了一种口吻说:“我知道你的名字也认得你。每个班前三名的学生我都认识。”我的心情突然灰暗起来而没有再开口。

⑦她终于落笔填写了公文函,取出公章在下方盖了,又在切割线上盖上一枚合缝印章,“吱吱吱”撕下并不交给我,放在桌子上,然后把我的休学申请书抹上糨糊后贴在公文存根上,做完这一切才重新拿起休学证书交给我说:“装好。明年复学时拿着来找我。”

⑧我向她深深地鞠了躬就走出门去。我听到背后“咣当”一声关门的声音,同时也听到一声“等等”。她拢了拢齐肩的头发朝我走来,和我并排在廊檐下的台阶上走着。我想尽快远离正在迎接新学期的洋溢着欢乐气氛的学校大门,便低着头加快了脚步,她又喊了一声“等等”,走过来拍了拍我的书包:“别把休学证弄丢了。”我点点头。她这时才有一句安慰我的话:“我同意你的打算,休学一年不要紧,你年龄小。”

⑨我抬头看她,猛然看见那双眼睫毛很长的眼眶里溢出泪水来,像雨雾中正在涨溢的湖水,泪珠在眼眶里打着旋儿,晶莹透亮。我垂下头避开目光,要是再在她的眼睛处多驻留一秒,我肯定就会号啕大哭。我低着头咬着嘴唇,脚下盲目地拨弄着一块碎瓦片来抑制情绪,感觉到有一股热辣辣的酸流从鼻腔倒灌进喉咙里去。我终于扬起头鼓起劲儿说:“老师……我走了……”她的手轻轻搭上我的肩头:“记住,明年的今天来报到复学。”

⑩我看见两滴晶莹的泪珠从她眼睫毛上滑落下来,缓缓流过一段就在鼻翼两边挂住。我再次虔诚地深深鞠躬,然后就转过身走掉了。

……

⑪二十五年后,卖树卖树根供我念书的父亲在弥留之际,对坐在他身边的我说:“我有一件事对不住你……,如果不是错过了一年,你就不会错过那年的高考录取了……”我对已经跨进黄泉路上半步的依然向我忏悔的父亲,讲了那一串泪珠的经历,父亲便安然合上了眼睛,喃喃地说:“可你……怎么……不早点给我……说女先生哩……”

⑫我今天终于把近四十年前的这一段经历写出来,对自己算是一种虔诚祈祷,当各种欲望膨胀成一股强大的浊流冲击所有大门、窗户和每一个心扉的当今,我便企望自己如女老师那种泪珠的泪腺不至于堵塞,更不敢枯竭,那是滋养生命灵魂的泉源,也是滋润民族精神的泉源……

13.下列对作品的概括和分析,不正确的两项是(5分)( )

A.“晶莹的泪滴”指教务处女老师为我失学感到无奈、伤心而流下的泪水,也喻指老师真诚、纯洁、善良的心灵。

B.“我”没有做更多的解释,原因是“我和她几乎没有说过话,甚至至今也不知道她的姓名”。

C.“我看着那双浮着忧郁气色的眼睛,忽然联想到姐姐的眼神。”运用细节描写,强调女老师如姐姐一样温柔地关心、鼓励“我”,为“我”分担痛苦。

D.文中写父亲在弥留之际的话,旨在表达父亲对当年自己决定的后悔,突出当时女老师劝“我”不要休学的意义。

E.“我”十四岁时没有把女老师“一串泪珠的经历”告诉父亲,是因为当年“我”对父亲让我休学十分不满。

14.在办理休学手续的过程中,“我”的心情经历了怎样的变化?请简要概括(4分)

答:________________________________________________________________________

15.结尾段画线文字的表达有何特色,这样写有何好处?请谈谈你的看法。(6分)

答:________________________________________________________________________

(乙)实用类文本阅读(15分)

阅读下面的文字,完成13~15题。

“数字边疆”拓荒者

张 懿

如果有人在40岁之前从未接触过电脑,那么他还有没有机会成为计算机专家,并成为该领域的院士?这是一个难以想象的挑战。

2011年底,新一届两院院士评选结果揭晓,新疆大学教授吾守尔·斯拉木当选为中国工程院信息与电子工程学部院士,也成为第一位维吾尔族院士。正是这位计算机领域的权威专家,第一次摸电脑时已年届不惑;而此后的30年,他勤力摸索,大胆创新,以一系列开创性的技术成就,极大地改变了边疆地区的信息化面貌,也让自己成为民族语言文字信息处理领域当之无愧的领军人物。

吾守尔出生在伊犁地区的一户农家,童年生活,是和种地、割麦子、放牧牛羊联系在一起的。吾守尔的父母都是农民,都不识字,但他们对子女的教育却极其重视。小学、中学,吾守尔成绩一直名列前茅,18岁那年,他考上了新疆大学。循着年少时那股探索自然的热情,吾守尔选择了物理系。在以优异的成绩完成学业后,吾守尔来到喀什师范学院担任教师。1981年,吾守尔从喀什师范学院调到母校新疆大学任教。顺应时代需求,新疆大学希望能开设计算机方面的课程。搞过数字电路的吾守尔被选中,前往北京和上海的学校进修。

这两次进修为自己的人生开启了一扇窗户,使他真正有机会进入计算机领域。也正是在进修过程中,他想到,能不能用少数民族语言实现计算机信息处理、控制和应用呢?当时,计算机的操作系统刚实现从英文到中文的转化,而且许多工作是由国内顶尖专家完成的。少数民族要进入信息化时代,没有基础,要挑战的难点更多,简直就是一个无可企及的梦想。

但吾守尔是一个坚持的人,同时,新疆大学的领导对他的想法给予支持。学校出资3万多元,作为研究经费。凭着这笔当时颇为可观的“投入”,吾守尔和他的团队开始了艰难而卓绝的研究。

只有用“拓荒”这个词,才能准确描述吾守尔的工作。就像游客无法体会在山岩上筑路的辛苦,普通人也许很容易把如今深深浸透入生活的IT工具,当成是理所应当。但你想一下,如今在新疆应用的大量信息化平台,只要与维语相关,背后的技术体系都是基于吾守尔团队的工作,就知何其不易。

在吾守尔之前,维语别说没有输入法,甚至连字母在键盘上的键位布局,都缺乏标准。这也难怪,过去维语还从来没有数字化输入的需求。而目前维语字符集编码和在键盘上的键位布局、字符点阵及数据,都由吾守尔主持设计,并最终成为国家标准。

从最早的DOS操作系统,到后来各个版本的Windows视窗,吾守尔团队开发了30多种民族语言操作系统及应用软件,还有一整套维文信息处理的理论、体例、技术体系和各个门类的信息化应用平台。由于维吾尔族文字与哈萨克族、柯尔克孜族比较类似,吾守尔团队的研究功效,让维、哈、柯三个民族跨越千万人受益。

最近几年,吾守尔团队正在努力攻关多媒体民文信息处理这更高层次、更具人工智能色彩的技术。他们开发的文字识别、语音识别与合成系统,能让计算机识别维语文字、听“懂”维语等民族语言,也能让电脑根据文字自动读出文章。

维吾尔族习惯把志存高远、无私奉献的男子比作雄鹰。吾守尔·斯拉木说,如果说高飞的雄鹰离不开阳光赋予的温暖和活力,离不开天空赋予的广阔和包容的话,那么,他愿把党和政府比作阳光,把培养他的新疆大学比作蓝天。他希望自己能坚持到生命的最后一刻,把所有能做到的,都回报给这片他挚爱的土地。

(选自2012年1月17日《文汇报》)

13.下列对文章的概括和分析,最恰当的两项是(5分)( )

A.文章以设问开头,用以突出吾守尔教授的挑战精神,最后引用他的话语作结,突出他的精神境界,结构完整,布局合理。

B.文章先写吾守尔教授被评为两院院士,再写他的家庭背景和在计算机领域的探索,倒叙和插叙交替运用,线索分明,中心突出。

C.文章略写吾守尔的人生经历,详写其在民族语言文字信息处理领域的研究过程和成果,有力地展现了“数字边疆”拓荒者的形象。

D.吾守尔·斯拉木在40岁之前从未接触过电脑,40岁以后半路出家,成为民族语言文字信息处理领域的领军人物,是自学成才的典型。

E.吾守尔·斯拉木教授是我国边疆地区信息化的拓荒者,他以一系列开创性的技术成就,极大地改变了边疆地区的信息化面貌。

14.吾守尔·斯拉木在民族语言文字信息处理领域有哪些主要成就?(4分)

答:________________________________________________________________________

15.文章以叙述为主,偶用议论,恰到好处,试举两处“偶用议论” 的例子,并分析其好处。(6分)

答:________________________________________________________________________

四、语言文字运用(13分)

16.阅读下面一段文字,按要求问答问题。(3 分)

深度不足的演讲家,常用长度来补偿。对于此类演讲家,余光中亦有妙论①:“有一种人演讲,不但贪长,而且逞响。愈浅的人愈迷信滔滔的声浪,以为‘’②(原比喻人的名声很大,此处形容声音很大)便足以征服世界。以前不用麦克风,这种人容易声嘶力竭;现在有了机器助阵,等于有了武器,这种人在回声响亮的喧_____(xiāo)里,幻觉自己的每句话都是警世的真理了。”

(1)①处画线的句子有语病,请改正。(1分)

答:________________________________________________________________________

(2)请根据提示,在②处写一个四字成语。(1 分)

答:________________________________________________________________________

(3)请在③处根据拼音填写正确的汉字。(1 分)

答:________________________________________________________________________

17.用一句话概括下面材料的主要内容。(不超过35字)(3分)

世界上一共有七千种语言,到本世纪末,至少一半会灭绝。也就是说,平均不到两个星期就会有一种语言从地球上消失。物种和生态系统正在崩溃,随着小的、未书面化的语言的消失,人们关于这些物种和生态系统的传统知识也会失传。

答:________________________________________________________________________

18.阅读下面的材料,回答问题。(7分)

人民网针对某省今年再次不公布高考状元与排名一事,在人民调查栏目进行了“您是否赞成公布高考‘状元’”网上调查。结果共有1126人参与了此次投票,其中,不赞成公布高考状元的共有761票,占总投票数的67.6%;赞成公布高考状元的共有220票,占总数的19.5%;无所谓与其他共145票,占12.9%。

对于上述做法,你有什么看法?请简要阐述。(要求:表达简明连贯,言之成理,150字左右)

答:________________________________________________________________________

________________________________________________________________________

五、写作(70分)

19.阅读下面的材料,根据要求写一篇不少于800字的议论文或记叙文。

一位母亲带正在读小学三年级的女儿去巴黎卢浮宫看画展。出发前,母亲就要求孩子读很多相关的书,并做详细的笔记。到了卢浮宫,母亲一直督促着孩子看画、记笔记。有时,孩子在某一幅自己喜爱的画前面多站一会儿,母亲便催促着:快,快,下一张,时间不多,卢浮宫的名作太多了。

对此,你一定有自己的体验或看法,请作文。

要求:(1)必须符合文体要求;(2)角度自选;(3)立意自定;(4)题目自拟;(5)不得抄袭,不得套作。 全品高考网:www.canpoint.cn阶段评估检测(八) ◄测评手册

阶段评估检测(八)

满分150分,考试时间150分钟。

一、古代诗文阅读(27分)

(一)默写常见的名句名篇(6分)

1.补写出下列名句名篇中的空缺部分。

(1)扪参历井仰胁息,________________。(李白《蜀道难》)

(2)____________,不可说也!(《诗经·氓》)

(3)______________,凌万顷之茫然。(苏轼《赤壁赋》)

(4)母、孙二人,更相为命,____________________。(李密《陈情表》)

(5)______________,川泽纡其骇瞩。(王勃《滕王阁序》)

(6)凭谁问:____________,尚能饭否?(辛弃疾《永遇乐·京口北固亭怀古》)

(二)文言文阅读(15分)

阅读下面的文言文,完成2~5题。

董生,徐州人,好击剑,每慷慨自负。偶于途中遇一客,跨蹇同行。与之语,谈吐豪迈;诘其姓字,云辽阳佟姓;问何往,曰:“余出门二十年,适自海外归耳。”董曰:“君遨游四海,阅人綦多,曾见异人否?”佟问异人何等,董乃自述所好,恨不得异人之传。佟曰:“异人何地无之?要必忠臣孝子始得传其术也。”董又奋然自许,即出佩剑,弹之而歌;又斩路侧小树以矜其利。佟掀髯微笑,因便借观。董授之。展玩一过,曰:“此甲铁所铸,为汗臭所蒸,最为下品。仆虽未闻剑术,然有一剑颇可用。”遂于衣底出短刃尺许,以削董剑,脆如瓜瓠,应手斜断如马蹄。董骇极,亦请过手,再三拂拭而后返之。邀佟至家,坚留信宿。叩以剑法,谢不知。董按膝雄谈,惟敬听而已。

更既深,忽闻隔院纷挐。隔院为生父居,心惊疑,近壁凝听,但闻人作怒声曰:“教汝子速出即刑便赦汝。”少顷,似加搒掠,呻吟不绝者真其父也。生捉戈欲往,佟止之曰:“此去恐无生理,宜审万全。”生皇然请教,佟曰:“盗坐名相索,必将甘心焉。君无他骨肉,宜嘱后事于妻子,我启户为君警厮仆。”生诺,入告其妻,妻牵衣泣。生壮念顿消,遂共登楼上,寻弓觅矢,以备盗攻。仓皇未已,闻佟在楼檐上笑曰:“贼幸去矣。”烛之已杳。逡巡出,则见翁赴邻饮,笼烛方归,惟庭前多编管遗灰焉。乃知佟异人也。

异史氏曰:“忠孝,人之血性;古来臣子而不能死君父者,其初岂遂无提戈壮往时哉,要皆一转念误之耳。昔解缙与方孝儒相约以死,而卒食其言;安知矢约归后,不听床头人呜泣哉?”

邑有快役某,每数日不归,妻遂与里中无赖通。一日归,值少年自房中出,大疑,苦诘妻。妻不服。既于床头得少年遗物,妻窘无词,惟长跪哀乞。某怒甚,掷以绳,逼令自缢。妻请妆服而死,许之。妻乃入室理妆;某自酌以待之,呵叱频催。俄妻炫服出,含涕拜曰:“君果忍令奴死耶?”某盛气咄之,妻返走入房,方将结带,某掷盏呼曰:“咍,返矣!一顶绿头巾,或不能压人死耳。”遂为夫妇如初。此亦大绅者类也,一笑。

(《聊斋志异·佟客》)

〔注〕 ①跨蹇(jiǎn):骑着毛驴。②綦(qí):很。③拏(ná):纷乱。④坐名相索:点名要你。

2.对下列加点词的解释,不正确的一项是(3分)( )

A.每慷慨自负 每:常常 B.适自海外归耳 适:刚刚

C.烛之已杳 烛:点燃 D.不能死君父者 死:为……而死

3.下列全都表现董生“慷慨自负”的一组是(3分)( )

①董又奋然自许 ②君遨游四海,阅人綦多,曾见异人否? ③斩路侧小树以矜其利 ④邀佟至家,坚留信宿 ⑤董按膝雄谈⑥自酌以待之,呵叱频催

A.①③⑤ B.②③⑤ C.①③⑥ D.④⑤⑥

4.下列对文章有关内容的概括与分析,不正确的一项是(3分)( )

A.董生有狂妄自大的缺点,既不自知,也不知人。他对佟客夸耀自己的宝剑,其实其剑根本无法与佟客的利刃相提并论。

B.父亲被盗贼拷打时,董生壮气冲天,寻弓觅矢,要与盗贼拼命,最终还是佟客仗义出手,帮其退贼。

C.蒲松龄认为,想要尽忠孝,是人人都具有的基本品质,然而,最终是成为义士还是懦夫,往往就在一转念之间。

D.某快役发现妻子与人私通之事,十分恼怒,强令妻子自缢以谢罪,可最终还是没能狠下心肠。

5.把文中画横线的句子翻译成现代汉语。(6分)

(1)董乃自述所好,恨不得异人之传。(3分)

译文:________________________________________________________________________

(2)宜嘱后事于妻子,我启户为君警厮仆。(3分)

译文:________________________________________________________________________

(三)古代诗歌阅读(6分)

6.阅读下面这首词,回答问题。

蝶恋花

[宋]范成大

春涨一篙添水面。 芳草鹅儿,绿满微风岸。 画舫夷犹湾百转,横塘塔近依前远。江国多寒农事晚。 村北村南,谷雨才耕遍。 秀麦连冈桑叶贱,看看尝面收新茧。

〔注〕①夷犹:犹豫迟疑,这里是指船行驶迟缓。②塔近依前远:看着前方的塔仿佛近了,其实还很远。③尝面:揉下麦粒炒干研碎,取以尝新。

(1)词的上片描绘了一幅怎样的画面?(3分)

答:________________________________________________________________________

(2)词的下片写农事,抒发了词人怎样的思想感情?(3分)

答:________________________________________________________________________

二、文学名著、文化经典阅读(16分)

(一)文学名著阅读(10分)

7.下列各项中对作品故事情节的表述,不正确的两项是(5分)( )

A.押赴西伯利亚的犯人起程了。那正是天气炎热的七月,街上差不多一片荒凉,少数几个过路人在路边阴影里走着。有的犯人经不起烈日照晒,当场中暑倒毙了。涅赫柳多夫一路为犯人恶劣的处境四处奔波说情,他几乎成了犯人的袒护者。(《复活》)

B.住在圣母院里,爱斯梅拉达重新恢复了希望。一天早晨,她走到屋顶边上,从圆形圣约翰教堂的尖顶上方俯视广场。她看到了她朝思暮想的弗比斯正勒马驰过广场。于是她请求伽西莫多去找弗比斯,可是弗比斯用大马靴踹了伽西莫多一脚,策马远去。(《巴黎圣母院》)

C.吴荪甫是一个野心勃勃的资本家。事业上,他曾略施小技,就挤垮了朱吟秋的丝厂,又组建了“益中公司”,一口气吞并了8个日用品制造厂。在家里,他和妻子虽貌合神离,感情不好,但他与赵伯韬生活淫荡腐朽不同,他始终专注于他的事业。(《子夜》)

D.觉民对琴低声说:“琴妹,我们学堂明年暑假要招收女生。”听到这消息,琴十分高兴,说到时候要第一个去报名。回家后,她找母亲商量,母亲的态度虽然让她很失望,但当她读了易卜生的《娜拉》后,又充满了希望,并写信给许倩如,表明自己的决心。(《家》)

E.大个子拿侬长相难看,头脑简单,内心纯朴。她在葛朗台家里起早贪黑,什么都干,像一条忠心耿耿的狗那样保护主人的财产。有时葛朗台也会怜悯地瞧着她说:“可怜的拿侬!”但在拿侬看来,主人的这种怜悯也是拿侬的全部幸福。(《欧也妮·葛朗台》)

8.简答题。(选做一题) (5分)

(1)阅读下面的《红楼梦》选段,根据原著故事情节,回答问题。

宝钗笑道:“你不必忙,我替你做些如何?”袭人笑道:“当真的这样,就是我的福了。晚上我亲自送过去。”一句话未了,忽见一个老婆子忙忙走来,说道:“这是那里说起!金钏儿姑娘好好的投井死了。”

金钏为什么投井?宝钗建议王夫人如何处理此事?贾政又是如何得知此事的?请简述。

(2)阅读下面的《三国演义》选段,根据原著故事情节,回答问题。

卓疾既愈,入朝议事。布执戟相随,见卓与献帝共谈,便乘间提戟出内门,上马径投相府来;系马府前,提戟入后堂。

吕布到“后堂”找谁?随后赶来的董卓是如何对待吕布的?后来吕布又是如何对待董卓的?请简述。

答第()题:________________________________________________________________________

(二)文化经典阅读(6分)

9.阅读下面的《论语》选段,回答问题。

①子曰:“夫仁者,己欲立而立人,己欲达而达人。能近取譬,可谓仁之方也已。”

(《论语·雍也》)

②有子曰:“其为人也孝弟,而好犯上者,鲜矣;不好犯上,而好作乱者,未之有也。君子务本,本立而道生。孝弟也者,其为仁之本与!”(《论语·学而》)

(1)请简要概括孔子和有子对“仁”的看法。(2分)

答:________________________________________________________________________

(2)结合有子的言论谈谈儒家是如何看待“孝弟”的。(4分)

答:________________________________________________________________________

三、现代文阅读(24分)

(一)论述类文本阅读(9分)

阅读下面的文字,完成10~12题。

构思的诞生几乎没有雷同的。要解释构思的产生,最好的办法莫过于借重比喻。一些极其复杂的事情,若用比喻来加以解释,往往能收拨云见天的效果。有一次,人们问天文学家金斯我们的地球有多大年纪了。“你们想象一下,”金斯回答说,“有一座巍峨的大山,比方说吧,高加索的厄尔布鲁山。你们再设想一下,有一只小麻雀在山顶上无忧无虑地跳来跳去,啄着这座山。这只麻雀把厄尔布鲁山啄光需要多长的时间,地球就已存在多长时间了。”

至于有助于领会构思是怎样产生的比喻,就简单多了。构思好比闪电。电日日夜夜在地面的上空积累,一旦空气中的电达到了饱和状态,一朵朵洁白的积云就会变成阴森的积雨云,于是从积雨云的稠密的带电的水汽中,便会爆发第一道火花——闪电。几乎紧接在闪电之后,一场暴雨便会倾泻而下。

构思就如闪电,产生于人的满含思想、感情和记忆的印痕的意识之中。所有这一切是逐步地、慢慢地积累的,等到电位差增大到一定程度时,就必然导致放电的现象。这时,意识这个被整个儿压缩的,还多少有点混乱的世界,便会诞生闪电,也就是说诞生构思。构思之得以产生同闪电之得以产生一样,往往只需要一个极为轻微的推动力。可能是一次偶然的相逢,可能是印在心中的一句话,可能是一场梦,可能是远方的呼声,也可能是水滴映射出来的阳光或者是轮船的汽笛声。

列夫·托尔斯泰看见了一株断掉的牛蒡,便爆发了闪电:产生了描绘哈吉·穆拉特的那部令人惊叹的中篇小说的构思。然而,托尔斯泰要是从未去过高加索,也没有听说过哈吉·穆拉特的事迹,那么牛蒡就无从触发他这个构思。唯其因为托尔斯泰心里对这个题材已有所酝酿,所以牛蒡才引起了他必要的联想。

如果说闪电好比是构思的话,那么豪雨就是构思的体现。体现为形象与语言的和谐的洪流,体现为书。

但是跟明亮的闪电不同,构思最初往往是模糊不清的。构思只可能逐步成熟,逐步吸引作家的才智和心灵,逐步趋于周密,趋于复杂化。但是所谓“构思酝酿”的过程却全然不像某些幼稚的人所想象的那种样子。这绝不表现为作家抱住脑袋坐在那里向壁虚构,或者独自一人像个狂人似的口中念念有词地踱来踱去。构思的形成和充实是个不间断的过程,每日每时,随时随地,在一切偶然的事件中,在劳动中,在我们“转瞬即逝的生命”的喜怒哀乐中,不停地进行着的。要想使构思成熟,作家决不可脱离生活,一味地去“苦思冥想”。相反,只有始终不渝地接触现实,构思才得以绽出鲜花。

(摘自康·帕乌斯托夫斯基《金蔷薇》,有删减)

10.下列对原文概括和分析正确的一项是(3分)( )

A.文章第一段引用金斯的故事,是为了告诉我们构思是如何产生的。

B.“几乎紧接在闪电之后,一场暴雨便会倾泻而下。”文中的这句话讲的是构思的产生如同暴雨的产生。

C.文章认为构思之得以产生往往只需要一个偶然的因素。

D.“当时,这部自由的小说的远景,我虽然透过魔法的水晶,却仍然没有看得分明。”这句话很好地说明了文章最后一段谈的构思与明亮闪电的不同。

11.文章最后一段,画线的“那种样子”指的是什么样子?作者认为构思的过程应该像哪种样子?(3分)

答:________________________________________________________________________

12.请概括本文作者所认为的构思产生的过程。(3分)

答:________________________________________________________________________

(二)文学类、实用类文本阅读(选考)(15分)

本题为选做题,请从甲乙两类文本中选定一类作答,不得再做另一类文本的题目。

(甲)文学类文本阅读(15分)

阅读下面的文字,完成13~15题。

沈从文

(1)有个小小的城镇,有一条寂寞的长街。

(2)那里住下许多人家,却没有一个成年的男子。因为那里出了一个土匪,所有男子便都被人带到一个很远很远的地方去,永远不再回来了。他们是五个十个用绳子编成一连,背后一个人用白木梃子敲打他们的腿,赶到别处去作军队上搬运军火的夫子的。他们为了“国家”应当忘了“妻子”。

(3)大清早,各个人家从梦里醒转来了。各个人家开了门,各个人家的门里,皆飞出一群鸡,跑出一些小猪,随后男女小孩子出来站在门槛上撒尿,或蹲到门前撒尿,随后便是一个妇人,提了小小的木桶,到街市尽头去提水。有狗的人家,狗皆跟着主人身前身后摇着尾巴,也时时刻刻照规矩在人家墙基上跷起一只腿撒尿,又赶忙追到主人前面去。这长街早上并不寂寞。

(4)当白日照到这长街时,这一条街静静的像在午睡,头发干枯脸儿瘦弱的孩子们,皆蹲到土地上或伏在母亲身边睡着了。作母亲的全按照一个地方的风气,当街坐下,织男子们束腰用的板带过日子。用小小的木制手机,固定在房角一柱上,伸出憔悴的手来,一面便捷地退着粗粗的棉线,一面用一个棕叶刷子为孩子们拂着蚊蚋。带子成了,便用剪子修理那些边沿,等候每五天来一次的行贩,照行贩所定的价钱,把已成的带子收去。

(5)许多人家门对着门,白日里,日头的影子正正的照到街心不动时,街上半天还无一个人过身。每一个低低的屋檐下人家里的妇人,各低下头来赶着自己的工作,做倦了,抬起头来,用疲倦忧愁的眼睛,张望到对街的一个铺子,或见到一条悬挂到屋檐下的带样,换了新的一条,便仿佛奇异的神气,轻轻的叹着气,用兽骨板击打自己的下颌,①因为她一定想起一些事情,记忆到由另一个大城里来的收货人的买卖了。她一定还想到另外一些事情。

(6)街上也常常有穿了朱红绸子大裤过身的女人,脸上抹胭脂擦粉,小小的髻子,光光的头发,都说明这是一个新娘子。到这时,小孩子便大声喊着看新娘子,大家完全把工作放下,站到门前望着,望到不见这新娘子的背影时才重重的换了一次呼吸,回到自己的工作凳子上去。长街在日里也仍然不寂寞。

(7)街上到黄昏时节,常常有妇人手中拿了小小的簸箩,放了一些米,一个蛋,低低的喊出一个人的名字,慢慢的从街的一端走到另一端去。这是为不让小孩子夜哭发热,使他在家中安静的一种方法,这方法,同时也就娱乐到一切坐到门边的小孩子。长街上这时节也不寂寞的。

(8)黄昏里,街上各处飞着小小的蝙蝠。望到天上的云,同归巢还家的老鸹,背了小孩子们到门前站定了的女人们,一面摇动背上的孩子,一面总轻轻的唱着忧郁凄凉的歌,娱悦到心上的寂寞。

(9)“爸爸晚上回来了,回来了,因为老鸹一到晚上也回来了!”

(10)远处山上全紫了,土城擂鼓起更了,低低的屋里,有小小油灯的光,为画出屋中的一切轮廓,听到筷子的声音,听到碗盏相磕的声音……但忽然间小孩子又哇的哭了。

(11)爸爸没有回来。有些爸爸早已不在这世界上了,但并没有信来。有些临死时还忘不了家中的一切,便托便人带了信回来。得到信息哭了一整天的妇人,到晚上,便把纸钱放在门前焚烧,红红的火光照到街上下人家的屋檐,照到各个人家的大门。②见到这火光的孩子们,也照例十分欢喜。长街这时节也并不寂寞的。

(12)阴雨天的夜里,天上漆黑,街头无一个街灯,狼在土城外山嘴上嚎着,用鼻子贴近地面,如一个人的哭泣,地面仿佛浮动在这奇怪的声音里。什么人家的孩子在梦里醒来,吓哭了,母亲便说:“莫哭,狼来了,谁哭谁就被狼吃掉。”

(13)卧在土城上高处木棚里老而残废的人,打着梆子。这里的人不须明白一个夜里有多少更次,且不必明白半夜里醒来是什么时候。那梆子声音,只是告给长街上人家,狼已爬进土城到长街,要他们小心一点门户。

(14)一到阴雨的夜里,这长街更不寂寞,因为狼的争斗,使全街热闹了许多。冬天若半夜里落了雪,则早早的起身的人,开了门,便可看到狼的脚迹,同糍粑一样印在雪里。

(选自《品中国散文》上海科学技术文献出版社,有删改)

13.下面对作品的概括与分析,不正确的两项是(5分)( )

A.文章第(2)段紧承段首,进一步写出湘西小城镇长街的寂寞,点明了长街寂寞的社会原因,营造了一种淡淡的哀戚氛围。

B.第(5)段画线①处是心理描写,反复写“一定想起(到)……事情”,起强调作用,说明了妇女们对男人的思念牵挂之切。

C.第(11)段画线②处以哀写乐,这里写不谙世事、无知孩童的欢天喜地、无忧无虑,反衬的是妇人们失去男人的无限悲哀。

D.文章写“白日里街上走过新媳妇,阴雨的夜里狼的光顾与争斗”用以衬托长街妇人失去丈夫后的孤苦、无依和生活的乐观。

E.写法上作者有意模糊散文、小说的体式,文章有清晰的叙事线索,自然、朴实、真切的日常生活画面有一种异样感人的魅力。

14.长街的黄昏呈现了哪些“不寂寞”的生活图景?结合文章内容,用简洁的语言加以概括。(4分)

答:________________________________________________________________________

15.沈从文作品独特鲜明的个人风格历来为后人所称道。请依据文章内容,从两个不同的角度说说你的见解。(6分)

答:________________________________________________________________________

(乙)实用类文本阅读(15分)

阅读下面的文字,回答13~15题。

变态不变态,大脑来表态

赵涵漠

悉德·皮尔森是个生长在美国东海岸的年轻人。他肌肉发达,拥有一头金色的头发、长长的睫毛和碧绿色的眼睛。他还懂得如何享受生活。一句话,皮尔森迷人极了。

这也许能够解释,为什么乔伊斯会愿意与这个只有一面之缘的男人约会。她搬到这里还不到一个星期,就和他在海边浪漫相约。两人用手机自拍,留下甜蜜的笑容。

乔伊斯没有意识到,危险即将来临。

这个只有19岁的姑娘被皮尔森和他的同伙强暴、殴打。那两个家伙甚至开船到150海里外,将乔伊斯抛进海中,等着她被鲨鱼吃掉。

在美国哥伦比亚广播公司出品的电视剧《犯罪心理》中,以上故事只是诸多心理变态者案例的一则。这些犯罪者和很多人的既定印象不同,不再是满脸横肉或者脏话连篇。恰恰相反,根据神经科学家的研究,心理变态者通常都很有魅力。

皮尔森也是如此。即便在警察局的审讯室里,他一点也不惊慌,脸上总是带着迷人的微笑。他是学校里的风云人物,开着名牌跑车上学,还是曲棍球甲级运动员。外人眼里,他唯一的“恶行”不过是让女朋友替考。

他在警察局却留有一沓厚厚的档案,包括破坏公物、入室偷窃、烧毁房屋。如今,他试着让鲨鱼咬碎一个正在与他约会的女孩的脑袋。

这些反差是科学家研究心理变态者的一大障碍。他们的行为难以捉摸、不负责任,有时甚至具有暴力倾向,但并没有典型的精神疾病症状:没有幻觉、幻听,不会感到迷惑、焦虑,在社交场合也不会显得笨拙腼腆。他们的智商通常还高于平均水平。这让人们迷惑:他们到底是疯了,还是有点坏?

上个世纪90年代,加拿大心理学家罗伯特·哈雷通过脑电图发现,心理变态者注意不到言语中情感的细微差别,也无法在一堆单词中分辨出褒义词和贬义词。2002年,一份来自美国国立精神卫生研究所的研究也表明,心理变态者很难通过别人的声音来感知情绪,也不能辨别哪些面部表情带有恐惧情绪。

这些研究证明,或许心理变态者并不是自私、冷血那么简单,而是具有严重的生理缺陷,陷入没有情感的世界。

谜底或许藏在大脑深处。科学家们怀疑,他们大脑内牵涉到对风险、奖赏与惩罚的敏感性的眶额皮层受到了损坏。还有产生害怕等情感的杏仁核,在动物实验中,杏仁核受损的猴子会径直向人走来全不害怕。这与心理变态者那种“毫无畏惧”的特质十分相似。

或许大脑的边缘系统也与此有关。这个系统负责情绪加工、目标搜寻、动机产生及自我控制。在磁共振成像仪下,心理变态者的旁边缘组织明显很薄,“就像一块软弱无力的肌肉”。

大脑功能的异常导致心理变态者体会不到自己和他人的感受。想象一下,如果一个人没有忧伤,从不后悔和自卑,对任何事、任何人都漠不关心,他会是什么样子?

《犯罪心理》中出现过一个13岁的连环杀手。他总是打扮成一个无家可归的可怜孩子。如果有一对善良的父母收留他,那将是噩梦的开始。他会在深夜将全家老小绑起来,再一刀刀、活生生地捅死他们。

对他来说,这一切正常极了,“难道你不好奇皮肤下面有什么吗?”

别以为这种人只出现在每周播出一集的电视剧里,心理变态者很有可能就隐藏在人群之中。根据美国的一项调查,这个国家每两百个人中就会有一到两个心理变态者。除了关闭在监狱里的囚犯,美国至少还有25万个心理变态者在其他角落里自由地生活。

相比之下,这个国家投入在抑郁症方面的研究资金高达数十亿美元。但寻找心理变态疗法得到的资助,连100万美元都不到。

这倒不难理解。目前,心理变态几乎被视为不可治疗的疾病。一些研究表明,他们接受治疗后的表现,甚至还不如不接受治疗。

科学家们只有寄希望于人们不再对心理变态者抱有偏见:“只有认识到他们不是魔鬼,只是由于情感缺陷而导致行为怪异的病人时,我们才会拥有更加安全的未来。”

(选自2011年1月5日《中国青年报》)

13.下面对文章的概括和分析,不正确的两项是(5分)( )

A.电视剧《犯罪心理》中,介绍了诸多心理变态者案例,“皮尔森想让鲨鱼咬碎乔伊斯脑袋”就是其中的一个故事。

B.心理变态者的行为难以捉摸、不负责任,有时甚至具有暴力倾向,但并没有典型的精神疾病症状。

C.心理学家发现,注意不到言语中情感的细微差别,不能辨别恐惧情绪是引发心理变态的重要原因。

D.研究表明,人脑的杏仁核一旦受损,对风险、奖赏与惩罚的敏感性就会受到影响,进而产生“毫无畏惧”的心理。

E.本文主要介绍关于“心理变态”的最新研究,但能寓科学知识于生动形象的言语,读来毫无生涩之感。

14.文章开头用较大的篇幅讲述了“皮尔森”的故事,有何作用?(4分)

答:________________________________________________________________________

15.对于“心理变态”这一社会问题,我们应当采取的积极措施有哪些?请结合文本给予概括。(6分)

答:________________________________________________________________________

四、语言文字运用(13分)

16.阅读下面一段文字,完成问题。(2分)

随着生活节奏加快,网络、电视以及各种娱乐方式不断发展,大大占用了人们的闲暇时间,人们的读书时间越来越少。此种现象带动了“快餐文化”的盛行,很多优秀作者没有坚定的立场,随着社会潮流走,向(gōnglì)_________化的社会投降:或书籍粗制滥造,或标题夺人眼球,或内容故作深奥,实质没有多少营养,导致61.9%的人觉得当前值得购买的新书越来越少了。

(1)请将原文画波浪线的内容改用一个成语表达。(1分)

答:________________________________________________________________________

(2)根据拼音提示,结合语境,在横线上填写正确的词语。(1分)

答:________________________________________________________________________

17.仔细观察下面的图,完成后面的题目。(4分)

这幅漫画揭示的道理常常被人忽视,或者常常让人感到难以理解,不可思议。请根据你的理解,写出这个道理。

答:________________________________________________________________________

18.阅读下面的材料,回答问题。(7分)

截至2010年10月1日,日本国内65岁以上的老年人达2958万人,占总人口的23.1%,随着老龄化社会的到来,最近在日本兴起了一种“代孝业”,只要子女肯花钱,就可以请代孝公司上门为老人提供服务,陪吃、陪聊、陪看病,一应俱全,还可以代理回乡探亲、扫墓等事宜。

如果在我国也流行请人“代孝”,你赞同吗?请简要阐述。(要求:表达简明连贯,言之成理,150字左右)

答:________________________________________________________________________

________________________________________________________________________

________________________________________________________________________

五、写作(70分)

19.阅读下面的材料,根据要求写一篇不少于800字的议论文或记叙文。

在近日上海举行的“玩的教育”研讨会上,教育专家和一线教育工作者强烈呼吁:尊重孩子的玩权,应当把“玩”作为一门科学,让孩子在“玩”中幸福成长。但在中国人的主流观念中,“业精于勤荒于嬉”,对玩和游戏是贬斥的。

对此,你有怎样的体验和认识?

要求:(1)必须符合文体要求;(2)角度自选;(3)立意自定;(4)题目自拟;(5)不得抄袭,不得套作。

参考答案 ◄ 测评手册

参 考 答 案

阶段评估检测(一)

1.(1)宫阙万间都做了土 (2)舞榭歌台 (3)沙鸥翔集 (4)匪我愆期 (5)枯松倒挂倚绝壁

(6)多于在庾之粟粒(一处1分,错、漏、添字的,该处不给分)

2.D [解析] 善:教育好,名词活用为动词。

3.B [解析] ①描写的是道士所建的茅庐;③是整个武夷山九曲溪的环境。

4.C [解析] 武夷精舍的规模仅有邻近道庐的一半。

5.(1)大概山中所有的乐趣,都被朱元晦所私人拥有,对比于他,我常常觉得很惭愧。(大意1分,“盖、悉、为、私、每”等词语译错一处扣半分,扣完2分为止)

(2)元晦既然已经明白这些道理,就尝试着用这些道理教导那些来向他求学的人。(大意1分,“既、有以、识、夫”每处译错扣半分,扣完2分为止)

[参考译文]

武夷山位于福建省的正北部,山势雄伟幽深曲折,自汉代以来,武夷山的名字就出现在祭祀的大事中。福建的其他山峰,都是后来才见于记载的。武夷山最大的一座山峰,上大下小,高高地耸立,就像一个巨人戴着帽子。沿着缝隙中的石阶可以攀登,世间传说逃避秦朝乱世而登入仙界的人就是在这里留下尸骨的。有一道溪水从山峰下流出,悬崖绝壁高大、险峻,都在几十丈深,悬崖两侧巨石林立,宏伟壮观奇特秀丽。喜好游览的人也不能在一天中走完,于是就躺在小舟上沿着溪水逆流而上,环顾两岸。到达地面平坦、景物环绕集中的地方,一定要为其停船上岸,拄着拐杖走走停停(流连忘返),不忍离去。山中本来有许多猴子,鸟大多是白鹇、鹧鸪,听到人的声音,有的就磔磔地怪叫栖止于山崖上,然后无拘无束地慢悠悠地飞走而没有任何害怕的意思。水流潺潺有声,草木四季都有花开。有位道士在溪水的第六个转折处造了一间房舍,来供游人吃东西休息。游人常常喝酒还没喝到一半,天色已经接近黄昏,不能再停留了。

我的朋友朱元晦住在五夫里,五夫里在距离武夷山不到三十里的地方,近得好像是武夷山外园,有空的时候就到武夷山游玩。朱元晦和他的门生弟子带着书前来诵读,选取的是《诗经》和《楚辞》,(元晦和他的门生弟子)吟哦高歌,饮酒长啸,一定要停留数日才离开。大概山中所有的乐趣,都被朱元晦所私人拥有,对比于他,我常常觉得很惭愧。

淳熙十年,元晦辞去江东使节一职,享受祠官俸禄,又说:“如今我管理此地,终于能尽享山中之乐了。”因为他游览的次数多了,就在溪水的第五个转折处,背靠巨石的地方,规划精舍,采取道庐一半大小的规模。锄去茅草,只得到将近几亩大的地方。这里环境清幽,奇石佳木拱立于屋子的周围,互相映衬。元晦叫弟子们准备簸箕、铁锹,采集青瓦,栽种树木,弟子们相继完成了这些工作。元晦亲自规划了精舍的布局,中间的做厅堂,旁边的做书房,高的做亭,间架多的做内室。先生讲学、门人弟子学习课业、弹琴唱歌、饮酒赋诗,没有不在这里举行的。我听到这件事,恍然像刚从睡梦中醒来,醒来后,还隐隐约约记得这个地方的美景,而且元晦对我说:“请你记述一下吧!”

元晦,一介儒生,当时正用他的学问在这一带讲学,教育门生弟子。不像那些奇人隐士,藏于深山,练气功,吃灵芝,仰慕道家一类的人。秦汉以来,儒家的学说得不到阐明已经很久了。孔子所说的有志于道,指的是什么呢?孔子,圣人啊,他慢步或快走都有一定的道理。至于他登上泰山之顶,在舞雩台下吟诗唱歌,没有什么他不游历的。他的胸中自有广阔的天地,因而当时有弟子(曾皙)弹奏琴瑟,铿地一声停下,咏唱“春服既成”,竟然只有他被圣人所赞同。古代的君子对于游玩休息,哪里是拘拘束束的呢?

元晦既然已经明白这些道理,就尝试着用这些道理教导那些来向他求学的人,并和他们一起在精舍饮酒畅谈,使来人(求学者)都能享受这仙人般的山水乐趣,那又如何呢?

淳熙八年,颍川韩元吉记载。

6.(1)[答案]①首联由远及近描写所“望”之景;②中间两联因景生情,抒写“望”之感慨;③尾联点明“望”之方式和地点,以及由“望”引发的忧虑。(每点1分)

[解析]首联两句写野望时所见西山和锦江;颔联和颈联是在首联的景象基础上“望”景生情;尾联点出“野望”的方式和深沉的忧虑。

(2)[答案]①“三城戍”“海内风尘”表达忧国之情;②“诸弟隔”表达对亲人的思念之情;③“一身遥”表达漂泊孤寂之苦;④“迟暮”“多病”抒发年老多病的感伤与无奈;⑤“未……答圣朝”表达未能报国之憾。(任答三点即可)

[解析]本诗的情感抒发主要集中在中间两联。三四句由战乱推出怀念诸弟,自伤流落的情思。诗人自身孤单寂寞,怀念家国,不禁“涕泪”横流。颈联抒发自己年迈多病的悲伤以及无力报答圣朝的愧疚之情。

[诗歌赏析]

这首诗作于上元二年(761)成都草堂。诗以“野望”为题 ,是诗人跃马出郊时感伤时局、怀念诸弟的自我写照。

首两句写野望时所见的西山和锦江。“西山”在成都西,主峰终年积雪,因此以“白雪”形容。“三城”,松、维、保三州(在今四川松潘、理县一带),此时驻军严防吐蕃入侵,是蜀地要镇。“南浦”,南郊外水滨。“清江”,锦江。“万里桥”,在成都城南。

中间四句是野望时触发的有关国家和个人的感怀。三四句由战乱推出怀念诸弟,自伤流落的情思。“风尘”指“安史之乱”导致的连年战火。杜甫四弟:颖、观、丰、占。只杜占随他入蜀,其他三弟都散居各地。此时“一身遥”客西蜀,如在天之一涯。诗人怀念家国,不禁“涕泪”横流,真情实感尽皆吐露不由人不感动。五六句又由“天涯”“一身”引出残年“多病”,“未”贡微力,无补“圣朝”的惭愧。“供”,付托。“涓埃”,滴水、微尘,指毫末之微。杜甫时年五十,因此说已入“迟暮”之年。他叹息说:我只有将暮年付诸给“多病”之身,但“未有”丝毫贡献,报答“圣朝”,是很感惭愧的。杜甫虽流落西蜀,而报效李唐王朝之心却始终未改,足见他的爱国意识是多么强烈。中间四句,由于连用对偶而将诗人的家国之忧、身世之感,特别是报效李唐王朝之心,艺术地概括了出来。

七八句最后点出“野望”的方式和深沉的忧虑。“人事”,人世间的事。由于当时西山三城列兵防戍,蜀地百姓赋役负担沉重,杜甫深为民不堪命而对世事产生“日”转“萧条”的隐忧。这是结句用意所在。

杜甫“跨马出郊”,“极目”四“望”,原本为了排遣郁闷。但爱国爱民的感情,却驱迫他由“望”到的自然景观引出对国家大事、弟兄离别和个人经历的种种反思。一时间,报效国家、怀念骨肉和伤感疾病等等思想感情,集结心头。尤其为“迟暮”“多病”发愁,为“涓埃”未“答”抱愧。

此诗前三联写野望时思想感情的变化过程,即由向外观察转为向内审视。尾联才指出由外向到内向的原因。在艺术结构上,颇有控纵自如之妙。

7.[答案] BE(答对一项给3分,两项给5分)

[解析]B项,“惊慌失措”错,也没有打算回老家;丝厂总管应是莫干丞。E项,“美丽的侯爵小姐”错,侯爵小姐长得很丑,查理为了高攀才答应婚事;“哄骗善良的欧也妮偿还了自己父亲的债务”错,查理对欧也妮偿还债务的举动事先并不知情。

8.(1)司棋在抄检中被查出男人的衣物和书信。(2分)迎春不敢向太太求情,只能泪别司棋。(1分)司棋回家之后,因求嫁潘又安不成而自尽,潘又安闻讯也自杀殉情。(2分)

(2)徐庶。(1分)因曹操诈作徐母家书,徐庶被曹操骗到曹营。(2分)徐庶接受庞统建议,散布西凉马腾造反的消息,然后主动请求带兵阻击马腾,从而远离赤壁大战。(2分)

9.[答案] (1)不讲信用,不守礼制,不懂道德,(2分)不知羞耻,这些都是无耻的。(1分)

(2)孔子认为,一个人的行为,要用“德”“礼”来规范,才能远离“耻”。(1分)孟子认为人要有羞耻心,(1分)在现代生活中,我们不要让物质的诱惑埋没了我们的羞耻心。(1分)

[解析]第(1)题考查对材料内容的概括。原文中直接提及“无耻”的只有第三段,对应语句是“无耻之耻,无耻矣”,前两段虽然没有直接讲“无耻”,但却提到了“远耻”和“有耻”,据此显然也可推断出“无耻”的具体行为。第(2)题考查对作者观点的理解分析,原文较短,语意集中,考生只要将原文语句的含义与“耻”联系在一起,表达合理即可。值得注意的是,第一段是“有子”的话,不是孔子的观点,因此,第一则材料的内容不能掺杂到答案之中。

[参考译文]

①有子说:“所守的诺言如果符合于义,那么所说的话就能够兑现。恭敬、谨慎的态度符合于礼,才能远离耻辱。”

②孔子说:“……用道德教化引导百姓,用礼制去教育百姓,百姓就不仅会有羞耻之心,还会遵守规矩。”

③孟子说:“人不能没有羞耻之心,把没有羞耻之心当成羞耻,那就不会有耻辱了。”

10.B [解析] 面对命运的幽默比面对各种偶像的幽默更伟大,是因为它直接与天神开玩笑。

11.[答案]幽默是对生活的一种哲学式态度,(1分)它要求我们与生活保持一定距离,暂时以局外人的眼光来发现和揶揄生活中的缺陷。(1分)

[解析]本题考查对文章内容的概括。考生首先应根据题干找到对应的原文第④段,然后再在画线句的上下文寻找相关的原文语句,并进行一定的归纳和转化,最后分点答题。

12.[答案] 将“幽默”与“滑稽”对比,阐明“幽默是智慧的闪光”;(2分)将“幽默”与“嘲讽”对比,阐明幽默者“有一种超脱的态度”。(2 分)

[解析]本题考查对文章结构思路的理解。⑤⑥段谈“滑稽”与“嘲讽”,其目的在于说明“幽默”,因为本文的论述中心词是“幽默”,本题的难点在于正确理解“滑稽”“嘲讽”与“幽默”的内在联系。由于作者对“滑稽”“嘲讽”持有批判态度,据此可归纳出作者运用了“对比”,而“对比”的目的,则在于突出“幽默”的优点,这些优点,都可以通过筛选原文得到。

(甲)

13.[答案] BD(答对一项给3分,两项给5分)

[解析]B.“都是因为他死要面子,这为他后来的转变做了铺垫”错,死要面子是有的人的看法,而非九度的实际特征,而九度的生活态度始终未变,所以涉及不到“转变”这一说法。D.不是倒叙,是插叙。

14.[答案]九度是一个贫困却不潦倒,屡遭不幸却能以一种热情、坚强、乐观的心态,一种有尊严的生活方式,去面对人生的生活强者的形象。①坚强、乐观:他人生多舛,上大学时,生活贫困,工作后受牵连下岗,后来爱人查出了淋巴癌,做生意失败,又被人欺骗,但他丝毫没有消沉之气,而是坚强面对,勇于承担,并且总是能看到生活中的温暖,热情地对待生活。②自尊自立:拒绝接受贫困贷款,在校友录上的留言向来是报喜不报忧,拒绝接受同学们的帮助,在聚会上拿不出钱却不肯白占大家的便宜,一定要以别的方式补偿回来,这都表现了他的自尊自立。(4分。能归纳出两点并结合文本分析即可)

[解析]本题考查对小说人物形象的概括。分析形象首先要指出形象的身份属性和形象意义,然后再归纳其典型性格。由于题目要求结合文章内容答题,因此,考生答题时应注意在概括其典型性格之后补充对应细节内容。

15.[答案]示例一:认为必不可少。从小说的主题角度看,小说这一结尾画龙点睛,使读者对作品主题的认识达到一定的深度。从小说的情节安排角度看,这一结尾与前文九度的表现和题目相照应,使小说情节更加完整。从人物形象的塑造角度看,这一结尾点明了人物的形象特点,使人物的形象更加鲜明。

示例二:认为画蛇添足。从主题的角度看,如果没有这个结尾,小说会给人更大的思考空间,这样的结尾限制了主题的深度和广度。从小说的形象塑造看,人物的性格已经在前文的细节描写中得到了充分表现,没有必要用议论性的语言点明其性格特征。从小说的创作手法看,在小说中加入议论性的评论,会影响小说以人物形象表现主题的特征,混淆了文体。(6分。观点鲜明,理由条理清晰,能自圆其说即可)

[解析]本题考查对小说文本特征和艺术手法的鉴赏评价,小说的某一情节是否应当存在,主要取决于三个因素:一是主题需要,二是表现人物的需要,三是前后情节的照应需要。考生答题时,应注意结合小说具体内容,表达应尽量委婉合理。

(乙)

13.[答案] BE(答对一项给3分,两项给5分)

[解析] A.“是因为它们都是非常珍贵的艺术品”错,是因为它们一样能给人带来美感。C.体现了钱学森粗犷而任性的一面。D.“完美无瑕”错,作者也写了钱学森不近人情等特点。

14.[答案] ①严谨认真、永不满足的治学精神; ②实事求是的态度; ③不容置疑的才能;④勤奋的特点; ⑤淡泊名利的胸怀与情操;⑥粗犷而任性的个性。(4分,写出四点即可)

[解析]本题考查对传主形象的概括,题目难度不大。考生应根据原文顺序,从中筛选出如“唯美”“老实”“勤奋”“胸怀”“粗犷而任性”等关键词,再结合传主事迹适当充实,分点作答。

15.[答案]①学生时代的学习成绩并不能说明一个人将来能否成才;②伟人也是人,不必为尊者讳,更不必把伟人神化;③诚实是一个人最可贵的品质。(可任选一点进行分析,6分)

[解析]本题考查对语句含义的理解和对文章创作意图的评价。画线句之所以“语意丰富”,显然不是因为其表达上的隐含,而是因为这句话的具体内容让作者佩服不已,因此,考生答题时不能仅限于画线部分的句意,而应结合原文所引的钱学森的话语内容,答出钱学森的原话所包含的人生道理,然后再对这一道理进行评价。

16.[答案] (1)示例:鲜为人知 (2)诠(3)示例:这些文章,既给人以科学的教益,也给人以美的享受(也让人获得美的享受)。

[解析]第(1)(2)题涉及成语积累和拼音认读,主要考查考生的基本功。第(3)题考查病句修改,和原句主语“文章”搭配的谓语有两个,一个是“给人”,一个是“获得”;且与“获得”存在主客颠倒的搭配问题。“获得美的享受”的主语应是“人”,因此应将“获得”修改为“给人以”,或在“获得”前加“让人”。

17.[答案](恶意软件是)在未明确提示用户或未经用户许可的情况下,在用户计算机或其他终端上强行安装运行、侵害用户合法权益的软件。(3分)

[解析]本题考查下定义,属于句式变换的考点。题干提供的其实是多个短单句,要求考生做的是将这些短单句组成一个长单句。根据下定义的规则,考生应先将全句的主谓宾确定下来,即“恶意软件是……的软件”,然后再筛选能表现“恶意软件”本质特点的关键词,组成句子的定语部分。

18. [答案] 示例:硕士竞聘卖猪肉岗位,这是对国家人力资源的浪费。 十年寒窗,却终

于学非所用。虽说学历不代表实力,但研究生卖猪肉可以体现什么学术价值,知识力量?卖猪肉这活儿也可能需要统计学、经济学甚至会计学,但不至于需要硕士研究生吧。倘若一个妈妈指着“卖猪肉的”告诉孩子他是硕士时,孩子又会作何感想?我觉得正确的就业心态是认清自己,学以致用。

[解析]本题考查语言的综合运用能力,尤其是语言表达的简明、连贯。题干材料是一则简明的新闻,主要内容包括两个方面,一是新闻标题所说的现象,即“硕士竞聘卖猪肉工作”;二是材料作者对此事的评价,即“天才公司的招聘广告火了”。原则上说,考生可以针对以上两个方面内容,任选其一进行评说,但从操作的角度看,考生选择第一方面内容进行评说是比较容易组织答案的。观点可以是赞同“硕士竞聘卖猪肉工作”,也可以反对。理由可从以下三个角度考虑,一是考虑这样做能否实现个人价值,二是考虑这样做的社会价值,三是考虑这样做的社会影响。

19.略。

阶段评估检测(二)

1.(1)故国不堪回首月明中 (2)留取丹心照汗青 (3)此情可待成追忆 (4)惟吾德馨 (5)薄暮冥冥(6)化作春泥更护花(一处1分,错、漏、添字的,该处不给分)

2.C [解析] C项“东”与“并河”之间用“而”联结,表明两个词语的词性相同,因此“东”应作动词,意为“向东流”。

3.C [解析]本题题干的筛选条件有三个,一是“六井”,二是“修治”,三是“措施”。根据“六井”这一筛选条件无法排除句子。根据“修治”这一条件,可排除①句,此句讲的是开凿,而非修治;根据“措施”这一条件,可排除③⑥句,这两句都是说修治后带来的好处,而非“措施”。

4.B [解析] B项“开凿……终于刺史白乐天”有误,白乐天做的是疏浚工作,而非开凿工作。

5.(1)太守陈述古刚到这里,询问百姓担忧的事情。(意思对、语言通顺1分,“始”“病”各1分)

(2)这是天下的通病,难道只是对待水这样吗?(意思对、语言通顺1分,“患”“岂”各1分)

[参考译文]

钱塘江的潮水离开钱塘向东冲向西陵,潮水从远处涌来。它使低洼潮湿的盐碱地变成了种植桑麻的地方,时间长了又发展成为人口聚居的城镇,今天州县的平坦陆地,都是钱塘江的故地。这里的水质又苦又恶臭,只有靠山凿井,才能得到甘甜的泉水,可是泉水供给的范围又不够广。唐朝的宰相李长源开始开凿六井,引西湖的水来供给百姓饮用。后来刺史白乐天治理西湖疏浚六井,并且在西湖边上刻碑题字,直到如今人们还依靠这几眼井水生活。当初李长源开凿六井,其中最大的在清湖,叫做相国井,相国井的西边是西井,西井偏西而向北的叫做金牛池,又向北而偏西靠近钱塘城的分别是方井和白龟池,又向北而偏东到钱塘县治南面的叫做小方井。不过,金牛池废弃已经很久了。嘉祐年间,太守沈文通又在六井的南面,越过河流往东,到美俗坊的地方开凿了南井。水流出了涌金门,汇入湖中向北流去,上有三道水闸,水流注入石砌的水沟,穿城向东流去,南井、相国井、方井都从这里流出。至于西井,则是相国井的支流。而白龟池、小方井,都建造了暗沟从湖底流过,不用水闸。这就是六井的大致情况。

熙宁五年秋天,太守陈述古刚到这里,询问百姓担忧的事情。百姓都说:“六井不修治好,老百姓就没有水喝了。南井沟渠低下而井高,水在地底下流淌,大抵通常供应不过来。”陈公说:“咳,说得太过了,有我在这儿,能让百姓没有水喝吗!”于是命令和尚仲文、子圭办理此事。于是打开沟渠更换井壁,修补好了一些漏洞,这样一来,相国井的水滔滔而来,沟里的水都溢了出来,向南注入漕河,上千的船只轮流来取水,瞬息之间就可以灌满百斛。还疏浚涌金池,将它分为上中下三个部分,让洗衣浴马这样的事不在上池发生。并且在上池的门外设置两道闸门,其中一道闸门连接上池并将河水引入,另一道设在石围栏里,并列摆放五根竹管作为水管来引出池水,池水汇入河中向东流去,经过三桥然后流入石沟,进而注入南井。

第二年春天,六井全部修治好,正好赶上干旱之年,从江淮到浙右的各地水井都干涸了,老百姓到了用瓦罐装水奉如美酒相互赠送的地步。而钱塘的百姓肩上扛的,舟船载的,南自龙山,北到长河盐官海边,都有丰足的水供牛马饮用,供人们洗澡。

我认为水是人们急需的,干旱以致井水干涸,并非年年会发生的。因为与水有关的隐患不常有,而忽视了水是人们的急需之物,这是天下的通病,难道只是对待水这样吗?

6.(1)[答案]“锁”字运用拟人手法,(1分)写出了阴云四布、雾压朱楼的情景,(1分)表现了深闺女子郁闷(烦恼)的心情。(1分)(意思对即可)

[解析]赏析用词之妙,首先要理解词语本义,然后去揣摩其语境义,看有无运用手法,在表情达意、意境形成、形象塑造上有什么特殊效果。

(2)[答案]下片运用了以乐景写哀情的手法,(1分)通过绿杨、海棠、红杏等景物的描写,表达了词人隐藏在心中的无限哀(春)愁(烦恼、惆怅等)。(2分)(如果表现手法答“用鸟语花香来反衬自己的惆怅”也对)(意思对即可)

[解析] 词的下片主要是写景,所以要关注和描写抒情有关的表现手法,答题时要结合诗句具体内容进行分析。

[诗歌赏析]

朱淑真是一位多愁善感的女词人,这首词写一位闺中女子(实际上是作者自己)在明媚的春光中,回首往事而感到愁绪万端。

上片“迟迟春日弄轻柔,花径暗香流”两句,描绘出一幅风和日丽、花香怡人的春日美景。“弄轻柔”三字,言和煦的阳光在抚弄着杨柳的柔枝嫩条。“弄”字用得很妙,形象生动鲜明。然而开头所写的春光明媚,并不是眼前之景,而是已经逝去的美好时光。不然和煦的阳光与云雾是很难发生在同一时间内的。“云锁朱楼”的“锁”字,是一句之眼,它除了给读者云雾压楼的阴霾感觉以外,还具有锁在深闺的女子不得自由的象喻性。“锁”字蕴含丰富,将阴云四布的天气、深闺女子的被禁锢和心头的郁闷,尽括其中。

下片着重表现的是女主人公的春愁。这种春愁是由黄莺的啼叫唤起的。试想一个愁绪万端的女子,在百无聊赖之时,午睡醒来,听到窗外莺声巧啭,怎能不唤起她的春愁?

这首词笔触轻柔细腻,语言婉丽自然。作者用鸟语花香来反衬自己的惆怅,这是以乐景写哀情的手法。作者在写景上不断变换画面,从明媚的春日,到阴霾的天气。时间上从清明之前,写到清明之后,有眼前的感受,也有对往事的回忆。既有感到的暖意,嗅到的馨香,也有听到的莺啼,看到的色彩。通过它们表现了女主人公细腻的感情波澜。以听觉写鸟声的流动,使人辨别不出鸟鸣何处,词人的春愁,也像飞鸣的流莺,说不清准确的位置。这莫可名状的愁怨,词人并不说破,而是留给读者去想象,去补充。

7.(1)大观园 (2)钱梅芬 (3)屠维岳 (4)弗罗洛 (5)姑姑(每处1分,外国人名可酌情放宽要求)

8.(1)关羽斩颜良之后,听到了曹操的夸赞,便对曹操说了这番话。(2分)张飞用三声大喝吓退曹军,(1分)吓死夏侯杰,(1分)之后又断桥以阻拦曹军。(1分)

(2)写给在巴黎的朋友和恋人。(2分)欧也妮更加同情堂弟。(1分)她把自己的全部积蓄6000法郎送给堂弟,(1分)并与夏尔私订了终身。(1分)

9.[答案](1)相比之下,孔子的志向是希望老人得到安养,朋友间以诚相待,少年得到关爱,(1分)其关注的层面更为广阔。(1分)

(2)子路有志于治理好一个千乘之国,(1分)有好东西愿意和朋友们共享;(1分)他性格豪爽仗义,(1分)但性情鲁莽,过于崇尚勇力。(1分)

[解析]第(1)题考查对作者观点的概括。孔子的志向在原文中表述为“老者安之,朋友信之,少者怀之”,说的都是对别人的关爱,而子路、颜渊的志向则都是从个人的创业、成就出发,由此可见,孔子的志向更多地体现了“大爱”“仁者爱人”的特点,是一种更宽广层面的关爱。答题时,应将原文对应语句的含义用自己的话加以表述,并在此基础上加以抽象概括。第(2)题考查对材料内容的分析与概括,其中“评价”的要求略有超纲。考生首先要对子路之志进行概括,子路的志向在原文中有两处,一是三年治理好千乘之国,二是与朋友共享车马衣裘,据此可以推知,子路为人豪爽,但对如何治理好国家缺少清醒的认识,显得鲁莽、冲动,甚至自以为是。答题时,既要结合原文将子路的志向表述清楚,也要对子路的个性进行一定的概括评价。

[参考译文]

①(子路、曾皙、冉有、公西华陪孔子坐着)子路不管不顾地回答说:“拥有一千辆兵车的国家,夹在大国中间,外有敌军进犯,内又有饥荒发生;我来治理它,等到三年,可使百姓具备勇武精神,并且明白道义。”孔子微微一笑。

②颜渊、子路侍立于孔子身旁。孔子说:“何不各自谈谈你们的志向呢?”子路说:“愿把自己的车马衣裘与朋友共享,即使用坏了也不悔恨。”颜渊说:“愿不自夸长处,也不自夸功劳。”子路说:“希望听一听先生的志向。”孔子说:“(我的志向是)对老者,使他们安逸;对朋友,信任他们;年轻人,关心他们。”

10.C [解析]C项属于强加因果,“纯文学在上世纪80年代有较广阔的存在空间”与经济无关,而且也不全是因为书少,更主要的是因为当时的“精神阅读”的观念。

11.B [解析]B项“通俗文学、青春文学”使“青少年读者形成了低俗的阅读趣味”属于无中生有,其对纯文学阅读趣味的影响也尚未定论。

12.[答案]①阅读趣味的变化使青少年读者读不懂甚至排斥纯文学;②中老年读者因时过境迁,可能失去了阅读纯文学的热情;③中年读者的工作生活压力和老年读者的精力不济也使他们无法阅读纯文学。(每点1分,大意对即可)

[解析]本题考查对文章内容的概括。题干设问的语句出自于原文的倒数第二段,由句中关联词“所以”可以推断,题目的答案应在4~7段。由于文章将读者按年龄分段并分别进行阐述,因此,本题组织答案时,也应按读者群的不同进行分点作答。

(甲)

13.[答案] BC(答对一项给3分,两项给5分)

[解析]B.“仍旧把自己看作一名吕剧演员,她依然保留着作为演员的清高”错,体现了她处处为顾客考虑,心细;C.“对不尊重自己劳动成果的人表示了反感”错,表现了她对自己的劳动成果被漠视的失落。

14.[答案]①小说四次出现唱词,起到贯穿全文的线索作用。②暗示了主人公的身份,是以前县吕剧团的台柱子,也间接地表现了苏小青对以前的怀念。③文章情节上的推动作用,如《借亲》的唱词引出“醉酒者”。④渲染气氛,表达情感。如文章结尾的唱词,情感蕴藉,起到了意在言外的效果。(每一点1分。意思对即可)

[解析]小说中从头到尾出现的内容一般是起线索的作用,录音机里不断放出的唱词一直在出现,也起线索的作用;另外,答题一定要考虑小说的三要素和主旨角度,从人物的角度来讲,录音机里不断放出的唱词是戏曲唱词,暗示了主人公的身份;从情节的角度来讲,每个唱词后面都有事件的发生,所以推动了情节的发展;从主旨上来讲,唱词反复出现,渲染了气氛,深化了主旨。

15.[答案]观点一:苏小青一生都沉迷在演员的角色中。①苏小青修鞋时时刻不忘听吕剧。②虽是修鞋,但苏小青的打扮、气质和派头永远还是一名演员的模样。③苏小青用近乎艺术表演的形式对待修鞋。

观点二:苏小青能心安理得地做一个补鞋匠。①苏小青补鞋时脸色平静,一副心满意足的神态。②苏小青补鞋时态度认真,工作精益求精,显然已经完全进入了角色。③面对醉酒青年的挑衅,苏小青能够态度平和,不卑不亢。

观点三:追求艺术完美是苏小青的生活态度,这与其工作性质无关。①苏小青唱吕剧,成了剧团的台柱子;修鞋后,成了县城里手艺过硬的补鞋人。②苏小青修鞋认真细致,她既是修鞋,同时也是在展示艺术。③对生活艺术般的追求使得这个修鞋女子气质高雅,威严而不可辱。

(选择一个观点作答。每一点2分,答出3条给6分。意思对即可)

[解析]本题考查探究能力,无论选择哪种观点,都要观点明确,理由充分。回答理由时,要从自己所选的观点的角度,结合原文回答,切不可脱离文章内容,主观臆造答案。

(乙)

13.[答案] CE(答对一项给3分,两项给5分)

[解析]C项,“认为应该独善其身”理解错误,原文说“张謇远离官场并非出于文人的清高或英雄迟暮的消极”;E项,“性格上也是非常孤僻的”理解错误。

14.[答案]①不怕吃苦,执着,不服输。从他努力读书,五次参加科举考试就可以看出来。②热爱祖国,以强国为己任。辞官还乡,有一腔“救亡图存、振兴民族”的爱国情怀,“予为事业生,当为事业死”。③有胸怀和眼界。不仅经营好自己的集团,还努力发展整个地区的经济。④清廉。墓穴中仅有礼帽一顶,眼镜一副,折扇一柄。⑤甘于寂寞,忍受孤独。(每点1分,答出4点即可)

[解析]从发奋读书五次参加科举可以看出他的执着;实业救国可以看出他的爱国热情;从张謇在创办实业时说的话可以看出他的眼光和胸怀;从死后墓穴内的东西可以看出他的清廉;“他一生孤独,精神支撑是内心崇高的社会理想”可看出他甘于寂寞。

15.[答案]示例一:孤独有助于事业成功。因为只有孤独,才会使自己静下来,不受外界干扰,做自己认为正确的事情。如张謇不畏别人的眼光,为了中举付出26年的时光,参加五次科举考试,正是这种甘于孤独的精神使他最终获得成功。

示例二:孤独与成功没有必然联系。张謇的成功,源于他内心崇高的社会理想和念念不忘的兴国之梦。人可以甘于寂寞,但不能过于孤独,孤独会使自己变得狭隘,不容易打开思路,遇到困难时也更容易陷入绝望,不易脱离困境。由此可见孤独与成功没有必然联系,应该多与人沟通、交往。

(言之成理即可)(观点2分,结合文本分析4分)

[解析]探究时要有明确的观点。若认为“孤独与成功有关系”,主要从孤独可以让人冷静下来审视自己,可以集中精力做自己的事情;若认为“孤独与成功没有必然关系”,主要从孤独让自己封闭,容易陷入绝望困境等方面来回答。

16.[答案] (1)“体力劳动的强度高”改为“高强度的体力劳动”。(2分)

(2)图书室(有关文化方面的即可)(1分)

[解析] 第(1)题考查病句修改。“体力劳动的强度高”是一个主谓短语,与前两个短语的结构方式不匹配,无法并列。

第(2)题实质也是考查病句,填入的词语如果不产生语病,就应当符合两个方面的条件,一是填入的词语应是一个名词,能与“篮球场”构成并列短语;二是所填入的词语应属于“文体设施”中的“文”,否则就会前后矛盾。

17. [答案] 示例:群贤毕至,著宇宙华章(内容相关、结构相同、词性相对各1分)

[解析]本题是一道由课内向课外延伸的好题,既考查了对课文的背诵和理解,也考查了对联知识。背诵过关的同学由“少长咸集”马上能联想到“群贤毕至”,而且对仗工整。至于“观自然万物”的下联,考生可自由发挥,只要符合《兰亭集序》的内容,而且对仗整齐,都可视为正确答案。

18.[答案]示例一:男孩和女孩的差异不是危机,是正常现象。从成长发育角度看,中小学女生普遍要早熟一两年,理解能力、自理能力都要好一些;现行的中小学教育的应试体制,适合记忆力好、能静下心学习的女生。男生比较占优势的动手、实践等方面的能力,往往难以通过现在的书面考试方式检测出来,而且男生的创新能力、逻辑思维能力等,很多在工作中才有所体现。

示例二:关注教育中的“男孩危机”确实有必要。一些娱乐节目的影响,以及父教缺失、母亲溺爱等社会大环境导致男生不够阳刚。应“因性施教”,探索新的教育评价机制,让男孩的优势在学习中也能够得到认可;研究多元化的教学方法,尊重男孩喜欢竞争、对抗、刺激和实验等兴趣差异;在师资上,国家政策要鼓励男教师多进入幼儿园和小学,改善教师性别比例。

[解析]解答本题首先要准确地概括题目材料所介绍的“现象”,以便为表达看法提供评论对象。题目的两则材料在内容上存在互补关系,材料二实质是材料一的原因解说,因此概括题目材料所介绍的社会现象,可通过筛选材料一的关键词来实现。据此可概括出材料所提示的社会现象是:教育中存在“男孩危机”。对“男孩危机”的评论,可以从成因、后果、解决方法等多个角度考虑,也可以综合多个角度来组织答案。答题时应列举两个角度以上的理由,并注意语言表达的简明连贯。

19.略。

阶段评估检测(三)

1.(1)多于在庾之粟粒 (2)挟飞仙以遨游 (3)长太息以掩涕兮 (4)可以横绝峨眉巅 (5)居庙堂之高则忧其民(6)别有幽愁暗恨生(一处1分,错、漏、添字的,该处不给分)

2.C [解析] “竟”在文中应为“最终,终究”,因本句上下文并无转折之意,译为“竟然”显然不通。

3.B [解析]“庶几”意为“差不多”,因此,本题要求筛选出的是表现方干“类似”黔娄的具体行为,其中②说的是方干早年为求取功名而奔波;④表现的是方干的诗歌艺术水平;⑤是黔娄的表现,都与题干要求不符合。

4.C [解析] “几次前往,几次题字留念”有误,原文“几遍”,意为“几乎走遍”。

5.(1)过了一些日子,王大夫因病去世,这件事也就没有办成。(大意1分,“行”“以”各1分)

(2)先生以天下的淡薄之味为甘美,以处天下的卑贱地位而安心。(大意1分,“甘”“安”各1分)

[参考译文]

方干,字雄飞,桐庐人。他从小就有优秀的才能,却懒散粗疏不谋划生计。大中年间,方干考进士时落榜了,就隐居在镜湖中。镜湖北岸有茅舍,西面有松岛,每当风清月明之夜,他就带着年幼的儿子和邻居老人,坐着小船在湖面上来回,很是满足了自己平生的心愿。他住的地方水泉树木幽深隐蔽,周围的一花一草都能让客人流连忘返。方干家中贫穷,却保存了一具古琴,他漫步吟诗或醉后仰卧时以弹琴自乐。

徐凝当初作诗有名气,他一见到方干就很器重,二人于是成为师友,徐凝就教方干作诗。方干有一首赠给徐凝的诗写道:“把得新诗草里论。”当时人说“草里论”三字的反语是“村里老”,怀疑方干这句诗是在讥讽嘲笑徐凝,这种说法是不对的。方干容貌丑陋,嘴唇豁缺,生性喜欢欺侮别人。王大夫任浙东观察使时,恭请方干到府上,方干误作三次跪拜,人们就称他为“方三拜”。王大夫嘉许方干的操守,准备向朝廷推荐他做官,就托吴融起草奏章,过了一些日子,王大夫因病去世,这件事也就没有办成。方干早年进京会试往来于长安洛阳间,好事的名公巨卿争相聘请他,但他进士科名终究未能到手,于是回乡,不再有官场进退荣辱的念头。浙江一带凡有园林名胜,方干总要前去拜访它的主人,差不多所有的名胜之地都留下了方干的题诗。

当初,李频向方干学习写诗,李频进士及第后,诗僧清越写诗向方干祝贺说:“弟子已折桂,先生犹灌园。”方干在咸通末年去世,他门下的弟子在一起评论他生前的品行事迹,给他加谥号称“玄英先生”。乐安人孙郃等收集方干遗诗三百七十余篇,编为十卷。王赞评论方干的诗说:“深刻入肌,洗净骨骼,冰雪般晶莹,彩霞般绚丽。美食佳味自具,从不拾人牙慧。华丽而无花香之气,清苦而无瘦棘之态。一旦得意于心,倏忽与神交会。词语虽未写出,文意已经独到。”孙郃也评论说:“方干诗的杰出,如仙花超于常花;方干诗的鸣声,如鼍龙压倒凡响。”细察这些论述,评价是不过分的。

古代黔娄先生死时,曾参与门生们前来吊唁,他们问:“先生去世了,用什么词作他的谥号?”黔娄的妻子说:“用‘康’字。”曾参说:“先生在世时,食物装不满饭锅,衣裳遮不住身体,死后棺材小得连手脚都装不进去,旁边也没有酒肉供享。活着没过上好日子,死后也是冷冷清清的,有什么安乐可言,却把谥号叫做‘康’呢?”黔娄的妻子说:“先生在从前,国君要让他当宰相,他推辞不接受,这就是显贵有余。国君送给他三十钟米,他推辞不收,这就是富足有余。先生以天下的淡薄之味为甘美,以处天下的卑贱地位而安心,对贫贱不忧伤悲戚,对富贵不忙碌钻营,求仁而得到仁,求义而得到义,用‘康’作他的谥号,不也是很合适的吗?”方干,是韦带布衣之士,活着人称高尚,死后谥号玄英,其大略事迹,不是也和黔娄差不多吗?

6.[答案] (1)渲染夕阳下平凉凄迷荒凉的气氛,为下面的抒情做铺垫。(意思对即可)(2分)

(2)示例:抒发了诗人想仿效班超投笔从戎,建功立业的豪情壮志;更流露出诗人的遗憾之情,既对自己在谈经中虚度年华深感愧疚,又对自己想在边塞为国效力却已力不从心深表叹息。(每点2分)

[解析]第(1)题考查诗歌内容思路,借鉴了散文的考查方法。“白日斜”处于首句,又是景物描写,因此其内容作用是点明时间,渲染氛围;结构作用是为下文作铺垫,答题时,考生还要注意结合诗歌的具体内容,不能架空分析。第(2)题考查诗歌情感,其难点在于“投笔”典故的理解。“投笔从戎”说的是东汉时的班超扔掉手中的笔决心去边疆从军打仗,建立功业,诗歌借此典故,表达的是一种豪情。此外,考生应注意到,最后一句表达的情感显然与典故的意义不同。因此,本题至少应分两点作答,即既答出“豪情壮志”,又要答出“愧”“鬓有华”所透露出的愧疚和伤感。

7.[答案] BD(答对一项给3分,答对两项给5分)

[解析]B.①“听到这个消息”应是“出嫁前的晚上”;②“觉慧并不爱鸣凤”应是“此时觉慧正忙着写稿子”。D.①“姑妈派马斯洛娃去见他”应是“马斯洛娃决定亲自赶到车站去见他”;②“只和涅赫柳多夫说了几句话”应是“来不及和他说一句话”。

8.(1)华佗。(1分)华佗提出用斧头剖开曹操脑袋,取出风涎以医治头痛方法,(2分)曹操认为华佗是借机为关羽报仇,便将华佗下狱害死。(2分)

(2)当天夜里爱斯梅拉达被黑衣人骗上了船,上岸后她发现黑衣人是弗罗洛。(2分)弗罗洛威胁她,要她在绞刑架与他之间选择。(1分)爱斯梅拉达宁死不屈,弗罗洛将爱斯梅拉达交给隐修女,自己去叫军警。(2分)

9.[答案] (1)禹顺应水的本性,引水入海;白圭逆水设堵,排水入邻国(以邻为壑)。(每点2分,大意对即可)

(2)损人利己(不顾他人利益),急功近利(只顾眼前利益),都违背了“仁”的要求。(2分,大意对即可)

[解析]第(1)题考查概括能力,考生首先要从原文中筛选出表现白圭和禹的治水的词语,然后再把这些词语转化为自己的语言。原文中表现禹治水方法的主要有两个词语,即“道”和“四海为壑”,表现白圭治水方法的词语则是“邻国为壑”和“逆行”。第(2)题题干之所以要求“从道德层面上看”,显然是为了避免与上一题答案的重复。白圭治水的具体方法已在上一题考查了,而这一治水方法背后,显然还有其“道德缺陷”,原文用“仁人之所恶”来表达,也就是说,白圭治水的方法违背了“仁”。考生答题时,应当顺着这一思路,分别概括白圭治水的方法的“道德缺陷”。

[参考译文]

白圭说:“我治水的方法胜过大禹。”孟子说:“你错啦。大禹治水,是顺应水性,所以大禹把四海当作蓄水场所。现在你却是把邻国当作蓄水场所。倒流泛滥的水叫洚水,洚水就是洪水,是仁人最讨厌的。你错啦!”

10.D [解析] “阻塞了它们的交通”于文无据,“失去了生命力”的原因并非交通问题。

11.A [解析] “不在于”错,原文是“并不仅仅来自于”。

12.[答案] 旧城改造不能只追求现代化,而应当重视对城市历史文化的保护。(3分,大意对即可)

[解析]本题考查对语言的赏析和文章内容的概括。考生首先要将德国历史学家的这句话还原出本义,即“我们的现代化城市,中国以后一定会有,但中国的有历史文化积淀的古城,却是世上仅有的”,然后在此基础上推断出作者反对“大拆大建”、希望保护好城市历史遗迹的观点。

(甲)

13.[答案] CE(答对一项给3分,两项给5分)

[解析]C项错在“写出了当年家庭的艰难”;E项错在“告诫世人要像苦瓜一样,不但要有外表美,而且要拥有心灵美”,犯了牵强附会的毛病。

14.[答案]①从视觉、触觉的角度进行描写,写苦瓜白玉般的色彩,疣状突起部分长得细致,触摸起来清凉滑润。(2分)②运用衬托手法,用清朝的玉器“白玉苦瓜”来衬托苦瓜的颜色质地,衬托之中既突出了苦瓜的颜色质地之美,又表达自己的赞美之情。(2分)

[解析]本题考查对文章艺术技巧的赏析。散文中对景物的描写技巧常见的有:动静、虚实、五官感觉以及修辞手法。考生可以用这些手法一一对照原文,言之成理即可。例如,写玉雕的内容也可理解为“类比”手法。

15.[答案]作者在文章的最后通过写看苦瓜树,表现了自己在田里看到颗颗白玉般苦瓜的喜悦激动之情;通过对买苦瓜的描述,表明了美与苦是并生的,人不能只要美而不要苦的观点。(3分)与第②段照应(与前文的“白玉苦瓜”相照应),使文章结构严谨,升华了文章的主旨。(答“照应”给2分,若答点明主旨或卒章显志不给分)

[解析]本题考查对文章结构思路的理解分析。文章结尾内容往往有点题和呼应开头的作用。结合结尾段的具体内容,可以推断,“美与苦是并生的,人不能只要美而不要苦”一句正是全文的主题;而“白玉般的晶亮与透明”显然呼应了第②段;“美丽的错误”则呼应了第⑧段。

(乙)

13.[答案] BC(答对一项给3分,两项给5分)

[解析]本题考查筛选文中相关信息和分析综合的能力。A项,使钱玄同“成为‘五四’新文化运动的揭幕人之一”的是“向陈独秀主办的《新青年》杂志投稿,倡导文学革命”,而“提出改革大纲十三事”只是传主的主要活动之一;D项不准确,两件事相隔时间之近,虽然也可以体现出感情之深,但作者主要是为了说明传主得病之突然;E项,理解错误,钱玄同“古文造诣很深”与“主张废除方块汉字”并不矛盾,正是因为他对汉字研究的深入,才有“难认、难记、难写,不利于普及教育、发展国语文学和传播科学技术知识”的认识,才主张废除方块汉字。

14.[答案]①在经学研究方面,他有明确的思想方法,创见甚多;②在古史研究方面,独见很多,影响很大,一些观点得到了郭沫若的赞赏。(每点2分,大意对即可)

[解析] 主要对文章的第三段进行概括。

15.[答案]观点一:钱玄同是“对得起自己、对得起国家民族的一位完人”这一说法有道理。①他倡导文学革命,是“五四”新文化运动的揭幕人之一,成为鼓吹新文化,攻击封建主义,提倡民主、科学的勇士;②他在教学和学术研究方面有显著的贡献,在国语运动、文字改革方面也有突出的贡献;③他身处沦陷区,不做敌伪的顺民,保持名节。

观点二:钱玄同是“对得起自己、对得起国家民族的一位完人”这一说法不准确。①钱玄同在倡导文学革命、教学和学术研究、保持名节等方面确实对得起自己、对得起国家民族,是一位了不起的人物;②但他也有认识问题偏激的一面,例如他主张废除方块汉字,说中国根本改革之路在“欧化”等,这些说法虽然受时代的局限,但在认识上也有不全面之处。③乐颜的说法具有很强的个人情感色彩,而在国家民族危难之际保全名节似乎更是对得起自己。

(观点2分,理由任答两点给4分)

[解析]本题考查探究能力。要做到观点明确,明确观点后要有具体分析,要言之成理。“完人”主要从他的成就以及保持名节等角度阐释;不认同说法主要从他看问题有些偏激阐释。

16.[答案] (1)妄(1分) (2)示例:故步自封(或作茧自缚)(2分)

[解析] 本题考查汉字书写和成语积累,答题没有任何技巧,考生应重视日常的积累。

17.[答案] ②自由滑行(无动力飞行) ③再入大气层 ④着陆(软着陆)(每点1分)

[解析]本题考查筛选概括内容要点的能力。题目要求回答的是“飞船返回过程”的几个阶段,因此,筛选出来的词语都要符合“飞船飞行”的特点,而且,这一“飞船飞行”的动作,应当是持续一段时间的动作,这样才符合“阶段”的答题要求。

18.[答案]示例一:我赞成中南大学破格嘉奖刘路的举措。(2分)首先,刘路作为一个本科生,能够破解国际数学难题,既因为他卓越的数学天赋,也因为他出色的科研能力,这种能力值得肯定;其次,中南大学此举也能够给刘路在学术研究的道路上提供更好的保障,更大的平台;最后,这种行为也表示了唯才是举的决心,一方面能够吸引更多的人关注有创新能力的青年人的成长,另一方面也能够更好地鼓励年轻人进行学术研究。(5分)

示例二:我不赞成中南大学嘉奖刘路的举措。(2分)首先,因为刘路的突出贡献立即给予破格嘉奖的行为未免显得草率;其次,刘路还很年轻,给他太多荣誉不一定有利于他的成长,反而可能因为心理负担太重而产生副作用;最后,这也容易造成年轻人“一夜暴富”的心理,反而影响他们踏实工作。(5分)

[解析]本题考查语言的综合运用,材料的主要内容是:大学生刘路破解数学难题并获嘉奖优待。对于这一事件,值得评说的点只有“大学生”“嘉奖优待”,至于“破解数学难题”是没有评论的必要的。为此,考生概括自己的观点时,应注意观点表达的完整性和针对性,不能使用如“我赞同这种做法”这样含糊的说法。至于理由的阐述,应抓住“大学生”这一特殊身份,重点分析学校的嘉奖优待可能带来的利弊,以便支撑自己赞同或反对优待刘路的观点。

19.略。

阶段评估检测(四)

1.(1)扪参历井仰胁息 (2)不可说也 (3)凌万顷之茫然 (4)更相为命 (5)川泽纡其骇瞩(6)尚能饭否(一处1分,错、漏、添字的,该处不给分)

2.C [解析] 盍:何不。

3.D [解析]①是狼的外貌,与“强”不对应,也无法体现“中干”;④是人们对狼的反应,不直接表现;⑤是狼得意时的表现,不能体现其“中干”。

4. C [解析] “官吏不同意”之说有误,道士并没有面见官吏,文中道士之语都是对“士人”说的。

5.(1)我在夜里前去,假如有什么不测,还可跑回来占据我的洞穴,他们能对我怎么样?(“以”译为“在”,“脱”译为“假如”,“虞”译为“意外”,“若我何”译为“对我怎么样”,各1分。全句大意有较大出入,则先扣2分,再看得分点的翻译)

(2)庆幸的是它没有吃人,就饶了它吧。(“幸”“贷”各1分,全句大意有较大出入,得分点译对也不给分)

[参考译文]

原山有一处洞穴,位置并非在山的最顶端,与山下的村落极为接近。但由于所经过的路段较为险峻偏僻,以前大概是人所居住的地方。周围河谷阴冷漆黑,草木茂盛。近年来,有一匹狼占据了这一处洞穴。狼的形体又丑又脏,发出的声音很奇怪,能出入于百兽之中,也能变成人的模样。每天天黑之后,就背靠洞穴而嗷叫,能发出像小孩啼哭、寡妇哭泣、冤鬼哭号的声音,也能模拟市场的喧哗声,军队的击鼓吹号声,声音在树林山谷中回响,村落里的人都十分震惊害怕,久了之后也十分痛恨狼的叫声,但没有人敢对付狼。山中的野兽听到了,认为狼是一个了不起的英雄,都成群结队地去依附它。有一只狡猾的狐狸,很会巴结逢迎,就请求尊狼为百兽之王,其他的动物也都附和。于是狼大喜,发出的声音更加凶狠凄厉,有时也到乡村,偷盗一些鸡、猪来吃。要是碰巧与人相遇,也还不敢攻击人、吃人。狡猾的狐狸对狼说:“从这里到城里的市集才几里路,何不到城里一游?”狼心里有些害怕,但心里又想:我在夜里前去,假如有什么不测,还可跑回我的洞穴,他们能对我怎么样?于是狼就擅自闯入官吏的住处。官吏是海边人,他所常见的东西都是鱼鳖,见到狼大吃一惊,赶紧替它准备酒菜,礼节十分恭敬。狼心里十分得意,便放肆地吃喝起来。后来又几次去官吏的住处,渐渐地不再害怕白天。老百姓有时遇见狼,都吓得回头就跑,大家都以为是妖怪,于是互相传言,争着关闭门户。有一个读书人从墙上窥视一番后,判断说:“这是狼。”并赶紧告诉官吏,让他不要接近狼。但这时官吏已经无法拒绝与狼的来往,因此不肯听从,狼于是更加肆无忌惮,把自己的皮毛修整得十分光泽,放纵地来来往往。从村庄到城里,大家都相互告诫,小心地避开狼。

有一个道士,身怀绝技,隐藏在深山里,听到这个消息后就去拜访读书人,说:“先生你为什么这样怯弱?这样的话狼将会吃人了。假如不除去它,您和大家都将会成为狼的口中之物。”读书人说:“我本来也知道,但大家的心意不统一,也没有弓箭罗网等工具,况且官吏和狼还很亲密。”道士说:“官吏又能得到什么好处?俗语说:‘野兽进了门,主人就会被迫离开。’现在狼已经进入主人的房子了,实在没有办法,我来帮助您。”于是道士就写了讨狼檄文,把这件事告诉神仙。当天晚上,一场大雷电夷平了狼的洞穴。狼刚好外出还没回来,但回来后就没有了住处。这时,狐狸和其他野兽也都四散逃走,狼没有办法,只好逃窜到野外,畏畏缩缩,再也没有往日的威风。到了晚上,它的叫声凄凉断续,只是像野鬼号叫而已。村里的人聚在一起商量,认为狼已经没有威胁,便拿着棍棒,带着瓦片石块,打算把狼杀死。城里市集里的人听说了,也争着出城来打狼,道士笑着劝阻说:“近来神仙已经诛杀它一次了,所幸它没有吃人,就饶了它吧。你们现在只想着要杀了它,以前你们为什么会相互告诫,没有人敢去冒犯它呢?你们这些人啊,真是太过分了。”大家这才作罢,狼于是也远远地逃走了。

评论说:狼本来不能有什么作为,之所以能恐吓发动其他野兽、摆布官吏来耍威风,是由于它有用来安身的洞穴,能发出凶恶的叫声。狼失去了洞穴,也就一败涂地了。属于人的地盘本来就不能借给狼占据的。

6.(1)[答案]可分为三层。首两句为第一层,作者向黄州东坡表达思念之情,引起下文;(1分)次两句为第二层,是苏轼对别后黄州东坡的冷清荒凉景象的揣想;(1分)末三句为第三层,表达归耕东坡的意愿。(1分)

[解析]以句号为单位,本词共三句话,亦可以看作是三个层次。第一层写想念黄州东坡故地;第二层写想象离开后的凄凉之境;第三层写向往东坡故地的耕居生活。

(2)[答案]同意。整首词表现了作者对清静自适生活的怀念,以及遥想东坡、意欲归去的心情,(2分)而且语言清新,淡雅自然,充满“韶秀”气息。(1分)

[解析] “韶秀”是美好秀丽之意。从词的内容来看,本词亦有这层意思。语言自然清新,不饰雕琢,也没有磅礴豪放之气息。

[诗歌赏析]

词牌《如梦令》原名《忆仙姿》。五代时后唐庄宗李存勖所作,因嫌其名不雅,遂用尾句“如梦,如梦,残月落花烟重”,改为《如梦令》。为单调。

这首词是寄给黄州友人的。它表现了作者对当年(1080~1084)贬官黄州、躬耕于东坡时清静自适生活的怀念,和遥想东坡、意欲归去的心情。

“为向东坡传语”,此处东坡,系指地名。作者贬官黄州时,故友马正卿为他请得城东的营防废地数十亩,作者开垦躬耕于此,并以这个地名作了自己的别号。所以,此处作者请友人向东坡传话。以地当人,足见作者留恋故地的情感之深。本词在开头就造成悬念。词人说:“人在玉堂深处。”唐时翰林院设在宫中,称玉堂。此后玉堂成为翰林院的美称。这一句看似平淡无奇,实则颇有深意。潜台词是说:我现在身居翰林,你不必惦念我。这是告慰东坡。以下两句便是询问、悬想东坡的情况。分别多年,有谁来眷顾友人?那里恐怕是积雪满地,人迹罕至,友人大约会感到寂寞。想到这里,作者笔锋一转,直抒胸臆。希望再度奔赴黄州,于潇潇春雨之后,耕作于东坡之上。“归去,归去”,繁音促节,更显示归心之切。

这首词作者以地写人,表现作者思念故地的情愫。有告慰,有询问,有揣想,有向往,如面对老友,娓娓谈来,亲切自然。同时也流露出作者在京中新旧两党夹击下,意欲远离政治漩涡、出知外任的心思。果然,翌年三月,作者即以龙图阁学士出知杭州。

7.[答案] BD(答对一项给3分)

[解析]B项“与赵伯韬商谈筹办”有误,吴荪甫是与杜竹斋等人一同筹办“益中信托公司”;筹办“益中信托公司”是在双桥镇暴动之后而不是在“这时”。D项弗罗洛对卡西莫多的这一举动只是奇怪,妒火中烧是因为听甘果瓦说他与爱斯梅拉达是夫妇,并且追根究底。

8.(1)打听甘宁、黄盖投降的消息是否真实。(1分)连环计。(1分)庞统在西山庵诵读兵书,引蒋干与自己结识,之后在蒋干的建议下,连夜与蒋干过江投奔曹操,最终使曹操中了连环计。(3分)

(2)“他”指的是涅赫留朵夫。(1分)涅赫留朵夫靠副检察长谢列宁的帮助,将玛丝洛娃由原判服苦役改为一般移民流放,(2分)涅赫留朵夫觉得自己与玛丝洛娃的结婚不再有问题,于是萌生了这些想法。(2分)

9. [答案](1)孔子认为社会无道时可以远离政治,隐藏自己,明哲保身;孟子认为社会无道时应该牺牲自我来捍卫正道。(答对1点给2分,答对2点给3分)

(2)孟子认为不应该用牺牲道的原则来迎合人,应该坚守道义;不可拿原则做交易,换取个人的利益。(答对1点给2分,答对2点给3分)

[解析]第(1)题考查对材料内容的概括能力。答题时要注意题干的限制条件“无道时”,据此筛选出的原文语句是“卷而怀之”和“以身殉道”,将这两句转化为自己的语言表述出来即可。第(2)题考查对作者观点的概括和分析。“未闻以道殉乎人者也”一句包含两层意思,一是孟子反对“以道殉乎人”,二是表现孟子对于“道”的坚守。考生答题时,应结合孟子所说的“以道殉身”“以身殉道”,将这两层意思都阐述完整。

[参考译文]

①孔子说:“蘧伯玉真是一位君子啊!国家有道就出来做官,国家无道就(辞退官职)把自己的主张藏在心里。”

②孟子说:“天下政治清明时,用道义随身行事;天下政治黑暗的时候,用生命捍卫道义。没有听说过牺牲道义而屈从于他人的。”

10.D [解析] 这里说的是山水诗的好处,不是山水诗产生的原因。

11.C [解析] 对原文理解错误。原文说“古今诗文大家、艺术巨匠大抵都有‘读万卷书,行万里路’的经历。‘读万卷书’

正好可以弥补‘行万里路’的不足”,不能由此就说“古今诗文大家、艺术巨匠大抵都用‘读万卷书’来弥补‘行万里路’的不足”。

12.①有力地证明了“好的山水诗总是包含着作者深刻的人生体验”,“优秀的山水诗大都具有‘诗中有画,画中有诗’的特征”这两个观点;(2分)②同时也增加了文章的美感和情趣。(1分)(大意对即可)

(甲)

13.[答案] DE(答对一项给3分,两项给5分)

[解析] D项,不是向电车公司打的报告,是向居委会。E项,这是讽刺小说,批判工作作风而非赞扬。

14.[答案]①找到认识的电车司机,得知可到电车公司失物招领处寻找;②到电车公司失物招领处,见到了丢失的套鞋。③到居委会开证明,在打报告后拿到证明。④在失物招领处,拿到丢失的套鞋。(每点1分,意思对即可)

[解析] 梳理故事情节,他先打听认识的司机,得到认领套鞋的信息;然后去认领;接着去居委会开证明;最后拿到了套鞋。

15.[答案]①与前文形成呼应,强化了讽刺的效果:前文写找套鞋的波折,表现工作人员的死板教条,讽刺意味明显。(2分)②给读者留下回味的空间,更能引发人们的思考。(2分)③结尾处写对丢失的另一只套鞋不抱找到的希望,更加突出了讽刺的意味。(2分)

[解析] 从文章结构内容及作用方面考虑,意思对即可。和上文的故事情节相对照,起到了极强的讽刺作用,更能引发读者的思考。

(乙)

13.[答案] CD(答对一项给3分,两项给5分)

[解析] C项,“从贝多芬的身上获得信心和勇气”错;D项,“像雷雨一样令人可怖,永远充满着悲哀”错。

14.[答案]示例:渲染沉痛悲伤的氛围,也象征贝多芬一生遭受的苦难;暗示贝多芬不认识的人也对他无比崇敬,侧面表现贝多芬的伟大的人格魅力。

[解析]本题考查对文章结构思路和艺术手法的理解。传记文章中的环境描写,与散文、小说中的环境描写一样,其目的都在于渲染气氛,表现主题。因此,文中写天气的恶劣,就是为了表现贝多芬,表现作者对贝多芬去世的情感。“陌生的手替他阖上了眼睛”属于细节描写,细节描写的作用往往是表现动作发出者的情感,进而表现文章的主题,文中这一描写,表现的是人们对贝多芬的尊敬和对贝多芬去世的悲痛。

15.[答案]示例:因为贝多芬的一生就是“用痛苦换来欢乐”的一生,体现出勇敢、坚韧、不屈和乐观的精神。并且贝多芬以自己的经历和乐曲,教导那些受苦而奋斗的人学会隐忍痛苦;教会人们不向苦难命运屈服,努力奋斗;用他的苦难铸成乐曲来安慰和鼓励人们,给人勇气和力量。可以说,贝多芬用自己的人格魅力和精神完成了人生伟大的事业。(如有其他观点,言之成理亦可)

[解析]本题考查对语句含意的理解和文章创作意图的挖掘。题目所引述的语句是贝多芬墓碑上的铭文,显然是对贝多芬一生和精神的总结,只是表述的文字带有一定的诗意,不够直白而已,考生可以结合贝多芬的生平事迹,将这些铭文转换为明确的语言,概括出贝多芬的人生和精神。

16.C [解析]语段描写对象为二月兰,自然要对二月兰作说明,可确定首句为③;⑤介绍花形和颜色,紧随③;④紧接⑤强调“没有什么特异之处”;②表示转折,写二月兰分布广,“以多取胜”;①和⑥紧承②简略描写二月兰的生长过程。

17.[答案] 示例:听听春天的脚步声吧,她会带着新的一年到来。一切才刚开始。(3分)

[解析]本题考查仿写句子。横线上要填写的语句应与前文中“造物者说……”一句句式相同,即都运用感叹句和因果倒装的复句形式;内容相关,即都表达了对未来的乐观期盼。

18.[答案]示例一:我赞成《浅薄》一书的观点。首先,因特网对人的认知能力的确会产生有害作用,会降低人们专心致志和深思熟虑的能力。使大家生活在浅薄之中;其次,在网上“冲浪”、扫描、浏览的时间越多,我们就越适应网上的思考模式;再次,网络不仅改变了我们的生活和交际方式,也迅速而深刻地改变了我们的大脑,使我们的思考缺乏创造性。

示例二:我赞同《布鲁姆伯格商业周刊》书评的观点。首先,因特网的出现不仅是科技的进步,而且也是对人类能否对更多的事情进行更深入的思考的挑战;其次,因特网只是一个工具,能否分散我们的注意力,降低我们的理解力,从而把我们变成肤浅的思想者,取决于我们自己;再次,因特网的海量信息不断刺激我们的大脑,能够激发我们更深入的思考。

(观点鲜明1 分,阐述连贯合理6 分。这是一道开放题,其他看法只要源于材料,言之成理即可)

[解析]本题考查语言综合运用。题目材料分成两段,分别介绍了两种对立的观点,考生确立自己的观点时,只能从中选择一种。阐述理由时,应着重抓住“因特网”的优点或缺点,并按照由浅入深,由现象到本质的逻辑,将这些优点或缺点分条阐述清楚。

19.略。

阶段评估检测(五)

1.(1)金就砺则利 (2)徘徊于斗牛之间 (3)只是朱颜改 (4)梧桐更兼细雨 (5)丰年留客足鸡豚(6)茕茕孑立(一处1分,错、漏、添字的,该处不给分)

2.C [解析] 累:牵累,连累。

3.C [解析] ②说明其刚强耿直, ④是王亶望所设奸计,⑥说明其清廉。

4.B [解析] “百姓把他主持的治水工程称为周公闸、周公桥”是感激他兴修水利,造福于民,与其“廉洁正直”并无因果关联。

5.(1)(周梅圃)修复了风峪山堤坝,堵住山中积水,并疏导水流使它进入汾河,老百姓感激他。(“障”“导”“德”各1分)

(2)官吏中有人用征收粮食坑害百姓来取悦上级官员,(他们对这样的事)习以为常了。(“毒”“媚”“恒”各1分)

[参考译文]

梅圃,长沙人,姓周,名克开,字乾三,梅圃是他的自号。他凭借举人的身份被派到甘肃,并被任命为陇西知县,(后来)调往宁朔。他为人明白事理,敢于担当繁重的事务,做事情不辞劳苦。宁朔属于宁夏府,挨着黄河的有三条水渠:叫做汉来渠、唐延渠、大清渠,都是引黄河水入渠,用来灌溉百姓的农田。唐延渠经过的地方多沙而且容易泛滥,周梅圃整治该水渠使它变得狭长而且深邃,又稍微改变它的水流方向,唐延渠的行水得以安全畅通,而且渠道中设有暗洞,用来把过多的水排泄到黄河里,所以旱涝天气都依赖那条水渠。唐延渠的暗洞坏了,宁夏县的官吏想填了暗洞,而把唐渠的水都引入到汉渠,以利于宁夏的百姓,但(这样一来)宁朔就受害了。周梅圃全力监工修复原来的式样,使两县都得到好处。大清渠,是康熙年才开凿的,长三十多里,时间久了,首尾的石门都毁坏了,百姓不能再享受它的好处,周梅圃修复了它,都用时少但功效长远。周梅圃在宁夏有很多良好的政绩,而以治水的功绩最大,百姓把他主持建造的治水工程称为周公闸、周公桥。

不久,周梅圃被提拔为贵州都匀知府。跟从总督吴达善、侍郎钱维城处理贵州苗族的叛乱案件,对依法断案却处置不当的,他据理力争,毫不让步。被调往贵阳,也因为刚强耿直,触犯巡抚宫兆麟,因公事牵累被解除职务。受到引见,又被任命为山西蒲州知府,被调往太原。清理积聚的案件,修复了风峪山堤坝,堵住山中积水,并疏导水流使它进入汾河,老百姓感激他。

周梅圃经多次提拔,官至江西吉南道,因为犯了过错被降了官职,后又被提拔为浙江粮储道。在这个时候,王亶望担任浙江巡抚。官吏中有人以征收粮食加害百姓来取悦上级官员,(他们对这样的事)习以为常了;周梅圃早就听闻(这种事),对此深恶痛绝。周梅圃来到浙江,自己发誓不取丝毫好处,并向巡抚请求铲除积弊,与他约定要同心同德。巡抚表面说“好”,而实际上十分讨厌周梅圃,只是没有办法使他离开。相反上奏朝廷赞誉周梅圃有突出的才能,管理粮食储备是平常的政事,容易处理,而当时海塘正处在危急中,请求将周梅圃调去治理海塘。于是(周梅圃)被调任杭嘉湖海防道。周梅圃改建海岸石塘,石塘得到大的整治,而周梅圃终因积劳成疾死在任上,王亶望在官场中也最终因为贪婪而自毁。世人说(认为)王亶望如果能接受周梅圃的意见,哪里只是对国家有利,也有利于他自己的身家安全啊。周梅圃死后,家中十分贫困,天下人称赞清廉的官吏,都会提到周梅圃。

我认为:周梅圃,是乾隆年间一位守法循理的官吏。为守法循理的官吏写传,是史官的职责,撰写史传的文法非常严谨。我不担任史官的职位,为相互交好的朋友写家传,或许会有不切实的言辞。我称赞周梅圃治理政事的能力,为他作传,选事简单,我认为以后如有好的史官,选取我的文章并将它收录在循吏列传中,我也应该不会感到惭愧了。

6.[答案](1)筛,是“用筛子过东西”的意思,(1分)化无形为有形,形象地将阳光透过树叶的空隙,照在地上铺金撒翠的样子表现了出来,(1分)化平淡的现象为生动的动作,使画面活泼而富有生气。(1分)

(2)作者通过对“柳堤”“竹溪”“鸥鹭”“农父渔翁”等乡村景与人的描写及最后一句的议论,(1分)表达对田园自然风光的热爱及寄情田园、山水的闲适隐逸生活的情怀。(2分)

[解析]第(1)题鉴赏诗歌的语言。炼字题,就是通过关键字把握意境。炼字,一般是词类活用,或者运用了修辞手法,做题时首先应该指出这点,再从景与情两方面说明其效果,比如动静结合,化静为动,化意为象,化无形为有形,化无情为有情,化平淡为生动,化腐朽为神奇,变简单为多样等。第(2)题考查鉴赏文学作品的内容。直接抒情的,抓情感词,抓议论句;间接抒情的,抓意象(人物、事件、景物、器物),通过意象揣摩情感。如果是以景衬情,要分清是正衬,还是反衬;用了典故的,要注意区分是正用,还是反用。

7.[答案] AC (答对一项给3分,两项给5分)

[解析] A 项有误,不是觉民,而是“觉新”期待着朦胧的爱情;觉民是《家》 中向老太爷“抗婚”的胜利者,不存在“断绝”之说。C项有误,甘果瓦没有主动要求跟爱斯梅拉达交换身份代替她绞死,这是弗罗洛的主意;甘果瓦也不是为了报答爱斯梅拉达的救命之恩。

8.(1)说的是周瑜与黄盖使用苦肉计的事。(2分)赤壁大战前,黄盖故意与周瑜大唱反调;周瑜下令将黄盖打得皮开肉绽,卧床不起。随后黄盖诈降曹操,火烧曹军,立下大功。(3分)

(2)长安府府太爷的小舅子李衙内想娶张金哥,可金哥已与长安守备公子订了亲,张财主想与守备退亲,守备不答应。(3分)净虚老尼请王熙凤帮忙让守备退亲,王熙凤假托贾琏名义,叫人写信请长安节度使摆平此事。(2分)

9.[答案] (1)孔子认为,只有颜回能够长期行仁,其他弟子只能短时间行仁。(大意对即可)

(2)①孔子认为,一个人如果主动自觉追求仁德,一定会在仁德上有所收获,所以说培养仁德很容易。②同时,孔子又认为,君子要长期追求仁德,做到无论外界情况如何,都应恪守仁德。而自己的学生中,只有个别人能做到,因此,他又认为培养仁德很困难。(大意对即可,每点2分)

[解析]第(1)题考查对作者观点的概括,根据题意可筛选出原文信息应在第①段,即“回也,其心三月不违仁;其余,则日月至焉而已矣”,将本句的意思改用自己的话表达,即可得出答案。第(2)题考查对作者观点的分析。题目中已给出作者的观点,即“培养仁德很容易又很困难”,现在考生要完成的任务是,从原文筛选有关词句,结合文章理解转换为自己的语言,并扣题作答。从材料内容看,能表达“培养仁德很容易”这一观点的原文主要在于第②段,即“仁不,欲仁仁至”。能表达“培养仁德很困难”这一观点的主要是第①③两段,其中第①段是举例,第③段是从正面提出君子追求仁德时的要求,答题时,应以第③段为主要内容,并将第①段的举例抽象为理由,最终即可抽象为“君子要长期、坚定追求仁德,学生中只有个别人(颜回)比较接近这一要求”。最后将这些语句转换为自己的话,并注意扣题作答即可。

[参考译文]

①孔子说:“颜回这个人,他的心可以在长时间内不离开仁德,其余的学生则只能在短时间内做到仁而已。”

②孔子说:“仁难道离我们很远吗?只要我想达到仁,仁就来了。”

③孔子说:“君子没有一顿饭的时间背离仁德的,就是在最紧迫的时刻也必须按照仁德办事,就是在颠沛流离的时候,也一定会按仁德去办事的。”

10.B [解析]表述前后颠倒。原文的表述是“一个有核心、有级差、有组织的塔状结构,让位于一个无核心、无级差、无组织的面状结构”。

11.C [解析]强加因果,“每一个IP地址自由发声,都可能成为强大的文化媒体”不是“由编辑、审查、批准所组成的文化权力体系几近瓦解”、“优秀作品被低端文化淹没”的原因。

12.[答案]大量庸质与恶质作品(或“粗品”“低端文化产业”)败坏(妨害、影响)了读者的阅读情趣(兴趣、享受、习惯等),(2分)优秀作品的影响力大打折扣。(1分)(意思对即可)

[解析]本题考查对语句含意的理解。题目中的句子运用了比喻手法,表意较为隐含,因此,本题的解答关键在于还原“肠胃已经吃坏”“良药也都无济于事”的本义,从原文看,“肠胃已经吃坏”指读者没有阅读兴趣,“良药”指优秀作品,“无济于事”指不能感受优秀作品的魅力。

(甲)

13.[答案] BD(答对一项给3分,两项给5分)

[解析] B 项,“五曲幼溪津”的来历并非船工所讲述;D项,“不得志”“隐居”的说法于文无据。

14.[答案]①首尾呼应,点明作者的游踪;②用船工所讲述的“附会出的故事”,来反衬作者对武夷山的感悟,表现武夷山水给予人的“洗礼”作用。(每点2分,大意对即可)

[解析]本题考查对文章结构思路的理解。题干中没有“分别”二字,这说明“船工”的解说在文中的作用类似,答题时不必按两次分别作答。本文是一篇散文,文章的中心是游武夷山的感悟,根据“形散神不散”的文体特征,结合“船工”解说的具体内容,可以看出,正是这些解说引发了作者的“感”。这一点就是我们通常所说的“内容作用”,考生答题时应首先考虑这一点。当然,由于“船工”的解说共有两次,其在结构上当然会有呼应的作用,答题时也要一并考虑。

15.[答案]①武夷山风光秀丽,就像一幅古代的水墨画一样,对武夷山的游览要用“品读”的方法。②武夷山不仅风光美丽,而且富有文化内涵,有众多的文人题刻,用“读”更能准确表现这种游览的与众不同。③游览武夷山使作者有“洗礼”的感悟,这犹如读书之后的顿悟一样。(大意对即可,如有其他说法,言之成理亦可)

[解析]本题考查对文章艺术特点的赏析,同时也涉及对文章思想情感的解读。文章标题与文章的中心、作者的情感是联系在一起的,要解说标题命名的理由,显然要从文章的主要内容、中心主题以及作者的情感中寻找答案。从文章结尾两段的议论可以发现,文章的主体内容不但有武夷山水,也有武夷的人文遗迹、文化传统,作者从武夷山水中更获得了心灵的洗礼,而这些内容显然不是用眼就能“看”到的,为此,对武夷山只能用“读”的方式。

(乙)

13. [答案] BE(答对一项给3分,两项给5分)

[解析] B项,文中没有表现工作艰辛和身体虚弱。E项,文中没有提及当时动荡时局对其作品的影响。

14.[答案]画线部分都用整句,生动地描写了清华园优美的景致,(2分)反衬朱自清内心的寂寞,(1分)为下文写梦境做铺垫。(1分)

[解析]本题考查对文章艺术特点的理解和分析。文中画线句子从内容看是描写清华园的美景,从形式上看,主要运用了拟人手法,句式较整齐,语言典雅,这些都属于这段文字的特点。另外,“赏析”一般还要指出语段的作用,文中描写的清华园,是朱自清眼中所见的清华园,因此,这个语段的作用主要应是表现朱自清的心情,并与下文的描写形成呼应。

15.[答案]①《背影》从表面上看简单朴素,而实际上却是能发出极大的感动力的文章,凭借的是他对父亲的真情;②小饭馆里点的菜和情不自禁而写的小诗《我的南方》,是他真情的流露;③表现了重返北京后,更加热爱故土,思念家乡与亲人,感情真挚感人。

[解析]本题考查对文章内容的概括和分析。题干借对《背影》一文的评论,要求考生概括本文中能表现“真情”的事件,并分析其表现的真情的内容。答题时,考生可参考《背影》中以小见大表现真情的手法,从本文中找出类似的细节,如小饭馆里点菜等,并分析细节描写中所包含的情感。

16.[答案] 因此 则

[解析]本题考查词语运用。关联词填写主要要考虑前后句意,同时兼顾语体色彩。第①处填写的应是一个因果关联词,而且应是一个双音节词,以便照应其后的标点。第②处填写的关联词应表达并列关系,而且是一个单音节词。

17.[答案]示例:第十二届全运会志愿者标志的主体为由数字“12”组成的“心形”,表达了全运会不仅是充满激情的运动盛会,更是充满爱心的文化盛会。标志中部映出和平鸽与手形,寓意白鸽传播着广大志愿者的深情问候,志愿者们用双手将爱心奉献给他人和社会,体现了志愿者的工作理念。同时,白鸽与手形一同构成的“V”字形,是对本届全运会成功举办的美好祝愿。

[解析]图文转换要掌握标志主题和图形构造以及体现的寓意。该图是志愿者的标志,所以要体现出志愿者的主旨。“12”组成的心形,内部空白是鸽子的形状,这些都有一定的寓意,要分别阐释出来。

18.[答案]示例一:立法规定子女要经常回家看望问候老人,这种做法好。俗话说,“百善孝为先”,赡养老人是子女的一种伦理义务,老人得到儿女的亲情是一种伦理权利。国家的这种做法是对老年人权益的一种尊重,是给老人一份法律权利,更是对子女的一种警醒、督促,以法律推动亲情孝敬走进新时代,则是法治精神的升华。

示例二:“常回家看看”不能靠法律来强制执行。因为孝顺要发自内心,亲情要归于自然,表达孝意不能拘泥于形式。法律不可能解决所有问题,它只能规范公共领域,如果把道德法律化,只会适得其反,最终导致法律不被遵守,削弱了法律的效力。而且,不少子女不是不想常回家看看,而是因为各种客观原因,如路途迢迢、假期少、经济不宽裕等而无法常回家看看。(其他观点言之成理亦可)

[解析]本题考查语言的综合运用。题干的两则材料有内容的互释关系,第二则其实是第一则材料的原因。考生首先应确立评说的观点,从材料的主要内容看,关爱老人不具备评说的价值,只有“立法”这一角度才有争议的价值,因此,观点应表达为“赞同或反对立法规定常回家看望老人”。其次,考生应从至少两个角度阐述自己的观点,较为合理的做法是从孝道的意义和法规的作用等两个角度阐释。另外,本题属于语言运用题,表达应特别注意简明连贯,否则会影响得分。

19.略。

阶段评估检测(六)

1. (1)夫庸知其年之先后生于吾乎 (2)无案牍之劳形 (3)以手抚膺坐长叹 (4)身世浮沉雨打萍 (5)锦瑟无端五十弦(6)几处早莺争暖树(一处1分,错、漏、添字的,该处不给分)

2.D [解析] 薄:轻视。

3. B [解析] ①是对吴敏树散文的过誉之词;②是对陶公及韦、白、苏、陆闲适诗的评价;③是对柳宗元山水游记的评价。

4.C [解析] “比姚氏于吕居仁”的是吴敏树。

5.(1)他的文章不大令人满意的地方,在于可惜缺少雄健的风格、豪迈的气势。(“其”“厌”各1 分,大意1分)

(2)政务管理和军队事务,(我)都不能尽心料理,深深地对此感到愧惧。(“治”“悚”各1 分,大意1分)

[参考译文]

三月上旬,曾敬复一封信,想必已经收阅了。不久便接到您去年十二月的来信,以及大作诗文全集各五十卷,因此得知您健康平安,不胜仰慕。

大作中的古文拜读了一遍,比起以前只能看到的单篇短文,更感到卓绝。大体上说,文章处处顿挫多变,不卖弄诡奇的词汇和深奥的语句,而字字又像踩着危石而下步,落到纸面上却沉稳有力、无与伦比。其中闲适一类的文章,清新旷达,洒脱欢悦,超然世外,如《说钓》《杂说》《程日新传》《屠禹甸序》这类文章,好像翱翔在云外,俯视尘世而享有最大的欢乐。我曾经爱读陶渊明和韦应物、白居易、苏轼、陆游的闲适诗,看他们广泛采撷人情物态,妙趣横生,读来不禁神情喜悦而浑身轻松,让人想抛弃世事而追随他们去交游。但是可惜写古文的作家缺少这种恬淡闲适,只有柳宗元的山水行记,破天荒地漫游于此等境界,把外物和自我一起纳入极为理想的境域,这不是其他作家所能做到的,可现在我在您的大作中却常常能遇到这种文章。所以大作虽兼善众长,但我却格外看重这类文章。

您在给欧阳筱岑的信中,谈到桐城派时,不推重刘大櫆、姚鼐,甚至将姚氏比作(标榜派别的)吕居仁,如此批评是否稍为过分?刘氏当然确实没有超过同辈作家的成就,但姚氏却造诣深厚,自成一家,文辞典雅,意旨精深,他的那些被世人所称赞的文章,像《庄子章文库》《礼笺序》《朱竹君传》《仪郑堂记》《〈南园诗存〉序》等篇,都义理精当而文词俊秀,超世绝俗。他的文章不大令人满意的地方,在于可惜缺少雄健的风格、豪迈的气势。姚氏本来就有偏重阴柔的见解,又曾自己谦虚说才气薄弱。他评论古文也有很多极为精辟的说法,本朝的史官称他有古人所从未谈到的地方,独能发微探真。只是他一再称赞刘大櫆,这不免是出于私人交情的偏爱。总之,方苞以后,姚鼐确实应当是百年来的正宗大家,不能和刘大櫆列为同等而都加以轻视。这是我个人的浅见,敬请指正。

我返任江南,转眼已过半年,辖区之内太平安宁,雨水充沛,今年可望庄稼丰收。只是我个人精力日渐衰微,前不久病气发作,虽然已经痊愈,但视力模糊,无法恢复。政务管理和军队事务,我都不能尽心料理,深深地对此感到愧惧。我们家乡有匪寇团伙在暗中活动,益阳、龙阳等城,接连受到骚扰。这伙人四处游荡,没有职业,常想找机会兴风作浪。湖南省年年动乱,征剿他们,他们不害怕;安抚他们,却没有什么办法。即使十次迅速消灭了他们,但如果有一次迟缓拖延,那么家乡的灾祸便不堪设想,我为此而特别忧虑。

6.[答案](1)通过轻寒、晓阴、淡烟、流水、飞花、丝雨、小银钩等景物,(1分)营造了一种清幽、孤寂的氛围,(1分)表露出词人淡淡的忧愁。(1分)

(2)“轻”字传神地写出了暮春飞花飘落的形态,(1分)“细”字细腻地表现了春雨如丝的特点,(1分)词人用“飞花似梦”“细雨如愁”来表达复杂而难以琢磨的闲愁。(1分)

[解析]第(1)题考查对诗歌艺术手法的分析。本词运用了借景抒情的手法,这是考生易见的,难的是要准确地指出景物的内容和情感。另外,在缺少背景资料的情况下,考生不能随意判断词人为何而“愁”,最好是归纳为“闲愁”。第(2)题考查对语言的赏析。炼字题一般的答题步骤可分为三步:一是答出“轻”“细”所运用的手法,本题中没有明确的手法,因此可跳过此步骤;二是分析其所表现的意境特征,这往往是答题的关键,本题中,“轻”“细”所表现的应是一种清幽、孤寂的氛围;三是答出其在表达情感方面的效果,本题中,主要应点明“闲愁”。

[诗歌赏析]

此词抒写的是淡淡的春愁。它以轻淡的色调、白描的手法,十分熨帖地写出了环境氛围,即把那一腔淡淡的哀怨变为具体可感的艺术形象渗透出来,表情深婉、幽缈。

词的起调很轻,以“漠漠”二字状弥漫而上小楼的轻寒,一下子给春寒萧索的清晨带来寥廓冷落的气氛。紧接着加上“晓阴无赖似穷秋”,在凄清的背景上涂抹一层暗淡的色彩,进一步渲染了气氛之寂寞凄寒。“淡烟流水画屏幽”句,专写室内景。楼外天色阴沉,室内光景清幽,一股淡淡的春愁油然而生。下片开始转入对春愁的正面描写:飞花袅袅,飘忽不定,迷离惝恍;细雨如丝,迷迷蒙蒙,漫漫无际。作者在这里用了两个奇特的比喻:“飞花”之“轻”似“梦”、“丝雨”之“细”如“愁”。之所以“奇特”,不仅在于其喻体和喻指的恰当而新奇上,更在其反常上,而以抽象的情感喻具体的物象,说飞花似梦,细雨如愁。最后,词以“宝帘闲挂小银钩”作结,尤觉摇曳多姿。

此词以柔婉曲折之笔,写一种淡淡的闲愁,曲折传情而凄清婉美,《词则·大雅集》卷二称“宛转幽怨,温韦嫡派”。

7.[答案] AB(答对一项给3分,两项给5分)

[解析]A项,张军长并没有支持学生们的进步活动;“觉民和琴便决定筹办《利群周报》”说法有误,筹办《利群周报》的是觉慧他们。B项,“《道德经》”应是“《太上感应篇》”;“吴荪甫”应是“四小姐蕙芳”。

8.(1)宝玉使眼色阻止史湘云。(1分)史湘云生气,认为宝玉护着黛玉,收拾包裹要回家。(2分)黛玉也生气,认为宝玉使眼色是和史湘云合伙戏弄自己。(2分)

(2)诸葛亮。(1分)张飞大怒,要去屋后放一把火,把诸葛亮叫醒。(2分)刘备则拱立阶下,等诸葛亮睡醒才进去拜见。(2分)

9.[答案] (1)为政者要身体力行,凡事率先垂范,以身作则;为政者要立得正,行得正。

(2)治国的前提在于君主要严于律己。君主要治理好国家,必须端正自己本身,严格要求自己。如果自己端正,管理国政就不会有什么困难,如果自己不端正,随心所欲,为所欲为,就不可能去端正别人,其国家也无法治理。

[解析]第(1)题考查对材料内容的概括。根据题干中“为政者”的提示,我们可以找出原文对应的语句是“先之劳之”“无倦”“其身正”,将这些语句转换为自己的语言,即可得出答案。第(2)题考查对作者观点的分析。孔子的观点是为政者要“正”,支撑这一观点的理由主要有两个方面,一是正面的理由,即“正”可以“不令而行”、从政没有困难;二是从反面指出“不正”的后果,即“虽令不从”,不能“正人”。

[参考译文]

①子路问怎样管理政事。孔子说:“以身作则,使老百姓勤劳。”子路请求多讲一点。孔子说:“不要懈怠。”

②孔子说:“自身品行端正了,即使不发布命令,老百姓也会去干;如果自身不端正,即使发布命令,老百姓也不会服从。”

③孔子说:“如果端正了自身的行为,管理政事还有什么困难呢?如果不能端正自身的行为,怎能使别人端正呢?”

10.B [解析] “唯一的办法”这一说法过于绝对,不合文章原意。

11.[答案] 美和实际人生有一个距离,要见出事物本身的美,须把它摆在适当的距离之外去看。(3分)

[解析]本题考查对作者观点的概括。分析原文的结构可以发现,第①段举例,第②段过渡,举例所要论证的观点其实在第③段,将第③段的中心句提取出来,即为作者的观点。

12.[答案]①有了距离,才能使观者专心致志地欣赏形象本身的美;②有了距离,艺术才能弥补人生和自然的缺陷;③有了距离,作家才能将切身的经验和情感客观化,从而创造出艺术作品。(3分)

[解析]本题考查对文章内容的概括。筛选信息的区间在题干中有了明确提示,考生应从④至⑥段中找出各段的中心句,并适当压缩,分点作答。论述类文段的中心句,一般都是在段落的开头或结尾,考生筛选时要剔除属于解说和引申的语句,只留下概括性强的总结句。

(甲)

13.CE (答对一项给3分,两项给5分)

[解析]A项,应是表达了作者对故乡沉沦的深切忧虑;B项,“旨在说明他从小就有高远的志向”曲解文意;D项,“极大地伤害了老兵的感情”与文意不符。

14.[答案]贯穿全文的线索,使文章脉络清晰;(2分)突出了中国广大农村的田园风貌正在被毁坏,并表达了作者强烈的悲愤之情,引人深思。(2分)

[解析]本题考查对标题作用的理解。标题常见的作用主要有三个方面:一是内容上的作用,往往要结合全文的主题和作者的情感作答;二是结构上的作用,往往要考虑到标题的线索作用;三是考虑标题的表达技巧,看有没有设置悬念,引发阅读兴趣的作用。本文的标题,主要应考虑其对于情感主题的作用和结构上的线索作用。

15.[答案]①“一个人不能没有灵魂”是台湾老兵故事的概括,泥土就是老兵与故乡维系的血脉,就是他漂泊在外的灵魂的故乡。失去了故乡的泥土,老兵就失去了灵魂的寄托。(2分)②“一个人不能没有灵魂”在本文中是指一个人不能没有精神支柱,不能没有心灵的归宿。(2分)③生活中,无论我们走到哪里,都要心中有血地,都要念着自己的故乡。一个人不能没有灵魂赖以寄托的故乡,故乡的沉沦让人心痛;“一个人不能没有灵魂”,而故乡的沉沦让灵魂无处安身。(2分)

[解析]本题考查对文中重要语句的理解。答题时,首先要找出这一语句在文中的位置,并结合文章内容,先解说句子的字面意思;其次,要结合文章的主题,答出语句的内在含义;最后,还要根据题目的要求,答出自己的感悟。

(乙)

13.[答案] AC(答对一项给3分,两项给5分)

[解析]B项,应该是除教堂之外它是最高的;D项,应该是“正在成为”,“展示中国人的生活状态”;E项,应该是不但将中国的传统文化、民间文化、少数民族文化全面地推向世界,而且也注重介绍中国的当代文化创造,展现改革开放以来中国在文化建设方面的巨大成就。

14.[答案]①徐龙森的画作在比利时皇家大剧院这样一个巨大的欧洲古典建筑上空展出;②举办讲座和研讨会;③中国与比利时进行文化对话,将中国艺术和比利时艺术同台展出,或者联合演出。

[解析]本题考查对文章内容的概括,文章第一段总说了艺术节的盛况,但这些活动缺少具体的名称,不能称之为“主要活动”,考生概括时,不能以此段内容作为答题点,而应从后文的具体介绍中提取更具体的活动内容。

15.[答案]①徐龙森的画作显示出艺术在中国正在成为一种生活的体检,展示了中国人的生活状态。②举行讲座和研讨会,让欧洲观众深入了解了相关知识,并与中国文学家、艺术家互动,沟通了双方感情,弥补了文化方面的差异。③在中比文化的对话中,中比双方都能感受到文化背后的生命状态。

[解析]本题考查对文本社会价值的评价,但实质是考查对原文内容的筛选和概括。关于举办艺术节的目的和意义,散见于文章各个语段,既有作者的感悟评论,也有借中比双方有关人士的评价来间接表达,考生答题时,应首先将这些分散的语句筛选出来,然后再从对中国、对比利时、对双方文化等几个角度进行归类,分点作答。

16.[答案] (1)但(“然而”“但是”等表转折的关联词亦可)(1分) (2)禄(1分)

(3)“自然和农居生活”改为“对自然和对农居生活”(1分)

[解析]第①处填写关联词,应考虑横线后“只有”一词的提示,只能填写转折关联词。画线处考查病句修改,有一定难度,句子虽然带有关联词,但实质是一个单句,主语是“人生感叹或政治忧伤”,谓语是“得到了安息”,谓语前的成分,应当是一个状语,由此,即可看出“自然和农居生活”残缺了介词“对”。

17.[答案]近日,在中国自然科学基金委员会“纳米科技重大研究计划”小组以及中国科学院“百人计划”小组的支持下,中国科学家运用分子纳米技术打造出了一顶新颖的铂金“纳米皇冠”。

[解析] 短句变长句,先找出变成长句后的句子主干,即第一句话;然后再将其他句子变成这个句子的附加成分。

18.[答案]示例:大学生中文素养偏低是一个令人担忧的普遍现象。首先,大学生中文素养低下并非个别现象,早在中小学时期,重理轻文的现象就普遍存在,长此以往,大学生的中文基本素养可想而知。其次,大学生中文素养低下与他们过于依赖高科技电子产品也有一定关系,手机、电脑等交流媒介的智能化发展,使大学生不再需要扎实的中文基础知识,中文基础知识于是被日渐漠视,最终导致大学生的中文素养普遍偏低。

[解析]本题考查语言综合运用。材料介绍的是大学生中文基础知识缺乏的现象,由此得出的观点显然不能以“赞同”或“反对”来表述,考生要善于转换思维,从分析原因、解决问题等角度着眼,从而归纳出合理的观点,如参考答案示例,即从原因角度提炼的观点,如从解决问题的角度看,则可以得出如“增加中小学课程内容”“增加中文基础试题”等。至于理由的阐述,只要能围绕自己的观点,言之成理即可。

19.略。

阶段评估检测(七)

1.(1)言笑晏晏 (2)衣冠简朴古风存 (3)惠风和畅 (4)岸芷汀兰 (5)吟鞭东指即天涯(6)惟吾德馨(一处1分,错、漏、添字的,该处不给分)

2.C [解析] 数:责备。

3.D [解析] ②是说孙策去世后,朱治等人的举动;③表现朱治受到孙权礼遇;⑥表现朱治年老且怀念故土。

4.C [解析] 孙贲听从了朱治的建议,打消了遣送人质依附曹操的念头。

5.(1)此时刘繇担心被袁术、孙策吞并,于是(同孙策)结了仇怨。(大意对、语言通顺1分,“为”“搆”各1分)

(2)孙策赶跑刘繇以后,又往东平定会稽。(大意对、语言通顺1分,“走”“东”各1分)

[参考译文]

朱治字君理,丹杨郡故鄣县人。一开始当县吏,后来被推举为孝廉,州里又征召他为从事,追随孙坚征伐。汉灵帝中平五年,拜为司马,跟从孙坚征讨长沙、零陵、桂阳等三郡逆贼周朝、苏马等人,立下战功,孙坚上表奏请皇帝授予朱治行都尉一职。跟随孙坚在阳人一地击败董卓,进入洛阳。孙坚又上表授予朱治行督军校尉一职,特别率领步兵和骑兵,向东帮助徐州牧陶谦讨伐黄巾军。

适逢孙坚去世,朱治辅佐孙策,依附袁术。后来知道袁术毫无政德,于是劝孙策还军平定江东。当时太傅马日在寿春,征辟朱治为掾佐,又升迁为吴郡都尉。这时吴景已经在丹杨,而孙策为袁术攻打庐江,此时刘繇担心被袁术、孙策吞并,于是(同孙策)结了仇怨。而孙策一家老小都在州下,朱治于是派人从曲阿迎回太妃及孙权兄弟,用来供奉辅护(的用度),颇有恩德。朱治从钱唐准备进军到吴郡,吴郡太守许贡据守在由拳,朱治与许贡交战,大败许贡。许贡往南投靠山贼严白虎,朱治便进入吴郡,兼任太守职务。孙策赶跑刘繇以后,又往东平定会稽。

孙权十五岁时,朱治推举他为孝廉。不久孙策去世,朱治与张昭等人共同尊奉孙权。建安七年,孙权上表奏请朱治为(九真)[吴郡]太守,兼任扶义将军,以割娄、由拳、无锡、毗陵为奉邑,设置长吏。朱治征讨夷越,平定东南,擒捉黄巾军余党陈败、万秉等。黄武元年,封为毗陵侯,领郡如故。黄武二年,拜为安国将军,赐予金印紫绶,转封故鄣县。

孙权逾越位次为上将军,到他为吴王时,朱治每次进见,孙权总是亲自迎接,手拿笏板与之对拜,设宴款待,赠送赏赐,恩德敬意十分盛大,以至跟从朱治的随行小官,都能够奉礼私下拜见。朱治受到的特殊礼遇诸如此般。

当初,孙权弟弟孙翊,性子严厉急躁,喜怒恣意无常,朱治屡次责备数说,并且用道德礼义来使他明白。孙权堂兄豫章太守孙贲,女儿做了曹操的儿媳妇,等到曹操攻破荆州,威震南土,孙贲畏惧,想要遣送儿子到曹营做人质。朱治听说此事,请求往豫章会见孙贲,为他陈述利害,孙贲因此打消了(依附曹操的)念头。

孙权常慨叹朱治为王事忧虑操劳。朱治生性俭约,虽然身在富贵之位,但车马服装只用来侍奉。孙权特别优待朱治,自令他为督军御史主管下属城邑的文书,而朱治只领其中的四县租税而已。然而公族子弟及吴国四姓多出自仕郡,郡吏常以千人数,朱治大概几年就把他们全部遣送到王府去,所遣送的人数达数百人,每年进献食物给孙权,孙权报答常过于丰厚。当时丹杨地理位置较深僻,频频有奸贼反叛,而朱治又因为年纪渐老,思恋家乡风土,亲自上表孙权屯兵故鄣,安抚山越县。众多父老乡亲老朋友,没有不登门拜访的,朱治皆带领入内,与他们共同欢饮,乡亲们以此为荣耀。朱治在故鄣一年多,便回到吴都。黄武三年去世,在郡守三十一年,时年六十九岁。

6.[答案](1)开篇点题,点明时间和地点。“悠悠”“独上”“满目秋”等词,渲染了孤寂、萧索的气氛,为全诗奠定了凄凉、感伤的感情基调。(意思对即可)(3分)

(2)“压”字用得好。“压”字形象地写出了寒云之重,渲染了沉重压抑的氛围,从而表现出诗人心情的沉重和忧愁。(意思对即可,如回答“寒”字,言之成理亦可)(3分)

[解析]第(1)题考查对诗歌内容的理解。首联的作用一般可从内容和结构两个角度分析,从内容上看,首联用“怀乡访古”点明情感,用“江城”点明地点,用“秋”点明时间;从结构上看,则有渲染氛围、开篇点题、为下文抒情做铺垫的作用。第(2)题考查炼字。首先应指出炼得好的一个字,然后再结合内容分析其在表现意境、表达情感方面的效果。一般而言,炼得好的字往往是诗句中的动词或形容词。本题中,回答动词“压”最合理,如回答为形容词“寒”也可以。

[诗歌赏析]

作者在一派萧索的晚秋天气中登高临远,看到了金陵最有特征的风景:千里长江明净得如同一匹素白的绸缎,两岸苍翠的群峰好似争相聚在一起;江中的船帆在夕阳里来来去去,岸上酒家斜矗的旗招迎着西风在飘扬。极目远眺,那水天一色处的各种舟楫在淡云中时隐时现;一群白鹭在银河般的洲渚中腾空而起。如此壮丽的风光真是“画图难足”啊!为排解乡思而怀古,但往事如烟,相隔久远,难以追寻。独自踟蹰江边古城,扑入眼帘的只有萧索的秋景。晚烟凄迷中,一只水鸟孤零零地落在僻静无人的沙洲上。秋雨淅淅沥沥,江面上稀落地飘零着几片帆影,驶向归途。萧瑟的晚风,把山麓层林的红叶吹得七零八落。暗淡的寒云密布低空,阴沉沉地似乎要把古城压垮。这历史上虎踞龙盘、

人杰地灵的雄都,如今竟一派凄凉,无人过问;登临此地,谁也没有往日那种激扬奋发、踌躇满志的风情气韵。中间二联,融情于景,锻字炼句,显出作者的艺术匠心。用“萧萧”“惨惨”修饰“暮吹”与“寒云”,摹声绘色,写出了晚风凄紧,寒云惨淡,也写出了作者的心绪;“惊”运用拟人手法,既写出了秋风中“红叶”飘零的情状,也写出了诗人心头之“惊”;“压”字既形象地写出了寒云之重,也表现出诗人心情之沉重。这首诗歌的寥落沉郁,与北宋立国不久,逐渐形成积弱集贫的局面有关。时任左相的作者,对国势的日益衰微,感触必深。“故国凄凉”未必只是吊古之思,很可能是感慨国势衰微的曲折反映。

7.[答案] BE(答对一项给3分,两项给5分)

[解析]B项应是在陈姨太的提议下。E项“把她杀了”应为“把她交出去”,“把她拖到罗兰塔,交给军警”应是把她交给了隐修女居第尔看管,然后去找军警。

8.(1)哭的对象是郭嘉(郭奉孝)。(1分)曹操在败走华容道时的最后一笑,引出了关云长。(1分)曹军几经打击,此时已无力再战。(1分)无奈,曹操只得亲自哀求关羽放行。(1分)关羽感念旧日恩情,义释曹操。(1分)

(2)刘姥姥说的。(1分)对凤姐(王熙凤)说的。(1分)刘姥姥因家中生活艰难,到贾府攀亲,(1分)而凤姐又是贾府的当家,(1分)所以刘姥姥说这番话讨好凤姐,以求获得一些赏钱。(1分)

9.[答案] (1)通行天下(达之天下) 性善(人性本善)(各1分)

(2)“本”指“孝弟”,(1分)因为“孝弟”是人的天性,同时又是君子仁德的根本。(1分)对父母尽孝,对兄弟敬爱,就不会犯上作乱,最终就能促进社会和谐,提高全社会的道德水准。(2分)

[解析]第(1)题用填空的形式考查对文章内容的概括能力。题目没有要求用原文词语填空,考生可用原文,也可用自己的话概括。第(2)题考查对作者观点的概括与分析。根据文意,孔子的观点是“君子务本,本立而道生”,这里的“本”显然指“孝弟”,而“道”则指由“孝弟”而得到的良好结果,主要对应孔子所说的“不好犯上”和孟子所说的“达之天下”。

[参考译文]

①孟子说:“人不经学习就具备的,是良能;不经思考就知道的,那是良知。年幼的孩子,没有不知道要爱他们父母的;长大后,没有不知道要敬重他们兄长的。爱父母就是仁,敬兄长就是义。这没有别的原因,因为这两种人的本性能通行天下。”

②孝顺父母,顺从兄长,而喜好触犯上层统治者,这样的人是很少见的;不喜好触犯上层统治者,而喜好造反的人是没有的。君子专心致力于根本的事务,根本树立了,做人的正确原则就会产生。

10.B [解析] 根据第⑤段的表述可知,作者认为“这是一种错误的速断法”,并不赞同。

11.[答案]首先点出医治“奴性”这一痼疾,要有选择地将中国人性质中本有的优点恢复过来;(1分)其次指出应学习与借鉴异域思想文化中的优点;(1分)最后强调为文的目的,绝非请中国人做奴才。(1分)(逻辑层次清晰,意思对即可)

[解析]所谓“论述思路”,其实就是文章的层次,因此,本题的解答,首先是划分这四个段落的层次,由文中表现段落关系的关联词语可知,第⑩⑪段应同属于一个层次,而⑫⑬段则可以各自独立为一个层次。其次,考生还要概括各层次的大意,这一步,可通过筛选原文关键词来实现。

12.[答案]“满口爱国,满身国粹”,并不代表不带奴性;(1分)故意反对“洋气”也不代表不带奴性;(1分)一味向驯良(“静”)的方面发展的一般趋势,才是导致奴性的根本。(1分)

[解析]本题考查对文章语句含意的理解。文中的画线句并没有运用修辞手法,因此不存在表意隐含的特点,不宜通过“还原本义”的方法来解答。根据其在文中的位置可知,画线句是一个结论句,其表达的真意应是作者在全文所要表达的主要观点——要不要反对“洋气”,据此,我们就可以概括出语句的含意了。

(甲)

13.[答案] BE(答对一项给3分,两项给5分)

[解析] B项,是因为“不想再向任何人重复叙述我们家庭的困窘”;E项,是因为担心父亲担忧,不想加重父亲负担。

14.[答案]“我”开始时“信心十足”,后来经女老师提醒,意识到自己成绩优秀却无奈休学,十分可惜,所以心情变得“灰暗起来”,最后因为女老师的“泪珠”、安慰和叮嘱,深感女老师的真情挚爱,对女老师无比感激。(大意对即可)

[解析]本题考查对文章内容的概括。答题时,应从原文中筛选出表现心情的关键词,如第⑤段的“信心十足”,第⑥段的“灰暗”;然后结合事件的概括,答出心情变化的原因;最后形成一个完整的过程。

15.[答案](1)特色:运用比喻、借代、对比手法。(2分,没有回答“对比”不给分,其他两种手法答出一种即可)(2)作用:①抒发对女先生的感激之情,点明题旨;②升华文章主题,表达对纯洁人性的呼唤和对社会风气好转的期待。(4分,没有结合文本具体分析酌情扣分)

[解析]本题考查对文章艺术技巧的分析。表达的特色一般可从语言技巧和语言风格两个角度着眼,由于本段的语言风格没有突出的特征,因此应着重考虑语言技巧,特别是修辞手法、表现手法。画线句将现实中人们功利的做法与女先生的做法对比,并将欲望喻为“浊流”,用“大门、窗户”代指社会各阶层、各方面,从而突出了女先生的人性之美,也暗含对社会风气的某种期待。

(乙)

13.[答案] AE(答对一项给3分,两项给5分)

[解析]B项,“倒叙和插叙交替运用”有误,文中有倒叙,无插叙;C项,“详写其在民族语言文字信息处理领域的研究过程和成果”有误,文章详写其研究成果,略写其研究过程;D项,“自学成才的典型”错误。

14.[答案]①主持设计的维吾尔语字符集编码和在键盘上的键位布局、字符点阵及数据成为国家标准;②开发了30多种民族语言操作系统及应用软件;③创建了一整套维文信息处理的理论、体例以及技术体系和各个门类的信息化应用平台;④开发了文字识别、语音识别与合成系统。(每点1分,大意对即可)

[解析]本题考查筛选信息,归纳内容要点的能力。作答时首先要找准答题区域,本题的答题信息集中在七、八、九三段;其次要准确组织语言,不能照抄原文的文字,需概括文意,调整句式,重新组织语言。

15.[答案]①文章在叙述吾守尔的人生经历和研究成果之前,开篇便以设问引出“这是一个难以想象的挑战”的议论,既表现了吾守尔攻克难关的毅力,也能引发读者的阅读兴趣;②在叙述吾守尔的研究过程时,说“只有用‘拓荒’这个词,才能准确描述吾守尔的工作”,这些议论简明扼要地点出叙述的内容非同一般。(每点3分)

[解析]本题考查分析作品的文体基本特征和主要表现手法的能力。审题时要抓住题干中的“偶用议论”,找到文中“偶用议论”的实例。作答时既要指出实例,又要分析作用,注意语言通顺。

16.[答案] (1)“对于”改为“对” (2)如雷贯耳 (3)嚣(每题1分)

[解析]第(1)题考查病句修改,画线句较短,通过划分成分可发现,问题在于介词使用不当。“对于”和“对”都是介词,都表示动作行为所涉及的对象,一般情况下,二者可以通用,但当“对”含有“对待”“向”等意思时,“对”不能换成“对于”,本题就属于这种错误。第(2)题和第(3)题主要考查考生的基础知识积累,只能靠平时的阅读识记,没有答题技巧可言。

17.[答案] 濒危语言的消失(1分)将导致某些物种和生态系统的传统知识失传。(2分)(每超5字扣1分,扣完为止)

[解析]本题考查压缩语段的能力。语段结尾一句实质是中心句,考生应以这一语句为压缩的蓝本。当然,照抄原文会使字数超过要求,因此,答题时应对中心句的用词进行一定的转化,比如“小的、未书面化的”可转化为“濒危”。

18.[答案] 示例:我反对公布高考状元的做法。因为对于状元的崇拜,会让社会公众把分数摆在第一位,认为高考的成功就是人生的成功;会造成家长只重视孩子学习成绩,认为学习成绩等于一切,而忽视孩子们道德品质等素质的培养,最终导致整个社会取向的扭曲。而且,社会对状元的追捧,也会让状元对社会和自己的看法产生扭曲,这对考生以后的人生成长十分不利。(有其他观点,只要言之成理亦可)

[解析]本题考查语言的综合运用。题目明确要求对“公布高考‘状元’”发表见解,因此观点只能有两种,即赞同或反对。答题的难点在于理由的合理性和表达的简明、连贯。理由一般可从两个层面考虑,一是考虑“公布高考‘状元’”一事本身的好处或坏处,即重点说明这种做法对高考状元本人的影响;二是考虑这一做法对社会的影响。

19.略。

阶段评估检测(八)

1.(1)以手抚膺坐长叹 (2)女之耽兮 (3)纵一苇之所如 (4)是以区区不能废远 (5)山原旷其盈视(6)廉颇老矣(一处1分,错、漏、添字的,该处不给分)

2.C [解析] “烛”应为“照,照亮”。

3.A [解析]②为董生询问佟客经历的话,表现出董生的好奇心;④为董生热情邀请佟客去家里,表现出董生的诚挚;⑥是快役的行为,不是董生的行为。

4.B [解析]原文为“妻牵衣泣。生壮念顿消,遂共登楼上,寻弓觅矢,以备盗攻”。在妻子的哭泣中,董生壮念已消,两人舍弃父亲躲到楼上,寻弓觅矢是为了自卫,不是去救出父亲;最终也不是佟客仗义出手,盗贼拷打董父本身就是佟客使用幻术戏弄董生的。

5.(1)董生于是自己讲述自己喜好的东西(喜好剑术),遗憾的是不能得到奇人异士的真传。(“乃”“恨”、所字结构各1分)

(2)你应该向妻子嘱托一下身后事,我打开门替你唤起仆人。(“于妻子”“启户”“警”各1分)

[参考译文]

董生,是个好击剑的徐州人,总是一副意气风发自以为是的样子。偶然路途之中遇到一旅客,便骑驴同行。同那人谈话,那人言语豪迈;董便问他姓名,那人说是辽阳人姓佟;又问他去哪里,那人道:“我出门在外二十年,方才归来。”董生问:“您游走各方,见过很多人,可曾见到有什么奇异之人?”佟客道:“什么奇异之人?”董生便说起自己喜好击剑,遗憾的是得不到奇异之士的真传。佟客道:“奇异之人何处没有,但必须先得是个忠孝之人,奇异之士才愿意真传于他。”董生坚决表示自己是忠孝之人,随即拔出佩带宝剑舞剑吟唱自己如何怀才不遇,又乱砍路旁的小树自负地炫耀着锋利宝剑。佟客见之张须而笑,把剑借来一看。董生便递给了,他翻转把玩看了看,道:“此乃铠甲之铁铸造,被汗臭污染过,属最下等。我虽然不通剑术,但也有把剑尚还有用。”于是从衣下取出把一尺来长的短刀,便削董之佩剑,就像切个葫芦瓜果斩个马蹄般容易,应势就把剑给削断了。董生很是惊讶,忙请求把短刀给他看看,他爱不释手摸了又摸才还给佟。董生邀请佟客到家里,坚持留他多住几宿。恭敬地请教剑法,佟推说自己不懂。董生便双手按膝吹牛,佟客则只是在旁恭敬倾听而已。

夜深人静之际,忽然听到隔院传出争执喧闹之声。隔院为董生父亲居住,董生心感惊疑,便贴着墙壁凝神细听,且听见有人怒气冲冲地说:“叫你儿子立即出来受刑,我便饶过你。”不一会儿便好似有人被凌辱拷打了,那人的痛苦呻吟声不绝于耳,原来那是他的父亲。董生抓起长矛想冲过去,佟客制止道:“此去恐怕得没命活不成,还是应想个万全之策。”董生惶恐不安地向佟客请教,佟客说道:“强盗点名要抓的是你,就必定要抓到你加以残害才会甘心。你又没有别的亲人,应该先去向妻子嘱托一下身后事,我打开门替你唤起仆人。”董生应诺,进去告知了妻子,妻子拉着他的衣衫哭泣。董生救父的奋勇念头马上打消了,于是与妻子一道登上楼,寻找弓箭,准备抵御强盗攻入。急促慌张的还没准备好,就听见佟客在屋檐上笑道:“贼盗幸好已经走了。”拿灯烛一照,佟客早已幽幽然不见了踪影。董这才胆怯地徘徊着出来查视,方看到他父亲提着灯笼,从邻居家吃完酒回来,只有庭院前多了些茅草烧剩的灰烬。这才知道佟客是位奇人异士。

作者道:“忠孝,是人本该有的。以往臣子不能为君父而死的,当初也是要拿起武器勇敢赴敌的,只是都在一念之间耽误了事。昔日解缙与方孝孺相约死难与共,终还是未能实践其言。如何知道解缙发誓立约回家后,不是听进了床头人(妻子)的哭泣呢?”

郡县有一捕快,总是好些日子不回家,他妻子于是就同街巷里的无赖通奸。一天他回来,碰巧遇到少年从他家走出,很惊疑,懊恼地质问其妻。其妻不服。捕快发现了遗留在床头的少年物品,令其妻窘迫得说不出话来,只有跪地哀声乞求。捕快愤怒得很,扔了根绳子给她,逼迫她上吊自尽。妻子请求妆饰一下再死,捕快准许了。其妻便进屋里打扮梳洗,捕快自己斟上酒边喝边等,怒斥着频频催她快点。不久其妻身着艳丽服饰走了出来,涕泪交加拜道:“夫君果真忍心让我死吗?”捕快咄咄逼人火气十足,其妻返回屋中,刚刚把绳子打了结,捕快将手中杯盏一扔大呼道:“咍(具有讥讽的悔叹),回来吧!一顶绿帽子,还能压死个人不成。”于是夫妇俩人便和好如初。此人亦同解缙是一类,随之笑罢。

6.(1)[答案] 描绘了一幅春水涨满、微风吹拂、芳草碧绿、鹅儿嬉戏的清新、明净的水乡春景图。(3分)

[解析] 分析画面时要抓住词中的意象,在这首宋词的上片中,出现的意象有春水、芳草、鹅儿、微风。

(2)[答案]面对淳朴的农村生活的闲适安逸之情(或对农家生活的认同感、满足感),(1分)面对农桑丰收在望的由衷喜悦之情。(2分)

[解析]分析本词所抒发的感情时,主要注意最后两句。“秀麦连冈桑叶贱”,漫冈遍野的麦子拔穗了,桑叶也便宜了。“看看尝面收新茧”,将已熟未割的麦穗摘取下来,揉下麦粒炒干研碎,取以尝新。字里行间透着津津乐道、喜迎丰收的神情。

[诗歌赏析]

此词当是作者退居石湖期间所作,写的是苏州附近的田园风光。

“春涨一篙添水面。芳草鹅儿,绿满微风岸。”“一篙”,是指水的深度。“添水面”,有两重意思,一是水面上涨,二是水满后面积也大了。“鹅儿”,小鹅,黄中透绿,与嫩草色相似。“绿”,就是“绿柳才黄半未匀”那样的色调。春水涨满,一直浸润到岸边的芳草;芳草、鹅儿在微风中活泼地抖动、游动,那嫩嫩、和谐的色调,透出了生命的温馨与活力;微风轻轻地吹,吹绿了河岸,吹绿了河水。“画舫夷犹湾百转,横塘塔近依前远。”“画舫”,彩船。“夷犹”,犹豫迟疑,这里是指船行驶迟缓。“横塘”,在苏州西南,是个大塘。江南水乡河渠纵横,湾道也多。作者乘彩船往横塘方向游去,河道曲折多湾,画舫缓慢行进。看着前方的塔近了,其实还远。其实,作者并不急于到塔边,所以对远近并不在意,此时更使他欣悦的倒是一路的好景致。那水面上的小鹅,便很令人疼爱流连。这两句写船行,也带出了沿途风光,更带出了自己盎然的兴趣。全词欢快的气氛也由此而兴。

词的下片写到农事,视野更加开阔了。如此写,既与上片紧密相连,又避免了重复。“江国多寒农事晚。村北村南,谷雨才耕遍。”“江国”,水乡。“寒”指水冷。旱地早已种植或翻耕了,水田要晚些,江南农谚曰:“清明浸种(稻种),谷雨下秧。”所以现在“耕遍”正是时候。着一“才”字,这不紧不慢的节奏见出农事的轻松、农作的井然有序。“村北村南”耕过的水田,一片连着一片,真是“村南村北皆春水”、“绿遍山原白满川”,一派水乡风光现于读者面前,虽然农事紧张或更可说繁重,但农民们各得其乐,一切进行得有条不紊。“秀麦连冈桑叶贱,看看尝面收新茧。”“秀麦”,出穗扬花的麦子。“面”当为炒面,将已熟未割的麦穗摘取下来,揉下麦粒炒干研碎,取以尝新。这两句是写高地上的景象,虽然水稻刚刚下种,但漫冈遍野的麦子拔穗了,桑叶也便宜了,所以下面写道:“看看尝面收新茧”。“看看”,即将之意,透着津津乐道、喜迎丰收的神情。下片写田园,写农事,流露出对农家生活的认同感、满足感。

这是一首田园词,描绘出一幅清新、明净的水乡春景,散发着浓郁而恬美的农家生活气息,自始至终流露出乡村的人情淳朴,景色宁静、和谐,读了令人心醉。

7.[答案] BC(答对一项给3分,两项给5分)

[解析]B.看到爱斯梅拉达这般想见卫队长,伽西莫多就自告奋勇地去找他。C.“专注于事业”有误,吴荪甫有奸污女仆、玩弄交际花的卑劣丑行。

8.(1)因为宝玉与她开玩笑,被没有睡着的王夫人听见,王夫人因此要赶走她,她不堪羞辱,便跳井自尽。(2分)宝钗去安慰王夫人,建议多给点银子,并用自己的新衣裳给金钏妆裹。(2分)是贾环添油加醋把此事告诉贾政的。(1分)

(2)貂蝉。(1分)董卓大怒,用吕布的画戟掷吕布。(2分)后来,吕布背地里投靠王允,并与王允一起设计诛杀了董卓。(2分)

9.[答案] (1)①孔子认为实行“仁”在于推己及人;②有子认为孝弟是实行仁道的根本。(2分)

(2)儒家倡导“修身齐家治国平天下”。所以,在儒家学说中,一个人对父母是否孝顺,对兄长是否尊敬这绝不是一个个人问题,也不仅仅是一个家庭问题,而是关系到社会是否安定、天下是否太平的大问题。(4分)

[解析]第(1)题考查对材料内容的概括。可以根据“仁”字的提示,从文本中筛选出有关“仁”的语句,然后分别概括他们的看法。孔子讨论的是关于“仁”的方法,原文句子为“能近取譬,可谓仁之方也已”;有子讨论的是关于“仁”的基础,原文语句为“孝弟也者,其为仁之本与”。第(2)题考查对作者观点的分析理解。原文中,有子对“孝弟”进行了一系列的推论,即“孝弟”就不会犯上作乱,之后就可以“道生”,最后还可以成为“为仁之本”。考生应当根据原文的这一思路,用自己的话进行阐述。

[参考译文]

①孔子说:“至于仁人,就是要想自己站得住,也要帮助人家一同站得住;要想自己过得好,也要帮助人家一同过得好。凡事能就近以自己作比,而推己及人,可以说就是实行仁的方法了。”

②有子说:“孝顺父母,顺从兄长,而喜好触犯上层统治者,这样的人是很少见的;不喜好触犯上层统治者,而喜好造反的人是没有的。君子专心致力于根本的事务,根本树立了,做人的正确原则就会产生。孝顺父母、顺从兄长,这就是仁的根本啊!”

10.D [解析]A项,原文有“一些极其复杂的事情,若用比喻来加以解释,往往能收拨云见天的效果”。金斯的故事,是为了告诉我们比喻的解释效果。B项,闪电指的是构思,暴雨指的是构思的成果。C项,托尔斯泰的事例说明,构思先要有积累。

11.[答案]①抱住脑袋坐在那里向壁虚构,或者独自一人像个狂人似的口中念念有词地踱来踱去。(或者:脱离生活,一味地去“苦思冥想”)(1分)②作家决不可脱离生活,一味地去“苦思冥想”。相反,只有始终不渝地接触现实,构思才得以绽出鲜花。(2分)

[解析]本题考查对文章信息的筛选与整合。“那种样子”所对应的答题区间在于这一词语之后的解说文字中,而对于与“那种样子”对立的构思方法则紧接其后。考生只要坚持从原文筛选信息,并注意表达的概括,即可得出正确答案。

12.[答案]①人的意识之中逐步地、慢慢地积累思想、感情和记忆的印痕;②需要一个极为轻微的推动力,我们周围的世界,我们身上的一切都可能是推动力;③这一推动力引起了联想,并使构思趋于周密,复杂化。(一点1分)

[解析]本题考查对文章内容的概括。本题的难点在于文章并没有直接介绍构思的过程,而是运用比喻和举例,对构思的过程进行形象化的解说。考生要将文中比喻的本体和举例之后的总结转换为自己的语言,并根据构思的进程,分点作答。

(甲)

13.[答案] CD(答对一项给3分,两项给5分)

[解析] C项应为“以乐写哀”;D项“生活的乐观”错,应为“生活的凄苦”。

14.[答案]①妇人为孩子祈福的仪式,也娱乐了孩子们;②妇人们轻唱忧郁的歌,娱悦自己心头的寂寞。(每点2分,大意对即可)

[解析]本题考查对文章内容的概括。答题应特别注意题干的限制条件“黄昏”,从这一词语出发,可将筛选信息的范围确定为(7)~(8)段,因为从第(9)段开始,已出现“晚上”这一时间词语。另外,题目要求答出的是“不寂寞”的生活场景,答题时,应注意筛选出文中能表现长街表面上“不寂寞”的词语,如“娱乐”“娱悦”。

15.[答案]①独特的散文写法:有清晰的叙事线索,远离抽象的抒情,文字素淡朴拙;创造了一种杂糅文体,模糊了散文与小说的体式,丰富了散文的表现形式。②选材上,选取的是普通人的普通生活场景,对湘西小城镇底层人们为自己、为儿女,不畏贫贱,知天安命,艰难努力地活着的生存进行了冷静的审视和反思。③独特的审美眼光:湘西人循环往复、千年不变的无可记载的寂寞历史,有着独特的审美价值;在作者静静的叙述中呈现出的是淡淡的哀愁,带给人们的是无言的哀戚。(从两个不同的角度,每点3分;其他有创意的见解可酌情给分)

[解析]本题考查对文章艺术特色的探究。答题时,既要以本文为依据展开推理,又要跳出本文,从沈从文的艺术风格的角度来组织答案。一般来说,作家作品的艺术风格,可从选材特色、作品创作意图或情感价值、主要艺术技巧、语言特色等四个方面着眼。

(乙)

13.[答案] CD(答对一项给3分,两项给 5分)

[解析] C项是心理变态者的表现而非“原因”。D项只是“怀疑”,不是研究结果;对“杏仁核”的研究也只有动物实验的结果。

14.[答案]①介绍心理变态者的行为表现和危险性,说明心理变态研究的重要价值;②引出后文有关“心理变态”的科学研究;③通过生动的故事,增强文章的可读性和趣味性。(每点2分,任答两点即可)

[解析]本题考查对文章内容思路的理解。“作用”一般可分内容作用和结构作用。本题中,内容作用就是“介绍心理变态者的行为表现和危险性”;而“皮尔森”的故事处于文章开头,其结构作用显然是“引出下文”,答题时,这一点应结合文章具体内容进行详细解说。当然,由于开头部分还是一个“故事”,富有生动性、趣味性,也可以起到吸引读者的作用。

15.[答案]①了解心理变态的行为表现,提高自己的防范意识;②加强对心理变态成因和防治的科学研究;③加大对心理变态疗法的资助;④教育社会大众不要对心理变态者抱有偏见。(大意对即可,如有其他说法,言之成理亦可)

[解析]本题考查对文章内容的概括。由于本文的叙述较为零散,文章也没有明确指出应采取哪些措施,因此考生答题时应结合文章对心理变态者的介绍和心理变态的研究,合理推断,提炼出应对的措施。比如,由开头所讲的“皮尔森”的故事,可以说明心理变态者有一定的隐蔽性,据此可以推断,应对措施之一就是“提高防范意识”。

16.(1)随波逐流 (2)功利

17.[答案]示例:有时候,目标看上去越难达到,反而越会使人全力以赴,最终达到目标;相反,目标看上去越容易达到,就越有可能令人掉以轻心,反而无法达到。(大意对即可)

[解析]题目要求用议论性的句子揭示道理,而道理显然主要存在于两个靶的对比上:其中左边的靶是正常的,其圆心越来越小,而右边的靶却是刻意把圆心做得很大。按常理,应是右边的靶容易射中,但事实却相反,漫画的寓意就在这出人意料的结果中。

18.[答案]示例一:赞同请人“代孝”的做法。首先,请人“代孝”也是孝的一种表现形式,对比那种对父母不闻不问的可耻行为,显然更值得我们赞赏。另外,如今步入中年的人大多是独生子女,他们既要忙事业,又要照料老去的父母,还要养育嗷嗷待哺的孩子,往往分身无术,请人“代孝”可以使他们更高效率地完成自己的职责。

示例二:反对请人“代孝”的做法。首先,孝道是中华民族的优良传统,其要义是感恩和亲情,请人“代孝”尽管也能帮父母做一些事情,但已失去了孝道的内涵,不值得提倡。另外,“孝”还有许多表达的方式,一个关爱的电话,一句暖心的问候,都能温暖父母之心,根本没有必要请人“代孝”。(观点1分,理由6分。语言表达有错别字、语病等酌情扣分)

[解析]本题考查语言的综合运用能力,尤其是语言表达的简明、连贯。题目材料在本题中只是一个引子,题干要求回答的是对“在我国实行‘代孝’”的看法,当然,我国也有老龄化的问题,这是一个常识,即使没有题目的提示,考生也要了解。换句话说,我国也有如日本实行“代孝”的前提条件。当然,我国也有自己的文化与习俗,特别是关于“孝道”,而这也正是我们反对“代孝”的理由。

19.略。

  

爱华网本文地址 » http://www.aihuau.com/a/25101017/330554.html

更多阅读

2014年山东省高考语文试题浅析之二 山东省春季高考试题

2014年山东省高考语文试题浅析之二关于诗歌鉴赏题首先我把近三年来的诗歌鉴赏题置于一处:2011年阅读下面这首唐诗,回答问题。咏山泉储光羲山中有流水,借问不知名。映地为天色,飞空作雨声。转来深涧满,分出小池平。恬澹无人见,年年长

2012年高考语文试题及答案详解之一:全国大纲卷

2012年普通高等学校招生全国统一考试语文题(大纲版)详解井永战一、(12分,每小题3分)1.下列词语中加点的字,读音全都正确的一组是A.颀长(qí)悚然(sù)彰善瘅恶(dàn)韬光养晦(huì)b.人寰(huán)攫取(jué)寻瑕伺隙(xì)啮臂为盟(niè)C.抵牾(yǔ)横亘(gèn)造

声明:《一轮精品 2014高考语文一轮测评手册福建专用 :阶段评估检》为网友暧昧不动情分享!如侵犯到您的合法权益请联系我们删除